Exam 3

¡Supera tus tareas y exámenes ahora con Quizwiz!

13-10 The most frequent reason why a person would come to your office complaining of burning on urination is A. prostatitis. B. urethritis. C. a urinary tract infection. D. a sexually transmitted disease.

Answer C Urinary tract infections (UTIs) are the most common cause of burning on urination. UTIs cause approximately 7 million episodes of acute cystitis per year. Prostatitis may cause pain in the urethra or penis during or after urination. While there is pain on urination with urethritis and some STDs, these are rarer than a UTI.

97. There is a higher risk of balanitis in which of the following conditions? A. Renal insufficiency B. Diabetes mellitus C. Graves' disease D. Asthma

B) Diabetes mellitus. Balanitis is a yeast infection of the glans of the penis. Men who are not circumcised and who have diabetes mellitus are at higher risk for developing balanitis.

56. Which form of urinary incontinence is most common in elderly persons? A. stress B. urge C. iatrogenic D. overflow

B. urge

13. the causative organism of chancroid is: A) Ureaplasma species B) Chlamydia trachomatis C) Mycoplasma hominis D) Hemophilus Ducreyi

D) Hemophilus Ducreyi

6. Surgical intervention in BPH should be considered with all of the following except: A. recurrent urinary tract infection. B. bladder stones. C. persistent obstruction despite medical therapy. D. acute tubular necrosis.

D. acute tubular necrosis.

48. Patients with urethral stricture often report urine loss A. with exercise B. during the day C. associated with urgency D. as dribbling after voiding

D. as dribbling after voiding

How do you test for West Nile Virus?

Serologic testing to detect immunoglobulin M (IgM) antibodies is currently the best means of diagnosing West Nile virus infection.

69. chlamydial infections occur most frequently among women in which age group? a. < 25yrs b. 25-35yrs c. 40-50 d. > 60

a. < 25yrs

1. drugs that target the renin-angiotensin-aldosterone system are particularly beneficial in patients who have: a. diabetic nephropathy b. kidney stones c. chronic heart failure d. hypertension

a. diabetic nephropathy

17. which of the following classes of meds is LEAST likely to contribute to erectile dysfunction? a. macrolides b. H2 receptor blockers c. antihypertensives d. diuretics

a. macrolides

7. which of the following is found early in the development of chronic renal failure? a. persistent proteinuria b. elevated creatinine level c. acute uremia d. hyperkalemia

a. persistent proteinuria

10. ciprofloxacin given to tx a UTI would be CI in: a. pregnant pt b. 19yo female c. pt was HTN d. pt with pyelonephritis

a. pregnant pt

39. a 71-year-old male pt is taking dutasteride (avodart), a 5-alpha-reductase inhibitor. what effect might this have on his PSA level? a. it will increase b. it will decrease c. there is no predictable d. there will be clinically insignificant increase

b.

64. long-term calcium supplementation is recommended in postmenopausal women as its use reduces the risk of fracture by approximately: a. 25% b. 50% c. 65% d. 80%

b. 50%

133. the most common pathogen found in pts with pyelonephritis is: a. klebsiella b. e. coli c. streptococcus d. pseudomonas

b. e. coli

18. physical examination findings in LGV include: a. verruciform lesions b. lesions that fuse and create multiple draining sinuses c. a painless crater d. plaque-like lesions

b. lesions that fuse and create multiple draining sinuses

27. testicular torsion can produce: a. penile erythema b. scrotal edema c. scrotal erythema d. penile edema

b. scrotal edema

71. the incubation period for c. trachomatis is approximately: a. 24hrs b. 3d c. 7-14d d. 24d

c. 7-14d

5. an older adult has renal insufficiency, hypertension, osteoarthritis, hypothyroidism, and varicose veins. which medication should be avoided? a. tylenol b. beta blockers c. NSAIDs d. low dose ASA

c. NSAIDs

88. complications of gonococcal and chlamydial GU infection in women include all of the following except: a. PID b. tubal scarring c. acute pyelonephritis d. acute peritoneal inflammation

c. acute pyelonephritis

35. a patient is using the nuvaring as a contraceptive and asks you how long she is allowed to have the ring in her vagina at one time. as an NP, you would know that this contraceptive could typically be left in place for how many days? a. as long as 11 days b. as long as 16 days c. as long as 21 days d. as long as 26 days

c. as long as 21 days allows menstrual cycle to continue.

111. a pt with urolithiasis is more likely to: a. demonstrate RBC casts b. have chills & fever c. be of male gender d. have frequent UTIs

c. be of male gender

21. the most likely causative pathogen in a 26yo man with acute epididymitis include: a. e. coli b. enterobacteriaceae c. c. trachomatis d. psuedomonas species

c. c. trachomatis

99. the most likely causative pathogen in a 23-year-old woman with PID is: a. e. coli b. enterobacteriaceae c. c. trachomatis d. pseudomonas

c. c. trachomatis

60. in assessing a man with testicular torsion, the NP is most likely to note: a. elevated PSA level b. WBC reported in UA c. left testicle most often affected d. increased testicular blood flow by color-flow doppler US

c. left testicle most often affected

115. a 65yo caucasian male has a firm, nontender, symmetrically enlarged prostate gland on exam. his PSA is 3.9. his PSA level is influenced by: a. race only b. age & race c. prostate volume d. race,age,volume

d.

47. findings in estrogen deficiency (atrophic) vaginitis include: a. a malodorous vaginal discharge b. an increased # of lactobacilli c. a reduced # of WBCs d. a pH > 5.0

d. a pH > 5.0

91. in the person with HSV-2 infection, the virus can spread via: a. genital secretions b. oral secretions c. normal looking skin d. all of the above

d. all of the above

98. a male with gonorrhea might complain of: a. fatigue b. abd pain c. penile lesions d. dysuria

d. dysuria

14. a 14yo female has never menstruated she and her mother are concerned, what is most important for the NP to assess?

tanner stage

How is Zika transmitted?

through the bite of an infected aedes species mosquito (vector-borne) can be passed from a pregnant woman to her fetus through sex

25. is irritative voiding symptoms present in epididymitis?

yes

14-72 The most common gram-negative bacteria that causes both acute and chronic bacterial prostatitis is A. Staphylococcus aureus. B. Klebsiella. C. Escherichia coli. D. Enterobacteriaceae.

Answer C The most common gram-negative bacterium that causes both acute and chronic bacterial prostatitis is Escherichia coli. Other aerobic gram-negative bacteria include Klebsiella, Pseudomonas, Enterobacteriaceae, Proteus mirabilis, and Neisseria gonorrhoeae. Occasionally, other bacteria (Staphylococcus aureus and Streptococcus faecalis) are causes.

18. Poststreptococcal glomerulonephritis typically occurs how long following a bacterial pharyngitis infection? A. 4 to 6 days B. 1 to 2 weeks C. 3 to 4 weeks D. 2 months

B. 1 to 2 weeks

35. With fluoroquinolone use, length of antimicrobial therapy during uncomplicated pyelonephritis is typically: A. 5 days. B. 1 week. C. 2 weeks. D. 3 weeks.

B. 1 week.

58. According to recent epidemiologic studies, prostate cancer is the number cause of cancer death in men residing within the United States. A. 1 B. 2 C. 3 D. 4

B. 2

50. Factors that contribute to stress incontinence include A. detrusor overactivity B. pelvic floor weakness C. urethral stricture D. UTI

B. pelvic floor weakness

60. Risk factors for renal stones include all of the following except: A. male gender. B. vegetarian diet. C. family history of renal stones. D. obesity.

B. vegetarian diet.

138. A 13-year-old boy is brought in by his mother for a physical exam. During the genital exam, the nurse practitioner notices that the patient is at Tanner stage II. Which of the following is the best description of this Tanner stage? A. The penis is growing more in length than in width, the testicles become larger with darker scrotal skin, and the pubic hair is starting to curl B. The penis is growing more in width than in length with darker scrotal skin and more numerous pubic hairs that are darker, curly, and more coarse. C. The testicles become larger, and the skin of the scrotum starts to become darker with straight, fine, countable hairs on the genitals and the axilla D. The testicles and penile width and length are developing quickly

C) The testicles become larger, and the skin of the scrotum starts to become darker with straight, fine, countable hairs on the genitals and the axilla. Tanner stage II is when the testicles start to grow. The scrotal skin becomes thicker and starts to get darker (hyperpigmentation). The pubic hair is of a fine texture and straight, and there are few countable hairs on the genitals and the axilla.

How does intestinal anthrax manifest?

Intestinal anthrax develops 2-5 days after ingestion. Commonly presents as nonspecific abdominal pain with fever; may be associated with nausea, vomiting, malaise, anorexia, hematemesis, dysentery, and/or diarrhea Distributive or hypovolemic shock may develop depending on the severity of illness.

51. the best diagnostic test to identify the offending organism in acute bacterial prostatitis is: a. a urine culture b. a urethral culture c. antibody testing d. a urine gram stain

a. a urine culture

75. an elderly patient is diagnosed with herpes zoster ophthalmicus. which cranial nerve is affected with this disorder? a. CN VII b. V c. III d. IV

b. CN V trigeminal nerve. HZ ophthalmicus, it is infected by shingles virus.

56. a 73-year-old male pt reports that he is experiencing a weakened urinary stream, urinary frequency and urgency. he is waking up once or twice nightly to urinate. how should the NP proceed? a. DRE only b. DRE, UA, PSA c. PSA only d. PSA, DRE, BUN, Cr

b. DRE, UA, PSA

16. the most common causative organism of LGV is: a. ureaplasma genitalium b. c. trachomatis types L1 to L3 c. neisseria gonorrhoeae. d. h. ducreyi.

b. c. trachomatis types L1 to L3

96. a patient with dysuria has a urine specimen that reveals < 10,000 bacteria and numerous trichomonads. how should this pt be managed? a. ciprofloxacin and metronidazole b. metronidazole for 7 days c. ciprofloxacin for 3 days d. increased fluids and urinary tract analgesic

b. metronidazole for 7 days

6. the results of a patient's cervical cytology test indicate "atypical squamous cell of undetermined significance". Which test would you LEAST likely order as a follow-up? a. HPV testing b. a second pap smear c. US d. colposcopy

c. US US used for scanning for ovarian cysts in cases of PID. proper f/u would be all the other choices.

20. a 25yo female presents with lower abd pain, which finding would indicate the etiology as PID? a. presence of hyphae b. hematuria c. temp > 101F d. normal ESR

c. temp > 101F

111. the NP orders a 24-hour urine collection for pt with suspected kidney disease. the test measures all of the following, except: a. proteinuria b. albuminuria c. micro albuminuria d. GFR

d. GFR 24 hour urine measures creatinine clearance to assess for proteinuria, albuminuria, microalbuminuria

2. a sexually active adolescent male has a warty growth on the shaft of his penis. it is painless. this is likely: a. trichomonas b. syphilis c. herpes d. HPV

d. HPV not trichomonas bc of the discharge. syphilis produces painless lesion called chancre, ulceration with hard edge & clean, yellow base. herpes has lesions but are painful/burning. HPV produces warty growths.

72. which of the following include characteristics of a friable cervix? a. presence of a dull pain, particularly prior to menses b. a constant burning sensation c. presence of multiple polyps d. easily irritated and prone to bleeding, especially following intercourse

d. easily irritated and prone to bleeding, especially following intercourse

79. women with bacterial vaginosis typically present with: a. vulvitis b. pruritis c. dysuria d. malodorous discharge

d. malodorous discharge

An obese, 28-year-old female patient is seen in the office with acne, hirsutism and oligomenorrhea. Blood is drawn for a free androgen index and results reveal a 9.8 level. The diagnosis is PCOS, which puts the patient at additional risk for which conditions?

metabolic syndrome, infertility, type 2 DM

20. a 54 year old female presents with a small amount of vaginal bleeding of recent onset. she has been postmenopausal for approximately 2 years. what diagnosis is likely?

ovarian cancer

14-7 The most common type of genitourinary dysfunction after a transurethral resection of the prostate (TURP) is A. erectile dysfunction. B. urinary incontinence. C. retrograde ejaculation. D. decreased libido.

Answer C The most common type of genitourinary dysfunction occurring after a transurethral resection of the prostate is retrograde ejaculation (65%), followed by erectile dysfunction (15%), urinary incontinence (2.1%), and decreased libido (less than 2%).

13-13 Which of the following does NOT cause urine to appear reddish in color? A. Cascara sagrada B. Phenazopyridine (Pyridium) C. Phenytoin (Dilantin) D. Multivitamins

Answer D Multivitamins do not cause urine to appear reddish in color. Cascara sagrada, a stimulant laxative, may cause urine color to be red in alkaline urine and yellow-brown in acid urine; phenazopyridine (Pyridium), a urinary tract analgesic, will cause urine to appear orange to red; and phenytoin (Dilantin), an anticonvulsant, may cause urine to appear pink, red, or red-brown.

22. a likely causative pathogen in a 37yo man with acute epididymitis who has sex with men is: a. e. coli b. mycoplasma spp. c. chlamydia trachomatis d. acinetobacter baumannii

a. e. coli

118. a 20-year-old college athlete presents with weight loss and amenorrhea for 6mos. she had a period 7 months ago and denies sexual activity. the pts BMI has decreased to 16.4, her pulse is 52bpm and weak, and her BP is 84/54, what will the NP assess next? a. fingerstick glucose b. exercise routine c. eating habits d. medication use

a. fingerstick hypoglycemia is complication from secondary amenorrhea. exercise, eating habits, and med use would be assessed to determine differential diagnosis, after glucose test is administered.

80. in the person with HSV-2 infection, the virus can spread via contact through which of the following methods (select all that apply): a. genital secretions b. oral secretions c. intact skin

a. genital secretions b. oral secretions c. intact skin

12. which of the following is commonly found after 1 year of using DMPA (Depo-Provera)? a. weight gain b. hypermenorrhea c. acne d. rapid return of fertility when discontinued

a. weight gain

132. a 32yo female was exposed to HIV after sexual intercourse. when do majority of pts seroconvert if they are going to do so? a. within 3 mos b. within 4-6wks c. within 4 wks d. 1 wk

a. within 3 mos

21. a 50yo male comes to NP clinic for evaluation. he complains of fever 101ºF, chills, pelvic pain and dysuria. he should be diagnosed with?

acute bacterial prostatitis

17. what class of meds can be used to tx BPH and provide immediate relief?

alpha 1 blockers

9. a 26-year-old female presents with flank pain that waxes & wanes. what finding on UA should direct the NP's next action? a. blood b. nitrites c. leukocytes d. calcium

a. blood

44. during acute bacterial prostatitis the DRE usually reveals a gland described as: a. boggy b. smooth c. irregular d. cystic

a. boggy

16. all of the following findings are associated with secondary stage of an infection by the organism Treponema palldium, except: a. condylomata acuminata b. maculopapular rash of palms and soles c. lymphadenopathy d. condylomata lata

a. condylomata acuminata findings consistent with dx of syphilis, caused by T. pallidum organism, includes painless chancre, maculopapular rash of palms and soles, lymphadenopathy and condylomata lata. condylomata acuminata (genital warts) are caused by HPV and spread to others by skin-to-skin contact.

10. what effect does depo-provera have specifically on the endometrium? a. creates a thin, atrophic lining b. thickens the cervical mucus c. promotes local foreign body inflammatory responses d. causes lysis of implanted blastocysts

a. creates a thin, atrophic lining IUDs cause lysis of blastocyst before it implants or promoting local foreign body responses. depo-provera causes thickening of cervical mucus and interferes with sperm transport and penetration.

20. what is the typical initial dosage of ethinyl estradiol for combined oral contraceptives? a. dose of 35mcg or less b. 40mcg or < c. 45mcg or < d. 50mcg or <

a. dose of 35mcg or less products with < 50mcg of estrogen are "low-dose" and less likely to cause adverse events. higher doses may cause more adverse effects.

51. the recommended time to initiate screening for cervical cancer in women is: a. 3 years after first sexual intercourse b. at age 21 years c. at age 18 years d. prior to becoming sexually active

b. at age 21 years

88. which of the following meds for ED treatment has the longest half-life? a. sildenafil (viagra) b. tadalafil (cialis) c. vardenafil (levitra) d. avanafil (stendra)

b. tadalafil (cialis)

75. which HPV types are most likely to cause anorectal carcinoma? a. 1 and 3 b. 6 and 11 c. 16 and 18 d. 72 and 81

c. 16 and 18

89. what percentage of sexually active adults has serological evidence of HSV-2? a. 5.8% b. 14.5% c. 18.9% d. 35.6%

c. 18.9%

56. the average american man has an approximately ___% lifetime risk of prostate cancer & an approximately ____% likelihood of clinical disease a. 15, 5 b. 25, 8 c. 40, 10 d. 60, 15

c. 40, 10

7. DRE may be performed to assess the prostate gland. which term does NOT describe a prostate gland that may have a tumor? a. nodular b. asymmetrical c. boggy d. indurated

c. boggy this would indicate bacterial prostatitis, also edematous and tender

10. your post-menopausal patient complains of "hot flashes all the time" and insists that her quality of life is poor. you consider hormone therapy with which fact in mind? a. she will definitely be at increased risk for a CVA b. she will have less of a dietary need for red meats c. breast cancer has been linked for those taking HT d. the development of hirsutism, especially hair on the face, is something to carefully consider

c. breast cancer has been linked for those taking HT HT is CI in those with hx of breast cancer.

17. risk factors for acute glomerulonephritis include all of the following except: a. bacterial endocarditis b. goodpasture's syndrome c. crohn's disease d. polyarteritis

c. crohn's disease

98. the most common pathogen associated with pyelonephritis is: a. staphylococcus sp. b. streptococcus sp. c. h. influenzae d. e. coli

d. e. coli

116 Certain drugs have enhanced effects when combined with oral contraceptives. This is true for which of the following drugs? A. Beta blockers B. Phenytoin (Dilantin) C. Antacids D. Oral anticoagulants

Answer A Drugs that have enhanced effects when combined with oral contraceptives include beta blockers, alcohol, antidepressants, corticosteroids, and theophylline.

5-64 What is the first male secondary sexual characteristic to emerge? A. Testicular enlargement B. Growth of axillary and pubic hair C. Growth of facial and chest hair D. Deepening of the voice

Answer A Secondary sex characteristics occur in boys before puberty and may take 2 to 5 years to complete. Testicular enlargement is the first change, followed by penis and scrotal enlargement, deepening of the voice, growth of axillary and pubic hair, and finally growth of facial and chest hair.

13-20 The clinical presentation of a client with urolithiasis would include A. a gradual onset of nagging pain. B. marked leukocytes in the urine. C. a fever of 101°F or above. D. pain starting in the flank and localizing in the costovertebral angle.

Answer D The clinical presentation of a client with urolithiasis includes pain that typically starts in the flank and may localize at the costovertebral angle. The pain may radiate to the lower abdomen, groin, or perineum and is often associated with nausea and vomiting. Fever is unlikely unless there is a coexisting urinary tract infection. Hematuria may be present, but the urine should not have any leukocytes unless an infection is also present. Because the question is asked only about a client with urolithiasis and does not mention any infection, an assumption of an infection being present should not be made.

130 Jane, age 74, is incontinent of urine about 50% of the time. Which of the following is the absolute last resort in management of incontinence? A. Incontinence pads B. Intermittent catheterization C. A bladder training program (because of her age) D. Indwelling catheterization

Answer D The last resort for incontinence treatment would be indwelling catheterization. Long-term indwelling catheterization may result in an increase in infection.

9. Angiotensin-converting enzyme inhibitors can limit the progression of some forms of renal disease by: A. increasing intraglomerular pressure. B. reducing efferent arteriolar resistance. C. enhancing afferent arteriolar tone. D. increasing urinary protein excretion.

B. reducing efferent arteriolar resistance.

21. A 47 year old woman with lupus erythematosus is diagnosed with acute glomerulonephritis. Treatment options include all of the following except A. systemic corticosteroids B. systemic antimicrobials C. immune suppressors D. plasmapheresis

B. systemic antimicrobials

How does oropharyngeal anthrax manifest?

Oropharyngeal anthrax develops 2-7 days after ingestion. Presents as neck swelling and fever in the presence of an oral lesion The lesion starts as an edematous area that becomes necrotic and forms a pseudomembrane within 2 weeks. Sore throat, dysphagia, respiratory distress, and oral bleeding may also occur, depending on the lesion site. Soft-tissue edema and dramatic cervical lymph node enlargement usually follow.

23. which of the following is a reasonable tx option for a 30yo man with acute epididymitis who presents without GI upset and will be tx as an outpatient? a. PO doxycycline with IM ceftriaxone b. PO amoxicillin/clavulanate c. PO metronidazole with PO linezolid d. PO clindamycin with PO cefiximine

a. PO doxycycline with IM ceftriaxone

82. a preferred treatment option for trichomoniasis is: a. PO metronidazole b. clindamycin vaginal cream c. topical acyclovir d. oral azithromycin

a. PO metronidazole

16. which of the following drug therapies is associated with the rare complication of osteonecrosis of the jaw, especially in patients receiving cancer treatment? a. alendronate (fosamax) b. bactrim c. cipro d. amoxicillin

a. alendronate (fosamax) belongs to bisphosphonate drug class

17. which of the following is NOT a typical advantage of contraceptive rings? a. alleviation of depression s/s b. lighter periods c. fewer mood swings than oral contraceptives d. decreased period cramps

a. alleviation of depression s/s may provide fewer mood swings, it may worsen and not alleviate s/s depression and should be used with caution in pts with such condition. may lead to lighter periods and decreased cramps.

4. which of the following medications can contribute to the development of acute urinary retention in an older man with BPH? a. amitriptyline b. loratadine c. enalapril d. lorazepam

a. amitriptyline

19. treatment options for LGV include: a. doxycycline b. penicillin c. ceftriaxone d. dapsone

a. doxycycline

106. which of the following best describes lesions associated with condyloma acuminatum? a. verruciform b. plaque-like c. vesicular form d. bullous

a. verruciform

91. a 5-year-old child is assessed for dysuria and frequency. which abx would the NP prescribe upon finding urine cx positive for gram (-) bacteria? a. cipro b. bactrim DS c. levaquin d. doxycycline

b. bactrim DS appropriate for gram (-) bacteria such as E. coli. safe in pts > 2 mos. cipro and levaquin are quinolones and should not be given to < 18yrs. doxy not safe < 8 yrs.

10. according to the United States Medical Eligibility Criteria (US MEC) for Contraception Use, which of the following is a clinical condition in which the use of copper-containing IUD or levonorgestrel-releasing IUD should be approached with caution? a. uncomplicated valvular heart disease b. current PID c. hypertension d. dysmenorrhea

b. current PID

64. the incidence of pyelonephritis is: a. least common in young adults b. less common than UTI c. always associated with UTIs d. more likely in elderly males

b. less common than UTI

50. which symptom listed below might be seen in a male pt with BPH? a. pain with bearing down b. low back pain c. nocturia d. dysuria

c. nocturia

3. a patient with an IUD learns that she is pregnant. if the device is not removed, which of these complications is most likely to occur? a. placenta previa b. ectopic pregnancy c. spontaneous abortion d. abruptio palcentae

c. spontaneous abortion occurs in 50% of all users of IUD if pregnancy occurs. increased

96. the following PSA levels have been observed in a pt. what conclusion can be made following these annual readings? Year 1: 3.2 year 2: 3.8 year 3: 4.2 a. they are all WNL b. none are within normal range c. there is a steady increase that is worrisome d. there is a steady increase but not worrisome

c. there is a steady increase that is worrisome

74. treatment options for patients with condyloma acuminatum include all of the following except: a. imiquimod b. podofilox c. topical acyclovir d. cryotherapy

c. topical acyclovir

61. examples of phytoestrogens include all of the following except: a. red clover b. ginseng c. vitamin E d. soy products

c. vitamin E

19. in which of the following pts would the nuvaring be CI? a. 32yo female who had just undergone first trimester abortion b. 33yo who gave birth 8wks ago and is not breastfeeding c. 34yo who uses tampons d. 36yo who smokes

d. 36yo who smokes increased risk for thrombotic diseases. nuvaring can be started 5 days after first trimester miscarriage or abortion or after 4wks postpartum if not breastfeeding. tampons do not affect placement of nuvaring.

77. a recommended treatment for rectal gonorrhea is: a. PO amoxicillin b. PO azithromycin c. oral ciprofloxacin d. ceftriaxone injection

d. ceftriaxone injection

89. when taking a PDE-5 inhibitor, concomitant use of which medication must be avoided? a. statins b. sulfonylureas c. ACEIs d. nitrates

d. nitrates

53. which part of the prostate is readily palpable during a DRE? a. anterior lobe b. median lobe c. lateral lobes d. posterior lobe

d. posterior lobe

167. a 17yo boy reports feeling something on his left scrotum. on palpation, soft and movable blood vessels that feel like a "bag of worms" are noted underneath the scrotal skin. the testicle is not swollen or reddened. the most likely diagnosis is: a. chronic orchitis b. chronic epididymitis c. testicular torsion d. varicocele

d. varicocele palpation of varicose veins described as a "bag of worms" in scrotum is classic s/s of varicocele, an abnormal tortuosity and dilation of veins of pampiniform plexus within spermatic cord. most common on left side and may be associated with pain. occuirs in boys and young men and associated with reduced fertility. the condition is often visible only when pt is standing. chronic epididymitis and chronic orchitis are caused by bacterial infection and commonly cause burning, urinary frequency and pain. testicular torsion is an emergent condition in which the testicle becomes twisted, interrupting the blood supply to the testes; to avoid damage, the condition must be corrected within 6 hours.

27. are ulcerative lesions present in epididymitis?

no

29. is a boggy prostate present is epididymitis?

no

38. would you find scrotal swelling in a 28yo male otherwise healthy with gonorrheal urethritis?

no

39. would you find fever in a 28yo male otherwise healthy with gonorrheal urethritis?

no

COVID-19 risk factors

risk of severe COVID-19 illness. These include the following conditions: Cancer Cerebrovascular disease Chronic kidney disease COPD (chronic obstructive pulmonary disease) Diabetes mellitus, type 1 and type 2 Heart conditions (eg, heart failure, coronary artery disease, cardiomyopathies) Immunocompromised state from solid organ transplant Obesity (BMI 30 kg/m 2 or greater) Pregnancy Smoking, current or former Children with certain underlying conditions Down syndrome HIV (human immunodeficiency virus) Neurologic conditions, including dementia Overweight (BMI 25 to less than 30 kg/m 2) Other lung disease (including interstitial lung disease, pulmonary fibrosis, pulmonary hypertension) Sickle cell disease Solid organ or blood stem cell transplantation Substance use disorders Use of corticosteroids or other immunosuppressive medications Cystic fibrosis Thalassemia Asthma Hypertension Immune deficiencies Liver disease

16. which of the following contraceptives has 2 products commonly named paragard and mirena? a. diaphragm b. disposable barriers c. patch d. IUD

d. IUD

6. the most common reasons for discontinuing combined oral contraception use is breakthrough bleeding and: a. NV b. inconvenience of use c. cost d. high failure rate

b. inconvenience of use

45. what early finding may lead the provider to suspect renal artery stenosis in a 3yo male? a. decreased urine production b. increased BP c. decreased GFR d. coarctation of the aorta

b. increased BP

3-26 When should an African American man start to be screened for prostate cancer by a digital rectal examination? A. Age 60 and then yearly B. Age 40 and then every 2 years C. Never D. After discussing the latest research

3-26 Answer D The evidence is insufficient to determine whether screening for prostate cancer with prostate-specific antigen (PSA) or digital rectal exam (DRE) reduces mortality from prostate cancer. Screening tests are able to detect prostate cancer at an early stage, but it is not clear whether this earlier detection and consequent earlier treatment leads to any change in the natural history and outcome of the disease. Observational evidence shows a trend toward lower mortality for prostate cancer in some countries, but the relationship between these trends and intensity of screening is not clear and associations with screening patterns are inconsistent. The observed trends may be due to screening or to other factors, such as improved treatment. Results from two randomized trials show no effect on mortality through 7 years but are inconsistent beyond 7 to 10 years. Not all medical institutions agree on when men should begin screening (routine testing) for prostate cancer or even if a DRE should be part of the screening.

29. What does a potassium hydroxide (KOH) prep help the nurse practitioner diagnose? A. Herpes zoster infections B. Fungal infections C. Herpes simplex infections D. Viral infections

B) Fungal infections. The KOH prep test is performed to evaluate for tinea or candida (yeast) infection of the skin. In vaginal discharge, the yeast organism is outside the skin cells, so KOH is not needed to visualize it. But for skin cells, yeast is not visible unless the skin cell walls are destroyed by KOH. The test involves placing a sample of skin on a glass slide, with one or two drops of KOH (causes lysis of skin cells) and a coverslip on top. If done correctly, you can visualize the budding spores and pseudohyphae.

29. The notation of alkaline urine in a patient with a UTI may point to infection caused by A. Klebsiella species B. P. mirabilis C. E. coli D. S. saprophyticus

B. P. mirabilis

8. Tamsulosin (Flomax) is helpful in the treatment of BPH because of its effect on: A. bladder contractility. B. prostate size. C. activity at select bladder receptor sites. D. bladder pressure.

C. activity at select bladder receptor sites.

11. A 14-year-old male patient presents with acute pain in his left testicle radiating to the lower abdomen lasting for 2 hours. Which of the following is the best intervention? A. Manual detorsion B. Radioisotope scrotal scanning C. Referral for immediate surgery D. Color Doppler ultrasonography

D) Color Doppler ultrasonography. These symptoms are an indication of testicular torsion, which is a medical emergency. Since all blood for the testicle comes through the spermatic cord, the blood supply is cut off with a twist. The testicle will shrink (atrophy) if the blood supply is not restored within 6 hours. The spermatic cord needs to be untwisted (detorsion) to restore the blood supply. An equivocal diagnosis may be resolved by immediate imaging, if available. Color Doppler ultrasonography of the scrotum is preferred and needs to be performed immediately. Radioisotope scrotal scanning is also diagnostic but takes longer and is less useful. Immediate manual detorsion without imaging can be attempted during the initial examination with variable success. Because testes usually rotate inward, for detorsion the testis is rotated in an outward direction (e.g., for the left testis, detorsion is clockwise when viewed from the front, underneath the testis). More than one rotation may be needed to resolve the torsion. If detorsion fails, immediate surgery is indicated, because exploration within a few hours offers the only hope of testicular salvage. One study found that nearly three in four patients require testicle removal (orchidectomy) if surgery is delayed past 12 hours.

27. Which urine culture result is needed to confirm a UTI in an asymptomatic woman who has not had recent use of a urinary catheter? A. 102 cfu/mL or more B. 103 cfu/mL or more C. 104 cfu/mL or more D. 105 cfu/mL or more

D. 105 cfu/mL or more

10. Concerning herbal and nutritional therapies for BPH treatment, which of the following statements is false? A. The mechanism of action of the most effective and best studied products is similar to prescription medications for this condition. B. These therapies are currently considered emerging therapy by the American Urological Association. C. Major areas of concern with use of these therapies include issues of product purity and quality control. D. These therapies are safest and most effective when used with prescription medications.

D. These therapies are safest and most effective when used with prescription medications.

4. luis, 42yo male, presents with painless indurated ulcer on his anus. an inspection of his body reveals no other lesion and he cannot recall any recent illness. you prescribe the patient benzathine IM in single dose. which stage of syphilis are you most likely treating? a. primary b. secondary c. latent d. tertiary

a. primary presence of indurated ulcer without any pain/other symptoms suggests the pt is primary stage. it can be tx with single dose of benzathine penicillin G. secondary may be remedied with single dose, but typically presents with flu-like symptoms, rash and lymphadenopathy. latent syphilis is asymptomatic but can be seropositive, and the tertiary stage is generally indicated by multi-system involvement.

17. symptoms of LGV typically occur how long after contact with an infected host? a. 5-7d b. 1-4wks c. 4-6wks d. 2-3mos

b. 1-4wks

51. women who use diaphragms for contraception have an increased incidence of: a. PID b. UTI c. pregnancy d. STD

b. UTI

96. recommended comprehensive STI testing includes testing for all of the following except: a. HBV b. syphilis c. HAV d. HIV

c. HAV

83. treatment options for HSV-2 genital infection include: a. ribavirin b. indinavir c. famciclovir d. cyclosporine

c. famciclovir

94. treatment options for HSV-2 genital infection include: a. ribavirin b. indinavir c. famciclovir d. cyclosporine

c. famciclovir

29. which of the following is NOT a finding of acute bacterial prostatitis? a. fever b. dysuria c. scrotal edema d. urgency

c. scrotal edema this is an indication of epididymitis. acute bacterial prostatitis would have s/s of fever, chills, low back pain and dysuria, increased urgency or frequency of urination, nocturia.

1. a 28-year-old male patient has epididymitis. his most common complaint will be: a. burning with urination b. testicular pain c. scrotal pain d. penile discharge

c. scrotal pain usually develops over period of days. chlamydia and gonorrhea are common cause of epididymitis in men < 35yrs. occasionally it acutely develops and can have fever, chills and very ill appearing patient. burning with urination is possible if underlying cause is UTI, but this is more common in men > 35yrs.

65. treatment options for varicocele repair include all of the following except: a. open surgery b. laparoscopic surgery c. tx with a thrombolytic agent d. percutaneous embolization

c. tx with a thrombolytic agent

144. which pt is most likely to develop a UTI? a. 70yo nursing home resident b. anyone who consumes multiple carbonated beverages daily c. 60yo sexually active male d. 20yo sexually active female

d. 20yo sexually active female

39. which of the following is not a gram-negative organism? a. e. coli b. k. penumoniae c. p. mirabilis d. s. saprophyticus

d. s. saprophyticus

23. a common presentation of inguinal hernia is:

groin or abdominal pain with a scrotal mass

34. T/F, the use of COC reduces menstrual volume by approx 60% thereby reducing the risk of iron deficiency anemia

true

26. is penile discharge present in epididymitis?

yes

13-1 June, age 62 has been taking NSAIDs on a long-term basis. You know that this may result in which of the following? A. Hemodynamically induced acute renal failure B. Diabetes insipidus C. Neurogenic bladder D. Hypokalemia

13-1 Answer A For clients taking NSAIDs on a long-term basis, hemodynamically induced acute renal failure, acute interstitial nephropathy with or without nephrotic syndrome, salt and water retention, papillary necrosis and chronic renal injury, hyperkalemia, vasculitis, and glomerulitis may be found. Long-term NSAID use does not cause diabetes insipidus, a neurogenic bladder, or hypokalemia.

19-1 Joey passed a kidney stone the other day and brings it to urgent care so that you can identify it for treatment. The stone is teardrop shaped and red-orange in color. What type of stone is this? A. Calcium B. Struvite C. Uric acid D. Cystine

19-1 Answer C A uric acid stone (7% of kidney stones) appears teardrop shaped or as a flat square plate that is red-orange in color. A calcium stone (75% to 80%) may resemble RBCs or a large dumbbell in shape and is a light color. A struvite stone (15%) is flat or hexagonal in shape and is radiopaque. A cystine stone (<1%) is lemon yellow and sparkling.

3-30 When should a woman start getting Pap tests? A. When she becomes sexually active B. At age 21 C. During her first pregnancy D. Before birth control is prescribed

3-30 Answer B Women are advised to begin Pap testing at the age of 21, regardless of when they started having sexual intercourse. For a young teenager, birth control may be prescribed without first performing a pelvic exam. Earlier screening for cervical cancer may lead to unnecessary and possibly harmful treatments for an increasingly rare cancer, according to the American College of Obstetricians and Gynecologists (ACOG), the leading U.S. professional organization for obstetricians and gynecologists.

4-121 Susan is keeping a basal body temperature (BBT) graph as part of her infertility treatment. Today she shares the BBT graph with you. The BBT graph shows a nearly straight line. Which of the following is the best interpretation of Susan's BBT graph? A. The client is not ovulating. B. The client is not having intercourse. C. The client is ovulating late in her menstrual cycle. D. The client is not taking her temperature correctly.

4-121 Answer A A flat BBT graph indicates lack of ovulation. If ovulation were occurring, the BBT would rise 0.5°F to 1.0°F 24 to 48 hours after ovulation. Not having intercourse does not affect ovulation.

4-22 Sheila is pregnant and fearful of getting cervical cancer as her sister recently did. She asks about receiving the HPV vaccine. How do you respond? A. "As long as you're in your first trimester, the HPV vaccine is safe and effective." B. "Research has not shown this vaccine to be effective against preventing cervical cancer." C. "After the age of 20, the HPV vaccine is not recommended." D. "You should not receive the vaccination when you're pregnant."

4-22 Answer D HPV vaccination is not recommended for use in pregnant women. The two-dose vaccine is recommended for all adolescents (males and females) at age 11 or 12 years. HPV vaccination can be given starting at age 9 years. It is also recommended for women age 13 through 26 years of age who have not yet been vaccinated nor completed the series of vaccinations. Women who are lactating or immunocompromised may receive the vaccination.

114. a 28yo male nurse tells the employee health NP that he was tx for a UTI twice the previous year. the pt denies fever, flank pain, urethral discharge during the visit. which of the following is the best follow-up for this pt? a. refer the pt to a urologist b. prescribe the pt ofloxacin (floxin) for 2 wks instead of 1 wk c. advise the pt that he needs to void every 2 hours when awake d. refer the pt to the local ED, bc he has a very high risk of sepsis

A. refer to urology UTIs are rare in young men with normal urinary tracts. urethritis can mimic a UTI; rule out gonorrhea and chlamydia. urine cx needed before initiating abx and test of cure recommended for this group. therefore they should be referred to a urologist. may need IVP, renal US, CT scan or cystoscopy.

15) When ordering laboratory tests to confirm chancroid the NP considers that: A: concomitant infection with herpes simplex is often found. B: a disease specific serum test is available C: a wbc with differential Is indicated D: dark field examination is needed.

A: concomitant infection with herpes simplex is often found.

13-36 A factor contributing to stress incontinence is A. a decreased estrogen level. B. bladder irritation from a urinary tract infection. C. prostatic hypertrophy. D. a spinal cord lesion or trauma above S2.

Answer A A decreased estrogen level contributes to stress incontinence. Bladder irritation from a urinary tract infection contributes to urge incontinence; prostatic hypertrophy contributes to overflow incontinence; and spinal cord lesion or trauma contributes to reflex incontinence.

13-60 What is the most common cause of end-stage renal disease (ESRD)? A. Hypertensive nephropathy B. Glomerulonephritis (GN) C. Diabetic nephropathy D. Acute tubular necrosis

Answer C Diabetic nephropathy is the most common cause of ESRD. Hypertensive nephropathy is the second-most commonly occurring cause of renal failure, and glomerulonephritis is the third-most common cause. Acute tubular necrosis is a kidney disorder involving damage to the tubule cells of the kidneys, which can lead to acute kidney failure.

54. A 54-year-old white man with no obvious risk for prostate cancer opted to undergo prostate-specific antigen (PSA) screening and DRE testing. The DRE findings are normal and his PSA is 3.7 ng/mL. You recommend: A. repeating the PSA test immediately. B. repeat screening in 1 year. C. repeat screening in 2 years. D. repeat screening in 5 years

B. repeat screening in 1 year.

67. A 63-year-old man presents with abdominal pain, pain during urination, and red urine. Imaging reveals a renal stone in the ureter. An appropriate treatment option would be: A. percutaneous nephrolithotomy. B. shock wave lithotripsy. C. insertion of a nephrostomy tube. D. insertion of a bladder catheter.

B. shock wave lithotripsy.

30. is epididymoorchitis in later stages of the disease present in epididymitis?

yes

36. would you find dysuria in a 28yo male otherwise healthy with gonorrheal urethritis?

yes

37. would you find milky penile discharge in a 28yo male otherwise healthy with gonorrheal urethritis?

yes

52. Pharmacological intervention for patients with urge incontinence includes A. doxazosin (Cardura) B. tolterodine (Detrol) C. finasteride (Proscar) D. pseudoephedrine

B. tolterodine (Detrol)

15. an 80yo white male presents for an annual exam. he complains of difficulty initiating his urinary stream but denies burning and urgency with urination. on exam, he is found to have a firm, smooth, nontender and symmetrically enlarged prostate. what diagnosis is consistent with these findings?

BPH

63. Struvite stones are typically found in people: A. with type 2 diabetes. B. who live in colder climates. C. who abuse alcohol. D. with a history of kidney infections.

D. with a history of kidney infections.

77. routine pap tests can be considered for all of the following pt populations except: a. men with HIV b. men who have sex with men c. women with a hx of anogenital HPV infection d. all males under age 25yrs

d. all males under age 25yrs

14. treatment options for chancroid include all of the following except: a. azithromycin b. ciprofloxacin c. ceftriaxone d. amoxicillin

d. amoxicillin

30. if a patient experiences a painful, blistering eruption in a dermatomal distribution that resembles the following, which is the most likely diagnosis? a. erysipelas b. eczema c. xanthelasma d. herpes zoster

d. herpes zoster

64. Common symptoms of renal stones include all of the following except: A. pink, red, or brown urine. B. sharp pain in the back or lower abdomen. C. marked febrile response. D. pain while urinating.

C. marked febrile response.

22. a 22yo female states that she has multiple sex partners and inconsistently uses barrier protection. which form of birth control should the NP avoid prescribing in this pt?

IUD

43. A 68-year-old man presents with suspected bladder cancer. You consider that its most common presenting sign or symptom is: A. painful urination. B. fever and flank pain. C. painless gross hematuria. D. palpable abdominal mass.

C. painless gross hematuria.

9. Concerning BPH, which of the following statements is true? A. Digital rectal examination is accurate in diagnosing the condition. B. The use of a validated patient symptom tool is an important part of diagnosing the condition. C. Prostate size directly correlates with symptoms and bladder emptying. D. Bladder distention is usually present in early disease.

B. The use of a validated patient symptom tool is an important part of diagnosing the condition.

66. You see a 58-year-old man diagnosed with a kidney stone who reports pain primarily during urination. You consider all of the following except: A. improved hydration. B. alpha blocker use. C. prescribing a diuretic. D. analgesia use.

C. prescribing a diuretic.

31. Evidence-based factors that prevent or minimize the risk of UTIs include all of the following except: A. male gender. B. longer urethra-to-anus length in women. C. timed voiding schedule. D. zinc-rich prostatic secretions.

C. timed voiding schedule.

25. A 36-year-old afebrile woman with no health problems presents with dysuria and frequency of urination. Her urinalysis findings include results positive for nitrites and leukocyte esterase. You evaluate these results and consider that she likely has: A. purulent vulvovaginitis. B. a gram-negative UTI. C. cystitis caused by Staphylococcus saprophyticus. D. urethral syndrome.

B. a gram-negative UTI.

11. You examine a 32-year-old man with chancroid and anticipate finding: A. a verruciform lesion. B. a painful ulcer. C. a painless, crater-like lesion. D. a plaquelike lesion.

B. a painful ulcer.

13. Creatinine is best described as: A. a substance produced by the kidney. B. a product related to skeletal muscle metabolism. C. produced by the liver and filtered by the kidney. D. a by-product of protein metabolism.

B. a product related to skeletal muscle metabolism.

68. The most effective strategy for preventing renal stones is A. daily exercise B. adequate hydration C. limiting coffee consumption D. smoking cessation

B. adequate hydration

4. The use of which of the following medications can precipitate acute renal failure in a patient with bilateral renal artery stenosis? A. corticosteroids B. angiotensin II receptor antagonists C. beta-adrenergic antagonists D. cephalosporins

B. angiotensin II receptor antagonists

55. Risk factors for prostate cancer include all of the following except: A. African ancestry. B. history of genital trauma. C. family history of prostate cancer. D. high-fat diet.

B. history of genital trauma.

36. Risk factors for UTI in women include: A. postvoid wiping back to front. B. low perivaginal lactobacilli colonization. C. hot tub use. D. wearing snug-fitting pantyhose.

B. low perivaginal lactobacilli colonization.

61. Medications known to increase the risk of renal stones include all of the following except: A. hydrochlorothiazide. B. moxifloxacin. C. topiramate. D. indinavir.

B. moxifloxacin.

22. A complication of glomerulonephritis is A. type 2 diabetes B. nephrotic syndrome C. pyelonephritis D. bladder cancer

B. nephrotic syndrome

7. Finasteride (Proscar, Propecia) and dutasteride (Avodart) are helpful in the treatment of BPH because of their effect on: A. bladder contractility. B. prostate size. C. activity at select bladder receptor sites. D. bladder pressure.

B. prostate size.

46. Preferred therapy for nonmuscle-invasive bladder cancer without evidence of metastases is: A. cystectomy. B. intravesical chemotherapy as first-line, solo agent. C. transurethral resection with intravesical chemotherapy. D. systemic chemotherapy.

C. transurethral resection with intravesical chemotherapy.

133. Puberty begins during which Tanner stage? A. Tanner stage I B. Tanner stage II C. Tanner stage III D. Tanner stage IV

B) Tanner stage II. Puberty is defined as the period in life when secondary sexual characteristics begin to develop, identified as Tanner stage II for boys and girls.

45. Persistent microscopic hematuria would be the primary finding in about ___ % of individuals with bladder cancer. A. 10 B. 20 C. 30 D. 40

B. 20

11. An increase in creatinine from 1 to 2 mg/dL is typically seen with a ____ loss in renal function. A. 25% B. 50% C. 75% D. 100%

B. 50%

30. The nurse practitioner who is evaluating a patient with polycystic kidney disease will pay close attention to which area of the kidney?

C) Renal tubules. The basic functional units of the kidney are nephrons containing glomeruli and renal tubules. Polycystic kidney disease affects the tubules within the nephrons. The loop of Henle is responsible for reabsorption of sodium and not affected by polycystic kidney disease directly. Likewise, the glomerulus and Bowman's capsules are responsible for filtering blood and reabsorption back into the bloodstream and are affected by disorders such as glomerulonephritis and diabetic nephropathy.

An 80-year-old patient arrives at the primary care clinic with new changes in mental status, anorexia, and oliguria. The patient has a temperature of 99.0°F, pulse of 126, and diffuse lower abdominal pain on palpation. Which diagnosis is most likely? A. Pyelonephritis B. Cystitis C. Urosepsis D. Kidney stone

C) Urosepsis. An elderly patient with a serious urogenital infection will exhibit mental status changes, temperature within normal range, tachycardia, tachypnea, and decreased urinary output. Elderly patients are at high risk for sepsis and must be treated emergently before organ system failure occurs. Cystitis (urinary tract infection [UTI]) presents with increased urinary frequency, dysuria, and white blood cells (WBCs) and bacteria in urine, and there may be mental status changes. Pyelonephritis is a kidney infection, and the patient will present with severe flank pain, fever, malaise, and other symptoms similar to cystitis. A patient who has a kidney stone will experience colicky, intermittent pain as the stone is trying to move down the ureters and may have nausea and vomiting.

33. A 44-year-old woman presents with pyelonephritis. The report of her urinalysis is least likely to include: A. WBC casts. B. positive nitrites. C. 3+ protein. D. rare RBCs.

C. 3+ protein.

4. Noncontraceptive benefits of COC use include a decrease in all of the following except: A. iron-deficiency anemia. B. pelvic inflammatory disease (PID). C. cervicitis. D. ovarian cancer.

C. cervicitis.

34. An example of a first-line therapeutic agent for the treatment of pyelonephritis is: A. amoxicillin with clavulanate. B. trimethoprim-sulfamethoxazole. C. ciprofloxacin. D. nitrofurantoin.

C. ciprofloxacin.

28. a common side effect of thiazide diuretics is: a. prostatitis b. erectile dysfunction c. fatigue d. hyperkalemia

b. erectile dysfunction

27. syphilis may present as: a. a discharge b. rash c. painful lesion d. dysuria

b. rash

42. which of the following statements is false? a. progestin only emergency contraception can be taken as one dose or two doses b. ulipristal is available by prescription only c. progestin-only emergency contraception is usually available without prescription d. ulipristal is taken in 2 doses 12 hours apart

d. ulipristal is taken in 2 doses 12 hours apart

29. when can a woman safely conceive after discontinuing COC or LARC use? a. immediately b. after 1-2mos c. after 3-4 mos d. after 5-6 mos

a. immediately

108. which HPV types are most likely to cause genital condyloma acuminatum? a. 1, 2 and 3 b. 6 and 11 c. 16 and 18 d. 22 and 24

b. 6 and 11

3-1 When can Pap smears be safely discontinued? A. At age 80 B. At age 65 if the previous three Pap smears have been normal C. Never; they should be continued throughout life. D. After menopause or hysterectomy

3-1 Answer B The USPSTF recommends against Pap smears in women older than age 65 who have had adequate prior screening and are not otherwise at high risk for cervical cancer. At or after the age of 30, women may be screened every 3 years with cervical cytology alone or every 5 years with high-risk human papillomavirus (hrHPV) testing alone up to the age of 65 years. An exception to this is for women who are at higher risk of cervical cancer, such as women exposed to DES in utero or those who are HIV positive or immunocompromised for another reason. If a woman has had a hysterectomy, then there is a need for a Pap smear only if the cervix has been left intact. The USPSTF recommends against screening for cervical cancer in women younger than age 21 years.

3-13 Which tumor marker is specifically elevated in prostate cancer? A. Prostate cancer tumor marker (PCTM) B. Cancer antigen (CA) 125 C. Carcinoembryonic antigen (CEA) D. Prostate-specific antigen (PSA)

3-13 Answer D The tumor marker that is elevated in prostate cancer is prostate-specific antigen (PSA). Determined by a simple blood test, PSA is a tumor marker whose level in the bloodstream becomes elevated with prostate cancer, although it may also become elevated with benign prostatic hypertrophy (BPH). It may even be elevated in bike riders. Clinicians should review the research on the potential harm of performing PSAs and referral for follow-up or surgery. Ultimately, the well-informed patient should make the decision about testing. There is no prostate cancer tumor marker (PCTM). Levels of cancer antigen (CA) 125 are increased in the following cancers: epithelial ovarian, fallopian tube, endometrial, endocervical, hepatic, and pancreatic. It is also used to monitor for persistent or recurrent serous carcinoma of the ovary in the postoperative period or during chemotherapy. Measurement of the level of carcinoembryonic antigen (CEA) is used primarily for monitoring persistent, metastatic, or recurrent cancer of the colon after surgery and less frequently for breast or other cancers.

60. a female patient is 35 years old. she has never had an abnormal PAP smear and has had regular periodic screening. if she has a normal PAP smear with HPV testing today, when should she have the next cervical cancer screening? a. 5 years b. 3 years c. 2-3 years d. 1 year

a. 5 years

3-25 Screening test recommendations for HIV infection include A. all clients. B. persons getting married. C. teenagers who have been sexually active for 1 year. D. intravenous drug users and clients with high-risk behaviors.

3-25 Answer D The Centers for Disease Control and Prevention recommends that all clients with high-risk behaviors be encouraged to be screened for HIV antibodies to identify those who are already infected so that interventions can be started to further halt the spread of the virus.

3-31 Margo, age 50, is perimenopausal. She tells you she is taking dehydroepiandrosterone (DHEA) and wants to start on hormone replacement therapy (HRT). When she asks for your opinion, you tell her that A. taking both DHEA and HRT is not recommended because it is like "double dosing." B. DHEA is safe and will not affect prescribed medications. C. she will be safe as long as she takes the minimum dose of both therapies. D. DHEA has the same pharmacotherapeutic effects as HRT.

3-31 Answer A Dehydroepiandrosterone (DHEA) is a hormone that is abundant in the body and is naturally produced from cholesterol by the adrenal glands, with smaller amounts manufactured by ovaries. Taking both DHEA and hormonal replacement therapy (HRT) is not recommended because it is like "double dosing." It is not absolutely contraindicated, but because most clients do not adequately regulate the dosage of over-the-counter medications, the combination therapy may produce excessively high levels of estrogen. Additionally, because almost a third of clients today are taking some sort of herbal therapy, it is essential to ask in the history and physical what other therapies are being used.

3-53 When should you screen your male patients for testicular cancer? A. Beginning at age 15 B. At every annual exam C. Teach the patient to do the screening by TSE himself. D. Never

3-53 Answer D The USPSTF recommends against screening for testicular cancer in adolescent or adult men. We used to teach TSE screening to all men aged 15 to 35 but no longer recommend it. Certainly if a patient presents with a testicular mass, the clinician would follow up with an ultrasound.

3-59 In women with HIV infection, there is a high prevalence of additional infection from A. Chlamydia. B. syphilis. C. human papillomavirus (HPV). D. Candida.

3-59 Answer C Women with HIV infection have a high prevalence of human papillomavirus (HPV) infections. These infections are associated with an increased incidence of squamous intraepithelial lesions in HIV-seropositive women. They may also have Chlamydia, syphilis, and Candida infections, but HPV infections have a higher prevalence due to the reason above.

3-60 Which of the following definitions refers to the epidemiological term endemic? A. Outbreaks of an illness/disease that occur occasionally and are unrelated in space and time B. Presence of an event (illness or disease) at a much higher than expected rate based on past history C. Presence of an illness/disease constantly present or present at a rate that is expected based on history D. Presence of an event in epidemic proportions affecting many communities and countries in a short period of time

3-60 Answer C The definition of endemic is answer C. Option A defines sporadic; B defines epidemic; and D defines pandemic.

3-67 Most anal cancers are potentially preventable. Which of the following is a cause of anal cancer? A. Sexually transmitted infections (STIs) B. A low-fiber diet C. Hemorrhoids D. Foreign bodies used as sexual stimulants

3-67 Answer A Sexually transmitted infections (STIs) are a cause of anal cancer. For women, other risk factors for anal cancer include an increased number (greater than 10) of sexual partners, having their first sexual experience before age 16, having four or more sexual partners before age 20, and anal intercourse. For men, other risk factors include having more than 10 sexual partners and being homosexual or bisexual.

25. the frequency for cervical screening depends on the pt and her age. what is the longest recommended time interval btx cervical screens for pts who are 30-64 yo? a. 5yrs b. 3yrs c. 2 yrs d. 1 yr

a. 5yrs

4-23 Jack is 4 weeks old and has oral candidiasis because his mother had an intrauterine infection that was successfully treated with antibiotics. What would you anticipate treating Jack with? A. Vinegar and water B. Nystatin C. Fluconazole D. Antibiotics

4-23 Answer B The treatment for oral candidiasis is nystatin. A vinegar and water solution is used to rinse the breast nipples between feedings. Fluconazole is used for ductal candidiasis infection of the breast. Candidiasis is a fungal infection; therefore, antibiotics are not effective as a treatment.

4-27 Karen is taking oral contraceptive pills for fertility control. Karen calls the clinic and reports the presence of chest pain and shortness of breath. You instruct Karen to A. eat smaller meals more frequently to prevent gastric distention. B. stop taking the pills and use a nonhormonal contraceptive method. C. wait for the physician to return a telephone call to the client. D. go to the nearest emergency room immediately to be evaluated.

4-27 Answer D Shortness of breath and chest pains are potentially life-threatening complications associated with the use of oral contraceptives. This situation requires immediate attention. Waiting for the physician's return phone call will delay treatment. Changing contraceptive method and eating patterns does not address the immediate life-threatening problem.

76. which HPV types are most likely to cause condyloma acuminatum? a. 1, 2 and 3 b. 6 and 11 c. 16 and 19 d. 22 and 24

b. 6 and 11

144. terazosin (hytrin), an alpha-blocker, is used to tx which of the following conditions? a. BPH and hypertension b. chronic prostatitis and atrial fibrillation c. UTI and arrythmias d. BPH and chronic prostatitis

a. BPH and HTN alpha-blocker that relieves the s/s of BPH by relaxing muscles of bladder and prostate. lowers BP by relaxing blood vessels so that blood can flow more easily through the body.

135. The cytology (Pap smear) results for a 35-year-old woman reveal a high-grade squamous intraepithelial lesion (HSIL) of the cervix. The human papillomavirus (HPV) test is positive for the type 16 strain. Rate the three actions to take in chronological chronological order, from first to last, using the numbers 1, 2, and 3. A. ___ Colposcopy B. ___ Loop electrosurgical excision procedure (LEEP) C. ___ Cervical biopsy

A) 1, C) 2, B) 3. HPV strains 16 and 18 are highly oncogenic. This patient has cervical cancer and requires immediate biopsy, excisional treatment, and staging for cervical cancer. The correct order of these actions is as follows: (1) Refer the patient for a colposcopy, (2) obtain cervical biopsy specimens for pathologic evaluation, and (3) perform LEEP for removal of cancerous cervical tissue.

32. Which treatment will the nurse practitioner prescribe to the partner of a 22-year-old patient who has a positive nucleic acid amplification test for Chlamydia trachomatis? A. Azithromycin 1 g PO one-time dose B. Benzathine penicillin 2.4 million units IM C. Clindamycin 300 mg BID × 7 days D. Doxycycline 100 mg PO BID × 7 days

A) Azithromycin 1 g PO one-time dose. If a patient has a positive nucleic acid amplification test (NAAT) for Chlamydia trachomatis, both the patient and their sexual partner should be treated. While both azithromycin 1 g PO in a single dose or doxycycline 100 mg PO BID × 7 days are recommended regimens, azithromycin is the preferred treatment due to the compliance advantage of a single dose. Clindamycin is not recommended as it is only partially effective in eradicating C. trachomatis in men with nongonococcal urethritis. Benzathine penicillin is used to treat syphilis.

162. A 21-year-old woman who is sexually active is seen by the nurse practitioner for her annual gynecological exam. She is on birth control pills and uses condoms inconsistently. During the exam, the cervix appears inflamed with purulent discharge, but cervical motion tenderness and bimanual exam are both negative. The patient is diagnosed with mucopurulent cervicitis. The NAAT test result is positive for gonorrhea and negative for chlamydia. Which of the following is the correct treatment? A. Ceftriaxone (Rocephin) 500 mg IM × one dose B. Ceftriaxone (Rocephin) 500 mg IM × one dose plus doxycycline 100 mg PO BID × 7 days C. Ceftriaxone (Rocephin) 250 mg IM × one dose D. Ceftriaxone (Rocephin) 250 mg IM × one dose plus doxycycline 100 mg PO BID × 14 days

A) Ceftriaxone (Rocephin) 500 mg IM × one dose. The CDC treatment guideline for gonorrhea was updated in December 2020. The recommended dose is now ceftriaxone (Rocephin) 500 mg IM × one dose. Routine co-treatment for chlamydia is not recommended unless it cannot be ruled out. This patient had NAAT testing done for both. The result for gonorrhea is positive, but for chlamydia, it is negative. Since the chlamydia test result is known to be negative, there is no need to co-treat for chlamydia infection.

28. A 35-year-old smoker is being evaluated for birth control choices. The patient has a history of pelvic inflammatory disease (PID) along with an embolic episode after her last pregnancy. The nurse practitioner will recommend: A. Condoms and the vaginal sponge (Today Sponge) B. Estrogen patches C. Intrauterine device D. Depo-Provera (depot medroxyprogesterone)

A) Condoms and the vaginal sponge (Today Sponge). Contraindications for hormonal contraception include migraine headaches; cigarette smoking or obesity in women older than 35 years; history of thromboembolic disease; hypertension or vascular disease if older than 35 years of age; systemic lupus erythematosus with vascular disease, nephritis, or antiphospholipid antibodies; breastfeeding (may use progestin-only pills); hypertriglyceridemia; coronary artery disease; congestive heart failure; and strokes.

172. A nurse practitioner working in a community health clinic sees a male patient who expresses concern about starting a homosexual relationship with a new partner. All of the following are appropriate interventions, except: A. Contact their new partner to schedule HIV testing B. Educate the patient about using barrier devices during sex C. Prescribe a daily oral preexposure prophylaxis (PrEP) after an HIV test D. Schedule a follow-up appointment in 1 month

A) Contact the patient's partner to schedule HIV testing. A daily oral PrEP such as tenofovir emtricitabine is recommended as a prevention for sexually active persons who are at risk for HIV. Education on using barriers during sexual activity (condom/dental dams) is important, in addition to the PrEP protocol. Scheduling an appointment in a month is not the most appropriate action, because a plan can be initiated now. It is not appropriate ethically or legally to contact the partner. It would be appropriate to advise the patient to ask his new partner to have HIV testing.

120. The nurse practitioner is examining an 82-year-old male patient and notes that his penile gland and foreskin are inflamed. A reddened, moist macular lesion is present on the prepuce. The patient is being treated for a urinary tract infection with amoxicillin. The nurse practitioner will: A. Obtain a swab of the lesion for a potassium hydroxide (KOH) prep B. Prepare the patient for a needle biopsy of the lesion C. Obtain a serum sample for a rapid plasma reagin test (RPR) D. Prescribe 0.05% betamethasone cream BID × 4 weeks

A) Obtain a swab of the lesion for a potassium hydroxide (KOH) prep. The patient has symptoms associated with balanoposthitis. Fungal infections, specifically candida infections, are commonly associated with the use of antibiotics and are the most common identifiable infectious etiology associated with balanoposthitis. Therefore, the nurse practitioner will obtain a swab of the lesion for a KOH preparation to confirm the diagnosis of a fungal infection. There is no indication the client requires a biopsy to determine the cause of the eruption. An RPR test is used to screen for syphilis; the patient does not present with a chancre-type lesion or other findings consistent with syphilis. Prior to prescribing a 0.05% betamethasone cream, the nurse practitioner will confirm the causative agent associated with the diagnosis, as it can promote the exacerbation of a fungal infection.

38. The nurse practitioner is reviewing the laboratory results of a female patient who states that she has had one menstrual period over the last year and is experiencing problems sleeping. The patient's lab results include an estradiol level of 200 pg/mL, follicle-stimulating hormone (FSH) of 35 IU/L, and an anti-müllerian hormone (AMH) level of 0.2 ng/mL. What do the findings indicate? A. The patient is perimenopausal B. The patient is menopausal C. The patient is fertile D. The patient should be tested for ovarian cancer

A) The patient is perimenopausal. The patient's laboratory results indicate that the patient is perimenopausal. The patient's estradiol level reflects a perimenopausal state. The range of estradiol associated with perimenopause is 30 to 400 pg/mL. FSH >30 mIU/mL is indicative of decreased ovarian function, which is associated with perimenopause. An AMH measurement is predictive of ovarian reserve. The level of the AMH declines with age and is a predictor for menopause. An AMH level of <0.5 ng/mL is associated with perimenopause. The patient's laboratory results are not consistent with menopause, which is diagnosed after the menstrual period has been absent for 12 months. The patient does not require testing for ovarian cancer.

157. A 37-year-old male presents for a follow-up visit after a recent positive HIV diagnosis. Another staff member at the clinic asks the nurse practitioner if she can read the patient's chart, as the patient is the cousin of a friend. What is the responsibility of the nurse practitioner? (Select all that apply.) A. The nurse practitioner should inform the staff member of the patient's right to privacy B. The nurse practitioner should not provide the chart to the staff member C. The nurse practitioner should remind the staff member not to share the HIV status with anyone D. The nurse practitioner should ask the staff member to sign a waiver to maintain confidentiality after reviewing the chart E. The nurse practitioner should report the incident to the supervisor

A, B, E) The nurse practitioner should inform the staff member of the patient's right to privacy; the nurse practitioner should not provide the chart to the staff member; the nurse practitioner should report the incident to the supervisor. Under the Health Insurance Portability and Accountability Act (HIPAA), personal health information may not be used for purposes that are not related to healthcare. The staff member is not providing healthcare to the patient and should not have access to the chart. A supervisor should be informed, who will then report the infraction to the proper authorities. The obligation of the nurse practitioner is to protect the patient and all patient information.

12. Creatinine clearance usually: A. approximates glomerular filtration rate (GFR). B. does not change as part of normative aging. C. is greater in women compared with men. D. increases with hypotension.

A. approximates glomerular filtration rate (GFR).

59. A 64 year old woman presents with urge incontinence and has not been able to tolerate treatment with anticholinergic agents. You recommend the use of which of the following? More than one can apply A. botulinum toxin injections B. fesoterodine fumarate (Toviaz) C. mirabegron (Myrbetriq) D. finasteride (Proscar)

A. botulinum toxin injections C. mirabegron (Myrbetriq)

38. In children and the elderly, which of the following conditions can contribute to bladder instability and increase the risk of a UTI? A. constipation B. upper respiratory tract infection C. chronic diarrhea D. efficient bladder emptying

A. constipation

41. Long-term use of which medication has been possibly associated with increased risk for bladder cancer? A. cyclophosphamide B. saxagliptin C. rosuvastatin D. clopidogrel

A. cyclophosphamide

51. Factors that contribute to urge incontinence include A. detrusor overactivity B. pelvic floor weakness C. urethral stricture D. UTI

A. detrusor overactivity

57. Common adverse effects of musculotropic relaxants used in the treatment of urinary incontinence include A. dry mouth and constipation B. nausea C. headaches D. syncope

A. dry mouth and constipation

32. Hemorrhagic cystitis is characterized by: A. irritative voiding symptoms. B. persistent microscopic hematuria. C. the presence of hypertension. D. elevated creatinine and BUN levels.

A. irritative voiding symptoms.

6. A 68-year-old woman with heart failure presents with tachycardia, S3 heart sound, and basilar crackles bilaterally. Blood pressure is 90/68 mm Hg; BUN is 58 mg/dL (20.7 mmol/L); creatinine is 2.4 mg/dL (212.1 μmol/L). This clinical presentation is most consistent with: A. prerenal azotemia. B. acute glomerulonephritis. C. tubular necrosis. D. postrenal azotemia.

A. prerenal azotemia.

49. Patients with stress incontinence often report urine loss A. with lifting B. at night C. associated with a strong sensation of needing to void D. as dribbling after voiding

A. with lifting

107 A condition with the symptoms of prostatitis but without infection is A. prostatodynia. B. acute nonbacterial prostatitis. C. epididymitis. D. chemical prostatitis.

Answer A Prostatodynia is a noninfective prostate disorder that presents with pelvic pain, irritable voiding, and abnormal flow. It may be mistaken for inflammatory prostatitis. There is no adequate treatment, cure, or objective explanation.

73. which test is NOT suitable to diagnose shingles if the clinical presentation is questionable? a. CBC b. direct fluorescent antibody (DFA) c. PCR d. tzanck prep

a. CBC dx made with clinical presentation. tzanck can be used it is a rapid test to dx herpes viruses.

11. depo-provera and nuvaring share all of the following mechanisms of action EXCEPT: a. release of synthetic estrogen and progestin b. thickening of cervical mucus c. suppression of FSH d. suppression of LH

a. release of synthetic estrogen and progestin

14-47 A sexually active woman should be aware that genital herpes simplex virus A. may be transmitted to a partner or newborn even in the absence of lesions because of viral shedding. B. is suppressed during menstruation, physical or emotional stress, immunosuppression, sexual intercourse, and pregnancy. C. has recurrences that usually last the same length of time as the initial outbreak. D. requires the use of condoms only during outbreaks.

Answer A A sexually active woman should be aware that genital herpes simplex virus may be transmitted to a partner or newborn even in the absence of lesions because of viral shedding. Genital herpes may be transmitted to a partner at any time; therefore, condoms should always be used. Menstruation, physical or emotional stress, immunosuppression, sexual intercourse, and pregnancy may actually trigger herpes recurrences. Herpes recurrences usually do not last as long as the initial occurrence.

14-18 A treatment used to improve the chance of pregnancy in an infertile woman who has minimal or mild endometriosis is A. laparoscopic resection or ablation of the lesions. B. dilation and curettage. C. the use of gonadotropin-releasing hormone analogues. D. the use of birth control pills for 3 months and then an abrupt stop.

Answer A A treatment used to improve the chance of pregnancy in an infertile woman who has minimal or mild endometriosis is laparoscopic resection or ablation of the lesions. Although dilation and curettage removes tissue, it may not be the specific endometrial tissue involved. Gonadotropin-releasing hormone analogues suppress endometriosis by creating a pseudo menopause, which is not desired in a woman who wants to become pregnant.

54 Scott, age 22, has cryptorchidism that has never been treated. To what is he more susceptible than men who do not have this? A. Malignant neoplasm of the testes B. Epididymitis C. Prostate cancer D. Diverticular disease

Answer A Absence of one or both testes from the scrotum is associated with reduced fertility and increased risk of testicular germ cell tumors. These clients are more susceptible to testicular torsion and inguinal hernias.

13-34 Martha has been on peritoneal dialysis and is going to the operating room for placement of an arteriovenous (AV) fistula to begin hemodialysis. How do you respond when she asks you what an AV fistula is? A. "This is where an artery and a vein are sewn together internally." B. "This is where tubing is connecting an artery and a vein underneath the skin." C. "This is where a catheter is tunneled underneath the skin with a little cuff on the outside." D. "You'll have an exterior loop of tubing connecting an artery and a vein."

Answer A An AV fistula is an internal anastomosis of an artery to a vein. An AV graft is synthetic vessel tubing tunneled beneath the skin, connecting an artery and a vein. A dual-lumen hemodialysis catheter is an extended-use catheter, surgically tunneled under the skin with a barrier cuff. An AV shunt is an external loop of tubing connecting an artery and a vein. Each section of tubing is sutured into a vessel and brought through a skin stab wound.

13-18 An acidic urine pH favors precipitation of which type of kidney stone? A. Cystine stones B. Calcium phosphate stones C. Struvite stones D. Magnesium stones

Answer A An acidic urine pH favors precipitation of organic stones: uric acid and cystine stones. An alkaline urine pH favors the precipitation of inorganic stones: calcium phosphate and magnesium ammonium phosphate (struvite) stones.

14-62 Which glands are posterior on each side of the vaginal orifice and open onto the sides of the vestibule in the groove between the labia minora and hymen? A. Bartholin's glands B. Skene's glands C. Paraurethral glands D. Cystocele

Answer A Bartholin's (greater vestibular) glands are posterior on each side of the vaginal orifice and open onto the sides of the vestibule in the groove between the labia minor and hymen. Skene's (lesser vestibular, paraurethral) glands open onto the vestibule on each side of the urethra. Cystocele is a herniation of the posterior bladder into the anterior vagina with a primary symptom of incontinence.

14-39 Bloody penile discharge is most likely to be associated with which of the following? A. Cancer of the penis B. Herpes zoster C. Epididymitis D. Peyronie's disease

Answer A Bloody penile discharge requires close investigation, including the length of time of the discharge. Ulcerations, neoplasms, and urethritis are all common causes of bloody penile discharge. Bloody penile discharge is not usually seen in herpes zoster, epididymitis, or Peyronie's disease.

13-2 Cardiovascular failure is a major cause of which type of acute renal failure? A. Prerenal B. Intrarenal C. Postrenal D. Perirenal

Answer A Cardiovascular failure and hypovolemia are the two major causes of prerenal acute renal failure. Vascular disease, glomerulonephritis, interstitial nephritis, and acute tubular necrosis are causes of intrarenal acute renal failure. Extrarenal obstruction, intrarenal obstruction, and bladder rupture are causes of postrenal acute renal failure. There is no such thing as perirenal failure.

14-13 A history of urinary tract infections in males is often seen in men with chronic bacterial prostatitis. Other signs and symptoms of chronic bacterial prostatitis include A. irritative voiding symptoms, low back pain, and perineal pain. B. nausea and vomiting, as well as fever. C. loss of appetite and weight loss. D. irritative voiding symptoms, inability to ambulate, and fever.

Answer A Chronic bacterial prostatitis may have a variety of clinical presentations, but nausea and vomiting, loss of appetite, and weight loss, as well as an inability to ambulate, are rarely among the presenting symptoms of this disorder. Even fever is typically not present in chronic cases. Typically, there are irritative voiding symptoms that have persisted over time, low back pain, and perineal pain: any one, or all, of these symptoms may be present. Sometimes clients are completely asymptomatic, although bacteria might be present on urinalysis, and expressed prostatic secretions usually demonstrate increased numbers of leukocytes. Physical examination may be unremarkable as well, although in some cases the prostate will feel boggy or indurated. There is often a history of repeated urinary tract infections. Cystitis and/or chronic urethritis may be secondary or mimic prostatitis; however, cultures of the fractionated urine may localize the source of infection. Anal disease may share some of the symptoms of prostatitis, but physical examination should permit a distinction between the two.

14-19 What is the most common prostatitis syndrome found in males of any age? A. Nonbacterial prostatitis B. Prostatodynia C. Acute bacterial prostatitis D. Chronic bacterial prostatitis

Answer A Chronic nonbacterial prostatitis is the most common of the prostatitis syndromes. It is eight times more frequent than bacterial prostatitis. The cause is not completely understood, though atypical organisms, viruses, inflammatory processes, and autoimmune disorders have been postulated. Prostatodynia presents with signs and symptoms of prostatitis but no evidence of inflammation. Acute and chronic bacterial prostatitis are both caused by an infection from either aerobic gram-negative or gram-positive bacteria.

13-47 Which of the following accounts for half of the bladder tumors among men and one-third in women? A. Cigarette smoke, both active and passive inhalation B. Chemicals from plastic and rubber C. Chronic use of phenacetin-containing analgesic agents D. Working long hours and not voiding often

Answer A Cigarette smoke, from both active and passive inhalation, is a significant risk factor for urinary tract neoplasms and may account for half of the incidence of bladder tumors among men and a third among women. Other factors implicated in the formation of urinary tract neoplasms include chemicals and dyes used in the plastics, rubber, and cable industries; the chronic use of phenacetin-containing analgesic agents; and substances in the environment of textile workers, leather finishers, spray painters, hairdressers, and petroleum workers. The breakdown products of these chemicals and those from cigarette smoke are stored in the bladder and excreted in the urine, which causes a local influence on abnormal cell development. Working long hours with inability to void may often result in a bladder infection, not a neoplasm.

14-41 A patient presents to the practitioner with scrotal pain. The examinations of his scrotum, penis, and rectum are normal. Which of the following conditions outside of the scrotum may present as scrotal pain? A. Inguinal herniation and peritonitis B. Renal colic and cardiac ischemia C. Pancreatitis and Crohn's disease D. Polyarteritis nodosa and ulcerative colitis

Answer A Conditions outside of the scrotum that may present with scrotal pain are abdominal aortic aneurysm, inguinal herniation, pancreatitis, renal colic, peritonitis, intraperitoneal hemorrhage, and polyarteritis nodosa. Keep in mind that any client with scrotal pain should be considered to have testicular torsion until it is ruled out.

5-97 You are examining a 58-year-old man who complains of recent onset of erectile dysfunction (ED). The approach to his concern should be guided by which principle? A. There are multiple physical conditions associated with the development of ED. B. Almost all men will develop ED by the time they reach their 60s. C. Most ED is caused by hormonal imbalance. D. ED is correlated with the development of benign prostatic enlargement.

Answer A ED occurs in about one-quarter of men aged 65 and older. There are many potential causes of ED, including hormone dysfunction, vascular insufficiency, antihypertensives and other medications, and psychological issues. Sexual dysfunction is a sensitive topic that may be embarrassing or difficult for the patient to disclose. Evaluation of ED should include a detailed history and review of systems and a thorough physical examination. It is not true that almost all men will develop ED by the time they reach their 60s. Hormonal imbalance is not necessarily the most frequent cause of ED; it is most often multifactorial. ED is not directly correlated with the development of benign prostate enlargement. BPH is the most common cause of lower urinary tract symptoms (LUTS).

156 Betty states that she is infertile because of polycystic ovary syndrome (PCOS). She asks you what causes the symptoms of amenorrhea, hirsutism, acne, and obesity. You tell her they are related to A. androgen excess. B. estrogen deficiency. C. inadequate gonadotropin stimulation. D. excessive adrenocorticotropic hormone.

Answer A Excessive androgenic activity causes Betty's symptoms as described and related to PCOS.

6-112 Mark, age 26, has AIDS. He wants to know why you drew a viral load instead of a CD4 count at his last visit. What do you tell him? A. A viral load is a more accurate measure of the progression of the disease. B. CD4 counts do not contribute any information regarding diagnosis and treatment. C. A viral load and a CD4 count are similar, but a viral load test is less expensive. D. Once a person has been given a diagnosis of AIDS, the CD4 count does not change.

Answer A For clients with AIDS, a viral load test is a more accurate measure of the progression of the disease than a CD4 count. The CD4 count is a surrogate marker that provides important, but indirect, measures of the state of a client's HIV disease and immunosuppression. The prognostic information it provides is useful, but incomplete, because the CD4 count can fluctuate depending on the status of the immune system, such as when the client has a cold, or as a result of dietary intake. A CD4 count is still recommended every 6 months, but a viral load test is more accurate. A viral load test is not less expensive than a CD4 count, so the decision is not made on the basis of cost justification.

19-29 Herpetic whitlow is a herpes simplex infection of the A. fingertip. B. trunk or back. C. eyelids, periorbital area, or cornea. D. labia minora, labia majora, vagina, cervix, urethra, penis, or rectal area.

Answer A Herpetic whitlow is a herpes simplex infection of the fingertip. Lumbosacral herpes is a herpes simplex infection of the trunk or back. When it involves the eyelids, periorbital area, or cornea, it is called herpetic keratoconjunctivitis. Herpes simplex infection of the labia minora, labia majora, vagina, cervix, urethra, penis or rectal area, is referred to as genital herpes.

13-54 Which of the following is(are) associated with an elevated risk of renal cell carcinoma (RCC)? A. High levels of lead exposure B. Several episodes of kidney stones C. Frequent urinary tract infections D. A permanent indwelling urinary catheter

Answer A Higher blood lead (as well as in some studies, cadmium) levels are associated with an elevated risk of renal cell carcinoma (RCC). In one study, it is interesting that higher total serum calcium was associated with a significantly reduced risk of RCC. Several episodes of kidney stones, frequent urinary tract infections, and a permanent indwelling urinary catheter—while all inconvenient and undesirable—do not lead to an elevated risk of renal cancer.

13-45 Harvey has had type 2 DM for 1 year. Recently his serum creatinine has been elevated. Which medication do you think must be discontinued? A. Metformin (Glucophage) B. Glyburide (DiaBeta) C. Glipizide (Glucotrol) D. Glimepiride (Amaryl)

Answer A If Harvey has been on metformin (Glucophage) and his serum creatinine level is elevated, you should stop the metformin and start him on one of the other medications listed.

44. the average onset of perimenopause in north american women is between ages of: a. 35-40 b. 40-45 c. 45-50 d. 50-55

b. 40-45

14-55 If a patient presents with symptoms of benign prostatic hypertrophy, a digital rectal examination is indicated in order to A. screen for prostate or rectal malignancy. B. evaluate for hypospadias. C. rule out any neurological problems that may cause the symptoms. D. detect the presence of urethritis.

Answer A If a client has symptoms of benign prostatic hypertrophy, a digital rectal examination (DRE) is performed to detect prostate or rectal malignancy and evaluate anal sphincter tone. Hypospadias is a disorder in which the meatus of the urethra is inferiorly located on the glans. A DRE does not rule out urethritis.

14-5 If a woman is using the basal body temperature (BBT) method of birth control and does not want to become pregnant, when would you tell her to avoid unprotected intercourse? A. From the beginning of the menstrual cycle until the BBT has been elevated for 3 days B. Whenever the BBT is elevated C. Whenever the BBT is lowered D. From the end of the menstrual cycle until the BBT has been low for 5 days

Answer A If a woman is using the basal body temperature (BBT) method of birth control and does not want to become pregnant, tell her to avoid unprotected intercourse from the beginning of the menstrual cycle or at least from day 4 (the day a period starts is considered day 1 until the BBT has been elevated for 3 days). When using the BBT method, the temperature is taken daily after a minimum of 3 hours of sleep; before rising, eating, or drinking; and is recorded. The preovulatory temperatures are suppressed by estrogen, whereas postovulatory temperatures are increased under the influence of heat-inducing progesterone. Temperatures typically rise within a day or two after ovulation has occurred and remain elevated for 2 weeks until menstruation begins.

13-22 In clients receiving dialysis, which is the most common cause of end-stage renal disease in the United States? A. Diabetic nephropathy B. Chronic renal failure secondary to vascular disorders C. Acute tubular necrosis D. Kidney trauma

Answer A In clients in the United States receiving dialysis, diabetic nephropathy (Kimmelstiel-Wilson syndrome) is the most common cause of end-stage renal disease (about 25%). Both type 1 and type 2 diabetes are implicated, indicating the need for good diabetes control throughout clients' life spans. Most clients with acute tubular necrosis recover with conservative management (fluid monitoring, protein restriction, drug adjustments, and dietary and potassium control). Dialysis may become necessary; however, it is usually temporary. In clients with major renal vascular occlusive disease, vascular repair or percutaneous angioplasty has been shown to slow the progression of the disease. Kidney trauma is not an end-stage renal disease.

14-25 Jack and Jane have been married for 6 months and are unable to conceive. They ask the nurse practitioner if referral to an infertility specialist is needed. What would be the nurse practitioner's best response at this time? A. "Infertility cannot be diagnosed unless unprotected sex has occurred for at least 1 year." B. "Let's run some routine tests first; then I'll recommend someone." C. "Tell me about your sexual experiences." D. "It's usually a problem with the woman, so let's have Jane examined first."

Answer A Infertility is defined as 1 year of unprotected intercourse in which conception has not occurred. Although routine tests, such as thyroid studies, may be performed, a specialist will usually not see a couple until they have been trying to conceive for 1 year. Although you may ask how often they have been having intercourse—because once a month is certainly different from three times a week—the definition of infertility remains the same. The cause of infertility is found in the man 26% to 30% of the time, and most specialists perform a comprehensive diagnostic evaluation on both members of the couple.

14-30 Leiomyomas are found A. within the uterine wall. B. on the vaginal wall. C. within the cervix. D. on the fallopian tube.

Answer A Leiomyomas are commonly called uterine fibroids; they are the most common benign tumor of the uterus. Most are small and asymptomatic. Leiomyomas are classified by location within the uterine wall and can be subserous, submucous, and/or intramural. Rarely, a leiomyoma can be intraligamentous, cervical, or parasitic (deriving its blood supply from an organ to which it becomes attached). Most uterine fibroids are surrounded by compressed (otherwise normal) myometrium. When leiomyomas outgrow their blood supply, they can become necrotic and ulcerate.

14-54 Upon questioning your 42-year-old client about her sexual orientation, she explains she considers herself to be bisexual. You determine she needs A. mental health assessment because she is at greater risk of experiencing depression and anxiety. B. a series of vaccinations to prevent cervical cancer. C. prophylactic acyclovir. D. screening for colorectal cancer.

Answer A Lesbian, bisexual, and gay people are more likely to experience depression and anxiety at 1.5 times the prevalence than heterosexual people. Adults who identified as lesbian/gay had higher prevalence of common mental disorder when compared to heterosexuals. These effects were stronger for bisexual adults. The history does not provide information on number of sexual partners so the risk for HPV and HSV are not known.

13-15 Which diuretic acts by inhibiting sodium chloride reabsorption in the thick ascending limb of the loop of Henle? A. Furosemide (Lasix) B. Mannitol (Osmitrol) C. Hydrochlorothiazide (HydroDiuril) D. Acetazolamide (Diamox)

Answer A Loop diuretics, such as furosemide (Lasix), bumetanide (Bumex), and ethacrynic acid (Edecrin), all act to inhibit sodium chloride reabsorption in the thick ascending limb of the loop of Henle. Mannitol (Osmitrol) is an osmotic diuretic, hydrochlorothiazide (HydroDiuril) is a thiazide diuretic, and acetazolamide (Diamox) is a carbonic anhydrase inhibitor; these diuretics exert their effects through different mechanisms.

14-22 Of the following symptoms of premenstrual syndrome, the one most commonly expressed by women is A. fatigue. B. depression. C. breast tenderness. D. swelling of the extremities.

Answer A Of the symptoms listed in the question, the most commonly expressed symptom of women with premenstrual syndrome is fatigue (90%). Depression occurs about 80% of the time, breast tenderness about 85%, and swelling of the extremities about 67%. Other common symptoms include irritability (91%) and abdominal bloating (90%).

13-7 In the older adult, which physiological change affects pharmacokinetics? A. Decreased creatinine clearance B. Increased lean muscle mass C. Decreased total body fat D. Increased serum albumin level

Answer A Pharmacokinetics refers to the absorption, distribution, metabolism, and elimination of drugs and their metabolites. It is affected by the following physiological changes in the older adult: decreased creatinine clearance, decreased lean muscle mass, increased total body fat, decreased hepatic blood flow, decreased renal blood flow, decreased serum albumin level, and decreased total body water. Serum albumin levels typically decrease, not increase, in the older adult.

14-6 Which type of incontinence has an associated symptom of recurrent cystitis? A. Stress incontinence B. Urge incontinence C. Overflow incontinence D. Functional incontinence

Answer A Recurrent cystitis is an associated symptom of stress incontinence. Recurrent cystitis occurs more frequently in clients who have persistent residual urine resulting from an atonic bladder, cystocele, or diabetes mellitus. An associated symptom of urge incontinence (unstable detrusor contractions) is the inability to delay voiding long enough to reach the toilet. Neurogenic disorders are frequently associated with urge incontinence. Overflow incontinence is usually associated with benign prostatic hyperplasia in men and with neurogenic conditions in both sexes. Functional incontinence is incontinence caused by functional disabilities such as those associated with Alzheimer's disease or a cerebrovascular accident rather than any problem with structure.

5-9 Male reproductive system changes during middle and later adulthood result in which manifestation? A. Slight decrease in sperm production B. Testicular enlargement C. Decrease in estrogen-like hormone production D. Stable levels of testosterone

Answer A Reproductive system changes seen in men as they age include slightly decreased sperm production, decreased semen volume, testicular softening, and decreasing testosterone levels. Testicular enlargement is not normal and may indicate underlying pathology. Testosterone levels are not stable; they are decreasing. Estrogen-like hormone production is abnormal at any level.

127 Ms. Stanley is in your primary care office today complaining of burning on urination that has persisted for 3 days. She has no fever and no flank pain. This is the second urinary tract infection that she has reported this year, and she thinks that the infections are related to increased sexual activity. She does not use a diaphragm for contraception and has tried drinking cranberry juice to offset this problem, something she read about in a women's magazine. You order A. sulfamethoxazole and trimethoprim (TMP-SMZ) (Septra or Bactrim) 1 DS tab PO bid for 3 days. B. TMP-SMZ 1 DS tab PO bid for 7 to 14 days. C. TMP-SMZ 1 DS tab PO bid for 3 weeks. D. a single dose of TMP-SMZ 1 DS tab PO.

Answer A Sulfamethoxazole and trimethoprim (TMP-SMZ) (Septra or Bactrim) 1 DS tab PO bid for 3 days is prescribed for an acute uncomplicated urinary tract infection (UTI). TMP-SMZ 1 DS tab PO bid for 7 to 14 days is ordered for a complicated UTI. For recurrent UTIs, one Bactrim may be taken post coital. In addition, remind the woman to void immediately after intercourse.

3-58 Which group experiences more rapid age-related decreases in glomerular filtration rate (GFR) than Caucasians? A. African Americans B. Asians C. Ashkenazi Jews D. Native Americans

Answer A The GFR is affected more in older African Americans than in Caucasians. Sodium is not excreted as well by the kidneys in hypertensive African Americans who have high sodium intake, and the kidneys have approximately 20% less blood flow as a result of anatomical changes in small renal vessels. End-stage renal disease (ESRD) is three to four times more common in African Americans, Native Americans, and Mexican Americans than in Caucasians.

13-32 The aging process begins to affect the kidneys with a progressive loss of nephrons by age A. 40 years. B. 50 years. C. 60 years. D. 70 years.

Answer A The aging process begins to affect the kidneys with a progressive loss of nephron units by age 40. The kidneys lose about 20% to 25% of their mass between ages 40 and 80.

6-48 Your 68-year-old client presents with a herpes zoster rash (shingles) that radiates from the left axilla across to his spine. It is red, oozing, and painful. You tell your client A. to fill a prescription of acyclovir right away and begin using it as soon as possible. B. to fill the prescription when he can but use calamine until he can get to the drugstore. C. that steroids are the only medication that will be helpful at this time. D. not to worry if he notices some eruptions on his face.

Answer A The best outcomes occur with the immediate use of antivirals such as acyclovir, valacyclovir, and famciclovir, which should be taken until finished. Steroid creams as well as calamine and Burow's solution are helpful and provide relief for some patients but are more supportive. Clients should be instructed to alert their practitioner if they develop any lesions on their face, as ophthalmic lesions can affect the trigeminal nerve and cause blindness—a referral in this case to an ophthalmologist is warranted.

14-59 Jill would like to be fitted for a diaphragm. She has been on numerous hormones in the past and does not like the side effects. She is 40 years old and does not want to get pregnant. Her partner uses condoms, but she would like a second method "just to be sure." It is important to remember that when properly fitting a patient for a diaphragm, it should A. allow a fingertip between the anterior edge of the diaphragm rim and the inside of the pubic arch. B. be small enough to allow for vaginal expansion. C. lie snugly over the pubic arch and under the cervix. D. provide firm tension against the vaginal walls.

Answer A The diaphragm is a barrier method of contraception that may provide up to 6 hours or more of birth control. It is a dome-shaped rubber cup that has a flexible rim and is inserted into the vagina before intercourse so that the posterior rim rests in the posterior portion of the vagina and the anterior rim fits snugly behind the pubic bone. The diaphragm covers the cervix and is used in conjunction with spermicide. When fit correctly, there should be just enough space to insert one fingertip comfortably between the inside of the pubic arch and the anterior edge of the diaphragm rim.

14-60 A patient's chief complaint is blood in the urine. The nurse practitioner knows that the most common cause of gross hematuria in the male population is A. a bladder infection. B. benign prostatic hypertrophy. C. bladder tumor. D. prostatitis.

Answer A The most common cause of gross hematuria is bladder infection (22%), followed by bladder tumor (14.9%), benign prostatic hyperplasia (12.5%), and prostatitis (9%).

13-40 Which type of kidney stone is the most common? A. Calcium oxalate B. Uric acid C. Calcium phosphate D. Cystine

Answer A The most common kidney stones are calcium oxalate stones. They make up about 75% of all kidney stones and are usually less than 2 cm in diameter. Uric acid stones, which are radiolucent, make up about 5% of all stones. Calcium phosphate stones, which occur with renal tubular acidosis, make up about 5% of all stones. Cystine stones, which are hexagonal crystals, are the result of an autosomal recessive genetic trait and make up less than 1% of all kidney stones. There are actually seven types of stones made of different types of crystals: calcium oxalate; calcium phosphate; a combination of calcium oxalate and calcium phosphate; magnesium ammonium phosphate (struvite or infection stones); uric acid; cystine; and miscellaneous types, such as those that occur with drug metabolites.

87 Mrs. Graves, age 38, has been on birth control pills for approximately 15 years. She is a smoker, has a blood pressure of 110/70 mm Hg, and has lipid levels within normal limits. You advise that she should A. discontinue the birth control pills because of her smoking. B. remain on the pill because her blood pressure and lipids are within normal limits. C. remain on the pill until her follicle-stimulating hormone level is greater than 30. D. discontinue the pill because of her age.

Answer A There is an increased risk of cardiovascular disease and thromboembolic disease in women who are older than 35 years of age and who smoke while taking oral contraceptives.

13-24 Thiazide diuretics are used for the treatment of which type of renal calculi? A. Calcium phosphate and/or oxalate stones B. Struvite stones C. Uric acid stones D. Cystine stones

Answer A Thiazide diuretics, along with phosphates and calcium-binding agents, are used for calcium phosphate and oxalate stones. Antibiotic therapy is used for the urinary tract infections that occur with struvite stones; allopurinol is used for uric acid stones; and penicillamine and sodium bicarbonate are used for cystine stones.

13-37 What two dysfunctions are the most common cause of vascular access failure or loss in hemodialysis (HD) for end-stage renal disease patients? A. Thrombosis and infection B. Ischemic gangrene and ischemic neuropathy C. Aneurysms and pseudoaneurysms D. Venous hypertension and central vein stenosis

Answer A Thrombosis and infection are the two most common sources of vascular access dysfunction and loss for hemodialysis patients. All of the other answers are less common causes of vascular access problems.

14-43 Toxic shock syndrome (TSS) may be caused by which of the following? A. Tampon contamination with Staphylococcus aureus B. A short vaginal canal C. The use of superabsorbency tampons D. A urinary tract infection involving the bladder and kidneys

Answer A Toxic shock syndrome (TSS) is a potentially lethal disorder that is caused in almost all cases by absorption of one or more toxins produced by colonized Staphylococcus aureus. Several mechanisms are implicated as causative, although not proven. They include tampon contamination with S. aureus, damaged cervical and vaginal mucosa, the use of overly absorbent tampons (or the synthetic materials they are made of) for an extended period of time, absorption of bacteriostatic cervical secretions, alterations in the normal vaginal flora, mechanical blockage of menstrual fluids, and the enhanced multiplication of the organism in the menstrual efflux. The incidence of TSS has decreased steadily as a result of education, altered patterns of tampon use, and the removal of extremely high-absorbency tampons from the market. A urinary tract infection involving the bladder and kidneys does not cause TSS. The risk of getting TSS may be reduced by using a tampon with the correct absorbency for the volume of menses flow. The FDA has required tampon manufacturers to use the following absorbency ratings: light absorbency tampons, regular absorbency tampons, super absorbency tampons, super plus absorbency tampons, and ultra-absorbency tampons.

19 Urinary incontinence in older adults is widely underreported. It affects which of the following? A. 15% to 30% of those living in the community and as many as 50% of those in institutional settings B. 10% to 15% of those living in the community and 35% of those in institutional settings C. 40% of those living in the community and 60% of those in institutional settings D. 7% to 10% of those living in the community and 40% of those in institutional settings

Answer A Urinary incontinence may affect as many as 50% of older adults in institutional settings and 15% to 30% of older adults in the community and is considered to be underreported in both community and institutional settings.

13-42 Urine specific gravity is increased in clients with A. dehydration. B. diabetes insipidus. C. chronic renal failure. D. hypertension.

Answer A Urine specific gravity is increased in clients who are dehydrated; in those who have a pituitary tumor, which causes the release of excessive amounts of antidiuretic hormone; and in those with decreased renal blood flow, glycosuria, or proteinuria. The specific gravity of urine is decreased in clients who are overhydrated and in those who have diabetes insipidus or chronic renal failure. Hypertension in and of itself should not affect the urine specific gravity.

6-59 Martha is concerned because her 6-year-old daughter wets the bed almost every night. Appropriate counseling statements include which of the following? A. "Bed-wetting is not uncommon. Sometimes this condition is inherited." B. "This problem has serious repercussions for the child and must be dealt with on a variety of levels." C. "Bed-wetting is not an inherited problem." D. "A self-help group for clients may prove helpful in dealing with this recalcitrant behavior."

Answer A Usually no serious physical problem is present with nighttime enuresis, although the child may have a small or immature bladder. When counseling the parents of a child with nighttime enuresis, tell them that it is a common problem that is often inherited. Stress the idea that the parents are not at fault. A family plan for dealing with the wet bed may include deciding who strips the bed and where the sheets should go. The child needs to play a major role in this plan, and positive reinforcement should be given when the child remains dry through the night. It should not be presented to the mother as something with "serious repercussions." There is no need to send the parents or child to a support group, and the term "recalcitrant behavior" connotes judgment.

14-52 When premenstrual syndrome symptoms do not respond to dietary and nonmedical therapies, which of the following drugs might you try? A. Antidepressants B. Antihistamines C. Corticosteroids D. Anticholinergics

Answer A When premenstrual syndrome symptoms do not respond to other treatments, you may prescribe any of the following drugs to aid in alleviating the symptoms: antidepressants, such as paroxetine (Paxil), fluoxetine (Prozac), or sertraline (Zoloft) to raise the levels of serotonin; diuretics to help relieve bloating; gonadotropin-releasing hormone agonists to suppress the menstrual cycle; antianxiety agents such as alprazolam (Xanax); and an oral progesterone to help relieve bloating and moodiness.

13-61 Why do middle-aged men not incur as many urinary tract infections as middle-aged women? A. Women are more prone to urinary stasis than men. B. Men have the bacteriostatic effect of prostatic fluid and a longer urethra. C. Women have a higher urine pH. D. Men are more fastidious than women when it comes to cleaning after voiding.

Answer B Middle-aged men do not incur as many urinary tract infections as middle-aged women because men have the bacteriostatic effect of prostatic fluid and a longer urethra. However, prostatic hypertrophy, commonly associated with aging, increases the risk of cystitis for older men. Men and women are equally prone to urinary stasis and have the same urinary pH.

19-55 Bob, age 59, presents with symptoms of obstructive benign prostatic hypertrophy (BPH). He states he has a decreased force of stream, hesitancy, postvoid dribbling, a sensation of incomplete bladder emptying, the inability to voluntarily stop the urinary stream, urinary frequency, and, especially, voiding many times at night. He also has hypertension. What medication might you try first to help his BPH? A. Doxazosin (Cardura) B. Tamsulosin (Flomax) C. Alfuzosin (Uroxatral) D. Dutasteride (Avodart)

Answer A While all of these medications may be used for BPH, the easiest and first drug should be an alpha-1 blocker, such as doxazosin, as Bob has hypertension in addition to his urological symptoms. Tamsulosin and alfuzosin are a1A-adrenergic blockers, which selectively target the receptors in the prostate. Dutasteride is a 5-a-reductase inhibitor and may take up to 6 months of therapy to reduce the prostate size.

13-52 Which of the following groups is most prone to developing nephrolithiasis? A. White men B. White women C. Black men D. Black women

Answer A White men are most prone to developing nephrolithiasis. The incidence of nephrolithiasis (stone in the kidney) is three times higher in men than women, with white men having three times more attacks than black men, except for struvite stones, which occur more often in black men.

14-44 Brianne, age 24, complains of urgency, frequency, and dysuria. Your dipstick test shows no hematuria, and her urine culture shows no growth. What is your next action? A. You suspect a sexually transmitted infection, so you obtain a culture of the urethra, do a potassium hydroxide wet prep, and obtain another urine culture. B. You suspect urethra irritation, so you tell her to take showers, not bubble baths, and to wear white, dry underwear and loose-fitting clothing. C. You suspect a urinary tract infection not visible yet on culture, so you start her on sulfamethoxazole and trimethoprim (Bactrim DS). D. You suspect that the vulva is irritated. You tell her to take a relaxing shower and dry the area well and come back in 1 week if there is no improvement.

Answer A You suspect a sexually transmitted infection (STI) because although the symptoms are suspicious for a urinary tract infection (UTI), the diagnosis is not supported by the dipstick and urine culture results. Your next action for Brianne is to obtain a culture of the urethra, do a potassium hydroxide wet prep to test for bacterial vaginosis, and obtain another urine culture. Doing so is the most efficient way of treating Brianne now. Discussing her social history might help you determine which course of action is most appropriate. However, if you do only one test now and it is negative, you might have to perform another diagnostic test, thereby delaying treatment again. A diagnosis of Chlamydiainfection is accomplished by culture or smears for Gram staining, but this is expensive and takes 2 to 6 days to obtain results. Other techniques include direct immunofluorescence assay and enzyme immunoassay. Diagnosis of gonorrhea is accomplished through cultures of the discharge (urethral, endocervical, rectal, pharyngeal, or conjunctive) using a modified Thayer-Martin medium or by Gram staining to look for typical gram-negative intracellular diplococci. Diagnosis of herpes simplex viruses is accomplished by the enzyme-linked immunosorbent assay technique or viral cultures. Another, less reliable method of diagnosis consists of serological antibody testing. The diagnosis of human papillomavirus infection is made by colposcopy. Trichomoniasis is diagnosed by pH that, as in bacterial vaginosis, is greater than 4.5, and by a microscopic finding of flagellated motile organisms resembling whips that are larger than white blood cells. By treating the symptoms as just a UTI or irritation, you could be ignoring the true problem, giving an STI time to spread. Medicating with antibiotics without identifying a definitive organism leads to antibiotic resistance.

8 You should be suspicious of the accuracy of the results of a positive protein dipstick test that you have done in the office under which of the following circumstances? A. The urine concentration is very diluted. B. The urine is very concentrated. C. The urine has a very high sugar content. D. The urine has blood in it.

Answer B A concentrated specimen could give an abnormally high value. A 24-hour urine collection would give a more definitive answer.

19-7 Marjorie has several renal calculi. You are recommending that she see a urologist to have a lithonephrotomy done. What does this procedure involve? A. Crushing the calculi under direct visualization using a lithotriptoscope B. Incision of the kidney to remove the calculi C. Incision of the bladder or ureter to remove the calculi D. Ultrasound waves applied outside the body to crush the calculi

Answer B A lithonephrotomy is an incision of the kidney to remove the calculi. Lithotripsy involves crushing the calculi under direct visualization using a lithotriptoscope. A lithotomy is an incision of the bladder or ureter to remove the calculi. Percutaneous ultrasonic lithotripter involves ultrasound waves applied to the outside of the body to crush the calculi.

13-14 Jill, age 40, has had urinary incontinence for several months and wants to have an evaluation to determine if any therapy will be beneficial. After having an initial pelvic examination, she has a postvoid residual catheterization. A residual volume of more than how many milliliters is abnormal? A. 10 mL B. 30 mL C. 50 mL D. 100 mL

Answer B A postvoid residual catheterization volume of more than 30 mL is abnormal.

19-39 You diagnose Minnie, age 72, with urge urinary incontinence (UUI). You want to put her on a medication that will reduce the muscle spasms of the bladder and urinary tract. Which medication might you use? A. Desmopressin B. Oxybutynin C. Baclofen D. Indomethacin

Answer B All of the medications may be used for UUI. Oxybutynin is an anticholinergic/antispasmodic that can reduce muscle spasms of the bladder and urinary tract. Desmopressin is an antidiuretic that acts on the kidneys to reduce the flow of urine. Baclofen is a muscle relaxant that may reduce urge frequency. Indomethacin is a prostaglandin synthesis inhibitor and, as a nonsteroidal anti-inflammatory drug, works by blocking the action of prostaglandin.

19-46 The most common sexually transmitted infection is A. genital human papillomavirus (HPV). B. chlamydia. C. gonorrhea. D. syphilis.

Answer B All of the medications may be used for UUI. Oxybutynin is an anticholinergic/antispasmodic that can reduce muscle spasms of the bladder and urinary tract. Desmopressin is an antidiuretic that acts on the kidneys to reduce the flow of urine. Baclofen is a muscle relaxant that may reduce urge frequency. Indomethacin is a prostaglandin synthesis inhibitor and, as a nonsteroidal anti-inflammatory drug, works by blocking the action of prostaglandin.

14-40 Which statement is true about the use of alpha blockers in the treatment of symptomatic benign prostatic hypertrophy? A. They are safe and effective and should be given in the morning before breakfast. B. They do not lower blood pressure in normotensive clients. C. Pedal edema is the most common adverse effect. D. Blood counts should be monitored periodically for reduction in the platelet count.

Answer B Alpha blockers are an effective treatment of symptomatic benign prostatic hypertrophy. They reduce symptoms in 60% to 70% of clients, with nearly 50% improvement in urinary flow rates. They do not lower the blood pressure in normotensive clients. Dosing must begin with the lowest dose, preferably at bedtime, so the client will sleep through any mild adverse effects such as malaise, fatigue, dizziness, or orthostatic hypotension. Pedal edema is a rare adverse effect. Blood counts should be monitored occasionally for reduction in white or red blood cell counts.

5-105 Your 49-year-old female client reports that she has not had a menstrual period in 3 months. She uses contraception. She is experiencing occasional poor sleep, sweating, and flushing. She believes she is entering menopause. Which step is appropriate to take next? A. Initiate calcium supplementation. B. Obtain pregnancy test results. C. Send blood samples for follicle-stimulating hormone (FSH) and estradiol levels. D. Refer her for a bone-density test.

Answer B Although amenorrhea and hot flashes are signs associated with the onset of menopause, a period of amenorrhea for the prior 12 months is necessary for the diagnosis. Women in the perimenopause phase may experience intermittent ovulation and irregular or widely spaced menstrual periods. It is still possible for this individual to become pregnant, and pregnancy should be ruled out. Beyond that, laboratory tests are not needed for women whose menstrual periods have ended and who have associated signs such as hot flashes. Calcium supplements and bone-density screening are recommended after menopause has occurred.

6-81 Primary prevention measures for sexually transmitted diseases and unwanted pregnancies should be based on an understanding of which of the following psychosocial determinants? A. Informing adolescents of disease risk is an essential component of primary prevention. B. There must be multiple approaches, and these should begin in middle childhood. C. Working for personality change encourages adolescents to give up destructive behaviors. D. Handing out latex condoms and showing people how to use them is the best defense against sexually transmitted diseases and unwanted pregnancies.

Answer B Although many sources cite the condom as the most essential form of primary prevention against sexually transmitted diseases and unwanted pregnancies, in reality the best prevention tool is the brain. For this reason, an adequate understanding of psychosocial and cultural determinants underlying sexual behaviors is essential to individualize counseling approaches depending on the client. Teaching should begin before puberty. Middle childhood is a "rationale period" for children. Sex education has vocal opposition in many subcultures. However, preventive teaching and participatory discussion need not focus on sex specifically. Teaching can be centered on making positive future choices and impulse control. This teaching should address girls in particular because of the lifelong consequences of unintended pregnancy and sexually transmitted diseases in women. Although sharing disease-specific information may help, it is only one prong of what should be a multipronged approach to prevention. Likewise, your job as a health counselor is neither to change your client's personality or worldview nor to solve their deep conflicts; rather, you should offer better and healthier ways for clients to get what they want.

13-44 In an assessment of renal function, what is the maximum amount of urine in a 24-hour period that a client would produce for a diagnosis of anuria to be considered? A. No urine output at all B. Less than 100 mL C. 100 to 150 mL D. 151 to 200 mL

Answer B Anuria refers to a 24-hour urine volume of less than 100 mL. It indicates a severe reduction in urine volume commonly associated with obstruction, renal cortical necrosis, or severe acute tubular necrosis. It is important to make the distinction between oliguria (less than 500 mL in a 24-hour period) and anuria so that the appropriate treatment may be initiated early.

13-11 Urine that appears brownish in color may result from which of the following? A. Beets B. Bile C. Rifampin (Rifadin) D. Phenazopyridine (Pyridium)

Answer B Bile produces urine that is brownish in color. Beets, rifampin (Rifadin), and phenazopyridine (Pyridium) can cause reddish or reddish-orange urine.

19-22 Jackie is 20 years old, sexually active, and presents to urgent care with lower abdominal pain. During a pelvic exam, you notice cervical motion tenderness. You diagnose her with pelvic inflammatory disease (PID). Which pathogen is the most common offending agent? A. Neisseria gonorrhoeae B. Chlamydia trachomatis C. Gardnerella vaginalis D. Bacterial vaginosis-associated bacteria

Answer B Chlamydia trachomatis remains the most common pathogen for PID. N. gonorrhoeae used to be the second-most common pathogen, but the rate of gonorrhea has declined. Emerging infection agents include Gardnerella vaginalis and vaginosis-associated bacteria.

13-41 Martha, age 42, states that she has intercourse only about once a month, but immediately afterward she usually gets a urinary tract infection. She is frustrated with having to come to the office frequently to have an examination with a urine test. She says that she is ready to "kick her husband out of bed." How do you respond? A. "By taking a low-dose antibiotic every day, you can prevent a urinary tract infection." B. "Take 200 mg of ofloxacin with a large glass of water after intercourse." C. "I'll write you a prescription for a 3-day pack of antibiotics with several refills to take when you need it." D. "Yes, not having sexual intercourse will resolve the problem."

Answer B Clients who recognize an association between a urinary tract infection and recent sexual intercourse can be instructed to take a single small dose of an antibiotic, such as ofloxacin (Floxin) with a large glass of water after intercourse. Urinating before and/or after sexual intercourse may also help drastically reduce the chance of acquiring a UTI. Certainly, not having sexual intercourse will resolve the problem; this is not realistic for a 42-year-old woman.

115 Sam just left your office with a diagnosis of prostate cancer after an extensive work-up. His wife is on the phone demanding to know what his problem turned out to be. How do you respond? A. "Don't worry. Everything will be okay after we send Sam to a specialist." B. "You'll have to ask Sam; I can't tell you." C. "He has prostate cancer, but you'll have to talk to Sam about what his choices are." D. You refuse to take the call.

Answer B Confidentiality involves respecting privileged information. Health-care providers are responsible for making sure health-care information is shared only with those designated by the patient. Sam needs to be the one who tells his wife about his diagnosis.

13-9 The most common cause of nephrotic syndrome is A. systemic lupus erythematosus. B. diabetes mellitus. C. routine use of NSAIDs. D. glomerulosclerosis.

Answer B Diabetes mellitus is the most common cause of nephrotic syndrome. Systemic lupus erythematosus, the routine use of NSAIDs, glomerulosclerosis, and diabetes mellitus are all causes of proteinuria.

14-83 Emotional support is best given to the client with a sexually transmitted infection by A. offering many alternatives. B. authentic active listening. C. assuring the client that everything will be okay. D. emphasizing the duration of the disease.

Answer B Emotional support is best given to the client with a sexually transmitted infection (STI) by authentic active listening. During times of increased psychological stress, minimizing choices is better than offering too many choices. The client with an STI needs support from others, and emphasis should focus on the prevention of recurrences rather than the specifics of the duration of the disease.

14-69 Erectile dysfunction (ED), which affects 18 to 30 million men in the United States, increases with age. In men older than the age of 50, what are the most commonly found contributors to ED? A. Endocrine diseases B. Vascular disorders C. Neurogenic diseases D. Psychiatric conditions

Answer B Vascular diseases account for nearly half of all cases of erectile dysfunction. These include atherosclerosis, peripheral vascular disease, myocardial infarction, and arterial hypertension. Less frequent, but nonetheless important, factors include systemic diseases, such as diabetes, scleroderma, renal failure, and liver cirrhosis, and neurogenic diseases, such as epilepsy, stroke (CVA, brain attack), multiple sclerosis, and Alzheimer's disease. Other contributing factors include psychiatric conditions and penile, endocrine, nutritional, hematological, and medication-associated causes.

5-80 Jon, age 13, has some enlargement of the scrotum and testes, a reddened scrotal sac, and some hair texture alteration, but his penis is not enlarged. He is in Tanner stage A. I. B. II. C. III. D. IV.

Answer B Enlargement of the scrotum and testes, a reddened scrotal sac, some hair texture alteration, and a penis that is not enlarged are all characteristics of Tanner stage II. For boys, the sexual maturity rating scale developed by Tanner consists of five stages based on characteristics of pubic hair and genitalia. In stage I, the penis, testes, and scrotum are preadolescent in development. In stage III, there is further growth of the testes and scrotum, and the penis enlarges and becomes longer. In stage IV, there is an increase in the size of the penis with a growth in breadth and development of the glans, further enlargement of the testes and scrotum, and increased darkening of the scrotal skin. The pubic hair resembles that of an adult in type, but with the distribution considerably smaller than in the adult. In stage V, the genitalia are adult in size and shape, and the pubic hair is adult in quantity.

14-32 Erectile dysfunction is a complex phenomenon with a variety of causes. The predominant cause is A. psychological. B. vascular. C. neurogenic. D. drug related.

Answer B Erectile dysfunction has an organic origin in approximately 70% of cases. Of those cases, approximately 80% are related to vascular problems. The most common problem is generalized atherosclerosis that interferes with normal arterial function. Other vascular etiologies include hypertension, peripheral vascular disease, arterial insufficiency, trauma, or congenital abnormalities. Neurogenic disorders of the somatic, parasympathetic (cholinergic), sympathetic, and central nervous systems can cause or contribute to erectile dysfunction. Other diseases associated with erectile dysfunction include Parkinson's disease, cerebrovascular accident (stroke and brain attack), Alzheimer's disease, and diseases that create perfusion neuropathies such as diabetes and alcoholism. Erectile dysfunction is drug related in 25% of cases, with the common offenders being antihypertensive agents, NSAIDs, digoxin, antidepressants, sedatives, and antiandrogens.

19-28 Jared is a 27-year-old bisexual male. His girlfriend wants him to receive the HPV vaccine. What do you tell him? A. "You are beyond the age of the CDC recommendation to receive this vaccination." B. "Great! You should receive three doses within 6 months." C. "Great! You should receive two doses, with the second injection given 6 to 12 months after the first one." D. "As you are at the upper age limit, you only need one dose."

Answer B For gay or bisexual males, the CDC recommends receiving three injections of the vaccine within 6 months, up to age 26, ideally before the first sexual contact. Later vaccination can still offer protection if the patient has not been exposed to HPV.

14-81 Your client, J.P., age 24, is in to establish care as a new patient. You ask about his gender identity and sexual practices because this information will A. indicate the client's natal sex. B. inform risk assessment. C. predict reproductive choice. D. help determine the number of sex partners.

Answer B Gender identity and sexual orientation information are important data to obtain to help determine risk factors for specific conditions. Reproductive choice, natal sex, and number of sex partners cannot be determined without asking those questions specifically.

6-62 Phil, 14 years old, reluctantly tells you that he thinks he has something seriously wrong with him because he has awakened in the morning with a wet sheet around his penis. What do you do? A. Perform a test to rule out a sexually transmitted disease. B. Tell him that this is a normal part of his sexual development. C. Ask him if he is emptying his bladder before he goes to bed. D. Tell him that this is abnormal and you want to refer him to a urologist.

Answer B If an adolescent boy tells you he thinks he has something seriously wrong with him because he has awakened in the morning with a wet sheet around his penis, tell him that this a normal progression in his sexual development. In this case, Phil may be embarrassed to ask his friends about this and may not realize that "wet dreams" are a normal progression in sexual development. Ejaculation occurs in the early morning hours during an erection while sleeping and is referred to as a nocturnal emission. There is no need to perform a test to rule out sexually transmitted diseases; there is nothing in this history as described that indicates this is a possibility. You do not need to ask him about voiding before sleep as he does not have enuresis. This is not abnormal for an adolescent boy, so no referral is needed.

14-17 Which patient will most likely never develop prostate cancer? A. Jacob, age 79, who had a transurethral resection of the prostate for benign prostatic hypertrophy B. Jeffrey, age 11, who recently had an orchiectomy after a traumatic accident C. Sid, age 70, who has a normal prostate-specific antigen level D. Johnny, age 32, who is taking steroids for bodybuilding

Answer B In the early stages of prostate cancer, the tumor is androgen dependent. Testosterone is the major androgen, and clients who have undergone an orchiectomy before puberty never develop adenocarcinoma of the prostate.

14-36 A 23-year-old sex worker comes in for a renewal of her birth control pill prescription. She says that everything is fine. On examination, you find grayish-white vaginal discharge, greenish cervical discharge, and cervical motion tenderness. Which of the following differential diagnoses is most unlikely? A. Gonorrhea B. Interstitial cystitis C. Bacterial vaginosis D. Chlamydia

Answer B Interstitial cystitis is a chronic disease with none of the symptoms given in the stem of the question. A client who presents with grayish-white vaginal discharge, greenish cervical discharge, and cervical motion tenderness may have gonorrhea, bacterial vaginosis, or Chlamydia infection. Gonorrhea may be asymptomatic, or the client may present with yellowish urethral or vaginal discharge. The discharge of bacterial vaginosis is typically gray-white, malodorous or fishy smelling, and pruritic. Chlamydia infection may present with or without a vaginal or urethral discharge.

14-4 Which technique uses a learned method to target muscle contraction and relaxation to assist with urinary continence? A. Biofeedback B. Kegel exercises C. Bladder training D. Prompted voiding

Answer B Kegel exercises are a learned technique of pelvic muscle exercises that help with urinary incontinence after 4 to 5 weeks of consistent daily exercise. When used with biofeedback, they can improve pelvic floor tone and reduce uninhibited bladder contractions. Biofeedback consists of capturing information about a normally unconscious physiological process and subsequently using it in an educational process to accomplish specific therapeutic results, in this case, continence. Bladder training is a form of behavioral modification that helps to restore a normal pattern of voiding and normal bladder function. Clients void at fixed intervals whether the urge to void is present or not. Prompted voiding is also a form of behavioral modification that uses a toileting schedule, verbal feedback, and reinforcement.

14-65 Jackie, age 29, presents with no menstrual bleeding for 4 months. Her usual menstrual cycle is every 21 days and generally lasts for 7 days. She has never missed a period before, and her pregnancy test in the office is negative. This disturbance in her cycle is termed as A. irregular menstrual bleeding (IrregMB). B. absent menstrual bleeding (amenorrhea). C. false pregnancy (pseudocyesis). D. early menopause.

Answer B No menses in a 90-day period is termed absent menstrual bleeding or amenorrhea. Irregular menstrual bleeding (IrregMB) is bleeding of more than 20 days in individual cycle lengths over a period of 1 year. Heavy menstrual bleeding (HMB) is excessive menstrual blood loss that interferes with a woman's physical, emotional, social, and material quality of life and can occur alone or in combination with other symptoms. Light menstrual bleeding is menstrual flow that is not as heavy as the woman's normal menstrual flow. If a client taking an oral contraceptive (OC) complains of acne, the estrogen in the OC should be increased or the progestin should be decreased. In addition, you should also discuss hygiene, diet, and topical antibiotic drug therapy.

14-16 Stein-Leventhal syndrome, one of the leading causes of female infertility, is more commonly known as A. pelvic inflammatory disease. B. polycystic ovary disease. C. endometriosis D. ectopic pregnancy syndrome.

Answer B One of the leading causes of female infertility, Stein-Leventhal syndrome, is polycystic ovary syndrome (PCOS). It is a condition that afflicts many women during their childbearing years. Symptoms of PCOS, which are related to androgen excess and not associated with estrogen deficiency, include amenorrhea, hirsutism, acne, and obesity.

14-3 Tina, your 37-year-old white female patient, states she has been in a strictly monogamous relationship for over 12 months. You understand that risk reduction counseling for STDs A. should be done at this and every visit. B. may not be necessary. C. needs to focus on pregnancy prevention. D. is unlikely to change behavior.

Answer B Persons in monogamous relationships have less exposure (risk) for STD's. However, in other situations, you may need to explore the subjects of abstinence, monogamy, condom use, the patient's perception of his or her own risk and his or her partner's risk, and the issue of testing for STDs.

18 Marjorie has a Bartholin's cyst. What is the most common offending pathogen? A. Gonococcus B. Staphylococcus aureus C. Streptococcus faecalis D. Escherichia coli

Answer B Research has not shown benefits of screening for prostate cancer until after age 45 if there are no risk factors present. Cancer.org recommends that starting at age 45, African American men and men with close family members who had prostate cancer before age 65 should discuss testing with a provider. They should then decide if they want to be tested with a PSA with or without a DRE.

9 Mr. Hughes is a 46-year-old African American man who is in your primary care office for a physical examination. You recommend which of the following as screening for prostate cancer? A. A digital rectal examination (DRE) B. None; there is no conclusive evidence for screening until age 45 and older C. A DRE and prostate-specific antigen (PSA) level test D. A PSA test

Answer B Research has not shown benefits of screening for prostate cancer until after age 45 if there are no risk factors present. Cancer.org recommends that starting at age 45, African American men and men with close family members who had prostate cancer before age 65 should discuss testing with a provider. They should then decide if they want to be tested with a PSA with or without a DRE.

14-49 Sildenafil (Viagra) 50 mg PO taken 1 hour before sexual activity is ordered for Mitchell for his erectile dysfunction. What medication must you make sure he is not taking before writing the prescription? A. An antihistamine B. A nitrate C. A stool softener D. An anticoagulant

Answer B Sildenafil (Viagra) has shown promise in clients with erectile dysfunction. It is not effective in men with psychogenic impotence or those with neurological or arterial disease. It potentiates the hypotensive effect of nitrate and is contraindicated for clients receiving nitrates, such as nitroglycerin.

14-87 Today is Friday and a patient calls your office worried that she missed her combined oral contraceptive pill on Thursday. You advise her that she should A. throw away the package, use another method and restart the pill after her menses. B. take the Thursday pill now, the Friday pill tonight, no need for backup contraception. C. begin using backup contraception and double up on pills for 2 days. D. throw away the Thursday pills and take the Friday pill tonight.

Answer B Taking the missed pill within 24 hours is recommended. Missing one pill will not induce ovulation, therefore backup methods are not necessary.

14-24 Which of the following scrotal disorders is most common in adolescents? A. Acute epididymitis B. Testicular torsion C. Atrophic testes D. Scrotal edema

Answer B Testicular torsion or torsion of the testicle on its spermatic cord is the most common scrotal disorder in adolescents. It produces an acutely painful, tender, and swollen scrotum. Because of the potential of circulation being constricted, it is a surgical emergency. Acute epididymitis occurs chiefly in adults and is an acutely inflamed epididymis that is tender and swollen. Atrophic testes are small, soft testes associated with several conditions, such as cirrhosis, myotonia dystrophica, administration of estrogens, and hypopituitarism. Scrotal edema is usually associated with generalized edema in adults and is usually related to cardiac or nephrotic conditions.

14-28 The action of a 5-alpha-reductase inhibitor in the treatment of benign prostatic hypertrophy is to A. relax the smooth muscle of the prostatic capsule. B. reduce action of androgens in the prostate. C. relieve bladder obstruction. D. improve urinary flow rates.

Answer B The 5-alpha-reductase inhibitors are prescribed for their ability to induce apoptosis and atrophy, as well as reduce the action of androgens in the prostate. Alpha-adrenergic blockers relax smooth muscle of the bladder and prostatic capsule, improve flow rates, and relieve obstruction.

14-21 The Mobiluncus species is responsible for which sexually transmitted infection? A. Condylomata acuminata B. Bacterial vaginosis C. Human papillomavirus D. Lymphogranuloma venereum

Answer B The Mobiluncus species causes bacterial vaginosis. The human papillomavirus is responsible for condylomata acuminata (genital warts). Lymphogranuloma venereum is a sexually transmitted infection characterized by localized lymphatic infection with a Chlamydia origin.

14-35 Which of the following drugs may diminish the effectiveness of oral contraceptives? A. Beta blockers B. Oral anticoagulants C. Antibiotics D. Oral hypoglycemic

Answer C Antibiotics, antacids, anticonvulsants, and barbiturates may diminish the effectiveness of oral contraceptives. Clients should be urged to use other methods of birth control when taking antibiotics, specifically rifampin and griseofulvin.

5-62 Jane, age 45, is having her annual physical. She wants to know when she can expect to go through menopause. Your response to her would be A. "You should not have any more periods after the age of 50." B. "Most women begin having irregular periods in their late 40 s, but menopause can occur any time from age 40 to 60." C. "It is typical to complete the process of menopause at your age. I'm surprised you haven't gone through it yet." D. "You should think about hormone replacement therapy now."

Answer B The age of menopause can vary widely. There is usually a 2-year period of irregular periods beginning in the late 40 s, but it occurs earlier for some and may be as late as age 60 for others. After a year goes by without a menstrual period, menopause is considered complete. Jane may or may not have periods after the age of 50, so option A is providing false information. Age 45 is early to have completed menopause, and option C makes the client seem abnormal. This is not a therapeutic response. Based on information in the question stem, there is no need to begin hormone replacement therapy.

14-37 Which of the following drugs may have their effects diminished when used in combination with an oral contraceptive? A. Corticosteroids B. Oral anticoagulants C. Antibiotics D. Anticonvulsants

Answer B The effects of oral anticoagulants, acetaminophen, some benzodiazepines, oral hypoglycemic agents, and methyldopa may be diminished when used in combination with oral contraceptives.

13-21 Why is the right kidney slightly lower than the left kidney? A. The spleen pushes the left kidney upward. B. The liver pushes the right kidney downward. C. The diaphragm displaces the left kidney. D. The right kidney is structurally larger than the left kidney.

Answer B The liver on the right side displaces the right kidney slightly downward.

13-48 The most common cause of sepsis in older adults is A. urinary stasis. B. a urinary tract infection. C. a kidney stone. D. pneumonia.

Answer B The most common cause of sepsis in the older adult is a urinary tract infection (UTI). An older adult who is ill with hypothermia or a high fever, has a change in mental status and a documented UTI, or has a suspected UTI and sepsis should be treated vigorously with adequate hydration, immediate use of potent antibiotics, and support of blood pressure. Urinary stasis may precipitate a UTI, and kidney stones may accompany one, along with genitourinary problems, but a UTI alone is the most common cause of sepsis in the older adult. Urinary tract infections are the origin of roughly 10% to 30% of severe sepsis or septic shock cases, and urosepsis mortality can range from 25% to 60%. If pneumonia worsens without treatment, it can lead to sepsis, but UTIs are the most common cause.

5-18 When does the onset of menses usually occur? A. Between ages 10 and 11 B. Between ages 12 and 13 C. Between ages 14 and 15 D. Between ages 16 and 17

Answer B The onset of menses (menarche) usually occurs at age 12 or 13; however, it may begin before or after that age.

14-64 When a woman has extreme spasticity, which position should she assume for a Pap smear? A. OB "stirrups" position B. Knee-chest position while prone C. V-shaped position without stirrups D. Side-lying position

Answer B The woman who has extreme spasticity should assume a knee-chest position while prone for a Pap smear. Assistance may be required to keep the legs away from the perineal area. The OB stirrups position allows a woman who has difficulty using foot stirrups to assume the standard pelvic examination position. The client may need assistance in putting her legs into the stirrups up to the knees, which can be padded for comfort. The V-shaped position may be used without stirrups or with one foot in a stirrup. One or two assistants may be required to help the woman maintain this position by supporting each straightened leg at the knee and ankle. The client's comfort may be increased by elevating her legs slightly or by using a pillow under the small of her back or coccyx. The speculum must be inserted with the handle up. The side-lying position does not require stirrups and is most appropriate for the client who feels most comfortable and balanced lying on her side. This position is also good for the extremely obese woman. An assistant may help elevate one leg if the client cannot spread her legs. The speculum can be inserted with the handle pointing either toward the woman's back or front, but the clinician should be sure to angle the speculum toward the small of the client's back and not straight up toward her head.

74 Marcus, age 60, has just been given a diagnosis of renal cancer. He asks what the risk factors are. You tell him that they include age older than 40 as well as A. tobacco and alcohol use. B. tobacco use and analgesic abuse. C. exposure to aniline dyes and a low-fiber diet. D. a diet high in fat and nitrates.

Answer B Tobacco use; analgesic abuse; carcinogen exposure; obesity; exposure to asbestos, cadmium, and/or gasoline; use of phenacetin or aspirin-containing analgesics; chronic hemodialysis; and acquired polycystic kidney disease are all risk factors for renal cancer.

14-29 Which of the following is a sexually transmitted infection? A. Candida vaginitis B. Trichomonal vaginitis C. Atrophic vaginitis D. Lactobacilli vaginitis

Answer B Trichomonal vaginitis is a sexually transmitted infection. Monilial vaginitis, atrophic vaginitis, and bacterial vaginosis (BV) are all nonsexually transmitted types of vaginitis. Vulvovaginal candidiasis (formerly Monilia species), although not an STI, may be transmitted between partners and between a mother and newborn. Atrophic vaginitis is present in postmenopausal women who are not on hormone replacement therapy. BV is the most common vaginitis in women of reproductive age, of which almost 50% are asymptomatic. It results in changes in the vaginal bacterial flora with a loss of lactobacilli, an increase in vaginal pH (pH 4.5), and an increase in multiple anaerobic and aerobic bacteria. BV may be caused by multiple bacteria and some cases may be transmitted sexually.

13-55 Which is the least expensive method for evaluating renal mass size? A. Palpation B. Ultrasound imaging C. Computed tomography (CT) D. Magnetic resonance imaging (MRI)

Answer B Ultrasound imaging works as well as computed tomography (CT) and magnetic resonance imaging (MRI) for evaluating renal mass size. While the urologist may like to interpret cross-sectional images, particularly CT, comparable modalities for the client and reduced costs for the health-care system need to be explored, especially if they are just as effective. While palpation may possibly detect a large renal mass, the size of it cannot be evaluated with this method.

14-66 Why is Lactobacilli in the vagina? A. To decrease glycogen metabolism B. To maintain an acid pH range C. To increase the development of white blood cells D. To maintain an alkaline pH range

Answer B Vaginal flora of a normal reproductive-aged female includes multiple anaerobic and aerobic species (10 to 1). The typical vagina pH ranges between 4 and 4.5. Lactobacilli in the vagina help to maintain an acidic pH range to help protect against reproductive pathogens.

19-8 George, age 45, presents to urgent care complaining of a fever, chills, low back pain, being tired, and aching joints. He states that he has problems with his urinary system with frequency, urgency, pain on urination, and voiding at night many times. On exam, you note a warm, tense, boggy, and very tender prostate. What is your diagnosis? A. Benign prostatic hypertrophy (BPH) B. Acute bacterial prostatitis C. Chronic bacterial prostatitis D. Nonbacterial prostatitis and prostatodynia

Answer B With acute bacterial prostatitis, the general complaints include a fever, chills, low back pain, malaise, and arthralgia. The urinary complaints include frequency, urgency, dysuria, and nocturia. On physical examination (PE), the prostate feels warm, tense, boggy, and very tender. With chronic bacterial prostatitis, the general complaints include perineal pain, low back pain, lower abdominal pain, scrotal pain, and pain on ejaculation, although symptoms may be absent. The urinary complaints include dysuria and irritative voiding. On PE, the prostate feels normal or boggy. With nonbacterial prostatitis and prostatodynia, the general complaint would be pelvic pain. The urinary complaints include irritative voiding, and on PE, the prostate would feel similar to chronic bacterial prostatitis. With BPH, the patient would not have a fever, chills, low back pain, or aching joints. His prostate may feel boggy but would not be tender.

14-53 You've just finished a Pap smear on Sadie, age 39. During the wet mount, you see cells with bacteria adherent to the cell wall giving it a stippled, granular appearance. What do you suspect? A. Candidiasis B. Bacterial vaginosis C. Trichomoniasis D. Cervicitis

Answer B With the wet mount for bacterial vaginosis (BV), you will see clue cells—characteristic epithelial cells with bacteria adherent to the cell wall giving a stippled, granular appearance. BV is the most prevalent form of vaginitis among childbearing women. With candidiasis, the microscopic examination of the vaginal solution diluted with saline or 10% KOH (potassium hydroxide) preparations will demonstrate hyphal forms or budding yeast cells in 50% to 70% of infected women. With trichomoniasis, you will visualize motile flagellated trichomonads.

13-25 Jake is having a urinary diversion procedure performed because of a urinary tract tumor. He states that he heard about a Koch pouch and asks you what it is. How do you respond? A. "It is an opening on the skin in which one or both of the ureters create a stoma on the skin surface." B. "It is an opening on the skin in which the ileum is formed into a pouch with an open stoma and the ureters are inserted into the pouch." C. "It is an internal ileal reservoir where the ureters are connected and in which valves are formed on the skin. The filling pressure closes the valves, preventing leakage and reflux." D. "It is an opening on the skin in which the ureters are inserted into the sigmoid colon. Urine empties into the rectum and is expelled with defecation."

Answer C A Koch pouch is a continent internal ileal reservoir or continent ileal bladder conduit in which a pouch is created to be an ileal conduit. Nipple valves are formed on the skin by intussuscepting tissue backward into the reservoir to connect the pouch to the skin and the ureters to the pouch. The filling pressure closes valves, preventing leakage and reflux.

13-17 Which diagnostic finding(s) may lead to a diagnosis of nephrolithiasis? A. Urinary uric acid output of less than 750 mg per 24 hours B. Urinary pH greater than 5.5 C. Urinary calcium output of greater than 300 mg per 24 hours D. Serum calcium of 8 mg/dL

Answer C A urinary calcium output greater than 300 mg in 24 hours (hypercalciuria) may lead to a diagnosis of nephrolithiasis. The following disorders and diagnostic findings may also lead to a diagnosis of nephrolithiasis: hyperuricosuria (urinary uric acid output greater than 750 mg in 24 hours), hyperoxaluria (urinary oxalate output greater than 40 mg in 24 hours), hypocitraturia (urinary citrate output less than 320 mg/day), gouty diathesis (urinary pH less than 5.5 and gouty arthritis), and hypomagnesuria (urinary magnesium output less than 50 mg in 24 hours). Serum calcium of 8 mg/dL is within the normal range.

13-8 Which class of antihypertensive drugs is contraindicated in clients with renal artery stenosis? A. Calcium channel blockers B. Beta blockers C. ACE inhibitors D. Cardiac glycosides

Answer C ACE inhibitors are the antihypertensive drugs contraindicated in clients with renal artery stenosis. In bilateral renal artery stenosis or stenosis to a solitary kidney, the renal perfusion pressure and the glomerular filtration rate (GFR) depend on the local renin-angiotensin system. When the system is blocked by an ACE inhibitor, a marked decrease in the efferent arterial pressure with subsequent decrease in renal perfusion pressure results, causing a diminished GFR.

14-67 Harvey is complaining of stress urinary incontinence. To assess the autonomic arch innervating the bladder, the nurse practitioner should test the A. inguinal reflex. B. neuronal reflex. C. bulbocavernous reflex. D. meatal resistance.

Answer C After palpating the prostate gland, which is the first step in evaluating a male client complaining of stress urinary incontinence, you should evaluate the autonomic arch innervating the bladder by testing the bulbocavernous reflex. By squeezing the glans penis, you should note contraction of the anal sphincter in an individual without incontinence. An absent reflex suggests that there has been an interruption of the normal neuronal arch. If the individual is able to contract the rectal sphincter voluntarily, neuronal competence is also positive. Meatal resistance is difficult to evaluate, and examination of the bulbocavernous reflex as noted above is most accurate.

19-5 Julia, age 15, presents to urgent care and requests testing for an STI. What do you tell her? A. "I'll need permission from one of your parents first." B. "You'll have to bring in your sexual partner so that I can test him or her as well." C. "OK, let's do that now." D. "I can test you now, but then I must notify your parent of the results."

Answer C All states allow for minors to consent for the diagnosis and treatment of STIs. Some states place an age restriction on this and state that a minor must be at least 12 or 14 years old before allowing the testing. Some states allow clinicians to tell the parents that their child is seeking STI testing, but only Iowa requires notification (especially in the case of a positive HIV test).

14-63 An occurrence of genital herpes is A. cured with acyclovir (Zovirax). B. best managed with trichloroacetic acid 80% to 90% applied directly to the lesion. C. expected to be completely resolved within 21 days (for the primary lesion). D. not a factor in continuing with intercourse.

Answer C Although the primary lesion of genital herpes normally resolves within 21 days, the client usually has recurrent episodes. Acyclovir (Zovirax) is a palliative management option, but the drug does not cure herpes simplex. Topical trichloroacetic acid is the treatment for genital warts, not herpes. Intercourse should be avoided when a lesion is present.

149 Mr. Brown is 50 years old. His prostate-specific antigen (PSA) level last year was 2.8 ng/mL. His PSA this year is 4.3 ng/mL. Your first course of action would be to A. refer him for possible biopsy. B. wait for 2 weeks, then repeat the test. C. prescribe sulfamethoxazole and trimethoprim (Bactrim) PO bid or doxycycline (Vibramycin) 100 mg PO bid for 4 to 6 weeks and repeat the PSA 1 week after completion of the medication. D. refer him for transrectal ultrasound.

Answer C Asymptomatic bacterial prostatitis can cause an elevated PSA test. PSA levels may return to normal 4 to 6 weeks after a course of antibiotics. For a 50-year-old Caucasian, the normal level can be up to 4.0 ng/mL. The only concern is that it rose so much from last year, thus the suspected bacterial prostatitis. As the 4.3 level is just over the normal limit, a biopsy and a transrectal ultrasound would not be indicated at this time.

14-79 Balanitis may evolve into a chronic problem. If this occurs, the patient will experience severe purulence and phimosis. The treatment plan should include A. a stronger topical antifungal ointment. B. change from an antifungal ointment to a powder. C. oral antibiotics. D. an antiviral ointment.

Answer C Balanitis is a cutaneous inflammation of the glans penis, and Candida usually plays a role. Treatment usually involves a topical antifungal ointment as initial treatment if lesions show mild erythema or shallow erosions. Chronic balanitis suggests relapse, especially from a sexual partner. This suggests the partner needs to be treated. Severe purulent balanitis suggests a bacterial cause. If severe enough to cause phimosis, oral antibiotics are indicated.

14-42 Candidiasis is more common in A. teenage girls. B. women on low-fat diets. C. women with diabetes. D. women with frequent urinary tract infections.

Answer C Candida albicans infection is more common in women with diabetes, as well as those who are pregnant, immunosuppressed, or using antibiotics or oral contraceptives.

14-34 Which sexually transmitted diseases are cofactors for HIV transmission? A. Syphilis and chlamydia B. Herpes and chlamydia C. Chancroid and genital herpes D. Chancroid and gonorrhea

Answer C Chancroid, genital herpes, and syphilis are cofactors for HIV transmission. In the United States, about 10% of persons who have chancroid are coinfected with Treponema pallidum (syphilis) or HSV (herpes simplex virus). This percentage is higher in persons who have acquired chancroid outside the United States.

14-88 Manny has been taking finasteride (Proscar) and states that he has had dramatic relief. He previously took terazosin (Hytrin), which also helped, and he asks the nurse practitioner (NP) about taking that again. The NP should tell him A. "Yes, let's try the combination therapy because two are better than one." B. "No, they are absolutely contraindicated together." C. "There is no evidence to support combination therapy." D. "When symptoms get so bad that you need two different medications, it's time for surgery."

Answer C Combination therapy with an alpha blocker such as terazosin (Hytrin) or finasteride (Proscar) is not supported by the literature. One study showed no improvement when combination therapy was tried. Combination therapy involves extra expense and increased risk of adverse effects and has unproven effectiveness; therefore, it should not be used until successful trials have ensued.

94 About what percentage of children have congenital anomalies of the genitourinary tract, with severity ranging from abnormalities that remain asymptomatic through adulthood to malformations incompatible with life? A. Less than 3% B. 5% C. 10% D. More than 10%

Answer C Congenital anomalies of the genitourinary tract are more common than those of any other organ system. The rate of occurrence is about 10%.

143 What is the major cause of chronic renal failure? A. Hypertension B. Glomerulonephritis C. Diabetes mellitus D. Obstructive uropathy

Answer C Diabetes mellitus is the major cause of chronic renal failure. A large percentage of patients on dialysis are patients with diabetes. Other causes of chronic renal failure are primary hypertension, glomerulonephritis, cystic fibrosis, and other urological diseases.

13-30 Extracorporeal shock wave lithotripsy is not recommended for which type of kidney stones? A. Oxalate stones B. Uric acid stones C. Struvite stones D. Cystine stones

Answer C Extracorporeal shock wave lithotripsy is not recommended for struvite stones because bacteria or endotoxins inside the stone may be systemically dispersed during the procedure, and these stones do not break up easily. Appropriate antibiotics are required. This procedure may be used for oxalate, uric acid, and cystine stones.

14-1 First-line treatment for polycystic ovary syndrome (PCOS) is A. a bilateral oophorectomy. B. oral testosterone therapy. C. a combination of diet modification, weight loss, and stress management. D. a laparoscopy with a bilateral wedge resection.

Answer C First-line treatment for polycystic ovary syndrome (PCOS) is a combination of diet modification, weight loss, and stress management because obesity and stress alone can contribute to androgen excess. Oral estrogens are considered the first-line treatment for hyperandrogenism, with combination (estrogen and progesterone) oral contraceptives being the medication of choice. Treatment of uncomplicated amenorrhea in PCOS requires, at a minimum, monthly or bimonthly administration of medroxyprogesterone acetate (Depo-Provera). A laparotomy with a bilateral wedge resection is a treatment for anovulation. Because of the possibility of adhesions and ovarian atrophy, surgical interventions are used only in women who have tried and failed clomiphene citrate ovulation induction and when all other noninvasive options have been considered.

13-3 If a client with acute renal failure (ARF) excretes 400 mL of urine on Tuesday, how much fluid intake (both oral and intravenous) should the client have on Wednesday? A. 400 mL B. 600 mL C. 900 mL D. 1,200 mL

Answer C If a client with acute renal failure (ARF) excretes 400 mL of urine on Tuesday, the amount of fluid intake (both oral and intravenous) the client should have on Wednesday is 900 mL. A client with ARF is initially managed conservatively by fluid and dietary management. The permitted daily intake is calculated by allowing 500 mL for insensible losses (respiration, perspiration, and bowel losses) and adding the amount excreted as urine in the previous 24 hours. The client excreted 400 mL as urine, so adding 500 mL for insensible loss equals 900 mL total intake for the next 24 hours.

6-45 Ilene brings in her 14-year-old daughter, Tracie, because she suspects that Tracie may be sexually active. What do you do? A. Start Tracie on medroxyprogesterone acetate (Depo-Provera) immediately. B. Perform a vaginal exam on Tracie. C. Ascertain in private from Tracie if she is contemplating becoming or is sexually active. D. Call child protective services.

Answer C If a mother brings in her daughter because she thinks her daughter is sexually active, you should not take as fact the mother's belief. You should find out directly from the daughter. If the daughter admits to being sexually active, you may want to do a vaginal examination and discuss contraception methods or counseling. Notification of a child protective services agency may be warranted if the daughter admits to, or if you suspect, sexual abuse or an incestuous relationship with her father, stepfather, or other male relative given the age of the child and state law on statutory rape, even if sexual activity is consensual and nonfamilial.

13-16 About 80% of all cases of dysuria in women are caused by ascending bacterial infection of the urinary tract by which organism? A. Klebsiella B. Proteus C. Escherichia coli D. Staphylococcus saprophyticus

Answer C In women, approximately 80% to 90% of cases of uncomplicated UTI are the result of the gram-negative rod bacteria Escherichia coli. The second-most common cause (5% to 20%) of uncomplicated bacterial infection is the gram-positive coccus Staphylococcus saprophyticus, although this agent is rare in complicated UTI. Other gram-negative rods identified as causative pathogens in a smaller number of cases, but particularly in complicated UTI, are Proteus mirabilis, Klebsiella, Enterobacter, Serratia, and Pseudomonas. In addition, the gram-positive coccus Enterococcus has been identified as a culprit.

13-56 Most calcium phosphate kidney stones are caused by A. a high dietary calcium intake. B. a high phosphorus dietary intake. C. primary hyperparathyroidism. D. hyperthyroidism.

Answer C Most calcium phosphate stones are caused by primary hyperparathyroidism. Treatment involves surgical excision of the parathyroid adenoma. If the surgery is not successful or is impossible, treatment involves hydration and the administration of orthophosphate. The client is then observed for hypertension and sodium and water retention. Calcium oxalate stones are another type of calcium stone. Oxalate is a naturally occurring substance found in food. Some fruits and vegetables, as well as nuts and chocolate, have high oxalate levels. The liver also produces oxalate. Dietary factors, high doses of vitamin D, intestinal bypass surgery, and several metabolic disorders can increase the concentration of calcium or oxalate in urine. Struvite stones form in response to an infection, such as a urinary tract infection. These stones can grow quickly and become quite large, sometimes with few symptoms or little warning. Uric acid stones can form in people who do not drink enough fluids or who lose too much fluid, those who eat a high-protein diet, and those who have gout. Certain genetic factors also may increase the risk of uric acid stones. Cystine stones form in people with a hereditary disorder that causes the kidneys to excrete too much of certain amino acids (cystinuria).

5-71 Sally, age 12, asks you when you think her periods will start. What do you tell her? A. "They should have started by now." B. "They will probably start any day now." C. "There is a range of normal for girls from 11 to 16." D. "It might not happen until you're 15."

Answer C Most girls experience menarche at around the same age as their mothers did; however, the average age for American adolescents today is 12.5 years, with a standard deviation of 1.3 years. If a girl has not begun to have breast changes by age 13 or has not started her periods by age 16, she should be referred for evaluation.

5-119 At what stage of sexual development should a boy expect to begin to have nocturnal emissions? A. When he starts to get facial hair B. When his scrotal sac begins to darken in color C. Six to 12 months after his penis begins to enlarge D. At 15 years of age, regardless of his sexual development

Answer C Nocturnal emissions ("wet dreams") usually begin during sleep at about age 14. The first ejaculation of seminal fluid usually occurs about 1 year after the penis begins its adolescent growth.

14-26 Benign prostatic hypertrophy is a common finding as men age. Classically, this condition may begin with difficulty initiating the urinary stream, hesitancy, urgency, postvoid dribbling, urinary frequency, nocturia, urinary retention, sensation of a full bladder immediately after voiding, and incontinence. These preceding symptoms would also cause the nurse practitioner to consider what other condition as a differential diagnosis? A. Epididymitis B. Testicular cancer C. Cancer of the prostate D. Balanitis

Answer C Other differential diagnoses for symptoms classically seen in men with benign prostatic hypertrophy, such as difficulty initiating stream, hesitancy, urgency, postvoid dribbling, frequency, nocturia, retention, sensation of a full bladder immediately after voiding, and incontinence, include diabetes mellitus, cancer of the prostate, and some neurological diseases that can lead to voiding disorders. Testicular cancer, balanitis, and epididymitis do not typically cause this array of symptoms.

14-85 A 17-year-old female presents to your office with the complaint of lower abdominal pain since her period ended 2 days ago. She has a new sexual partner in the past 3 months and does not use condoms. On physical examination, you find that she has cervical motion tenderness. You are concerned that she may have pelvic inflammatory disease (PlD). According to the Centers for Disease Control and Prevention (CDC) criteria, which finding supports this diagnosis? A. An oral temperature greater than 101°F and mucopurulent cervicitis B. A positive test for cervical infection and an adnexal mass C. Lower abdominal tenderness and adnexal tenderness D. Mucopurulent cervicitis and a decreased white blood cell count on saline microscopy

Answer C Pelvic inflammatory disease (PID) comprises a spectrum of inflammatory disorders of the upper female genital tract. Sexually transmitted organisms, especially Neisseria gonorrhoeae and Chlamydia trachomatis, are implicated in many cases. Empiric treatment for PID should be initiated in sexually active young women and other women at risk for sexually transmitted infections if they are experiencing pelvic or lower abdominal pain, if no cause for the illness other than PID can be identified, and if one or more of the following minimum criteria are present on pelvic examination: cervical motion tenderness, uterine tenderness, adnexal tenderness.

25 Which glomerular disease occurs 10 to 14 days after an acute illness (commonly streptococcal in children) and is characterized by tea-colored urine, mild to severe renal insufficiency, and edema? A. Henoch-Schönlein purpura glomerulonephritis B. Glomerulonephritis of systemic lupus erythematosus C. Postinfection glomerulonephritis D. Immunoglobulin A nephropathy

Answer C Postinfection glomerulonephritis occurs when glomeruli become leaky and red blood cells leak into the urine, causing the tea color. The loss of protein from blood into the urine causes a fluid shift into the subcutaneous tissue, and edema results.

14-23 Which of the following signs and/or symptoms of a genital herpes infection usually occur(s) first? A. Painful or pruritic vesicles B. Dysuria C. Prodromal tingling or pruritus of the genital region D. White, curd-like plaques on a red base in the vagina

Answer C Signs and symptoms of a genital herpes infection include tender inguinal lymph nodes, as well as painful or pruritic vesicles, dysuria, prodromal tingling or pruritus of the genital region (which usually occurs first), and cervical ulcerations. White, curd-like plaques on a red base in the vagina are seen with monilial vaginitis.

14-2 Small-quantity incontinence with nearly continuous dribbling is symptomatic of which kind of incontinence? A. Stress incontinence B. Urge incontinence C. Overflow incontinence D. Functional incontinence

Answer C Small-quantity incontinence, which produces nearly continuous dribbling, is symptomatic of overflow incontinence. Stress incontinence is symptomized by small-quantity incontinence on coughing, sneezing, laughing, and running. Urge incontinence involves an uncontrolled urge to void and is symptomized by large-quantity incontinence. Functional incontinence involves voiding normally with assistance.

142 A variety of drugs have been implicated in erectile dysfunction. These include, but are not limited to, A. diuretics, beta blockers, and ACE inhibitors. B. vasodilators, anticholinergic agents, and antihistamines. C. cimetidine (Tagamet), beta blockers, and diuretics. D. ACE inhibitors, calcium channel blockers, estrogens, and digoxin.

Answer C Some antihistamines, such as cimetidine as well as beta blockers and diuretics, have been identified as drugs that may cause erectile dysfunction (ED). Drugs that cause drug-induced impotence in one man may not affect another man. Some antipsychotics may also cause ED.

6-94 Human papillomavirus (HPV) is detectable in 99% of cervical cancers. A vaccine has been developed to prevent HPV infection and subsequent cervical intraepithelial neoplasia. What is the ideal time/age to administer this vaccine? A. At age 3 months B. At age 2 years C. Before the initiation of sexual activity D. Shortly after the initiation of sexual activity

Answer C The HPV vaccine is recommended for preteen boys and girls at age 11 or 12 so that they are protected before ever being exposed to the virus. HPV vaccine also produces a more robust immune response during the preteen years. Finally, older teens are less likely to get heath checkups than preteens. The HPV vaccine is given in 3 shots. The second shot is given 1 or 2 months after the first shot. Then a third shot is given 6 months after the first shot. The CDC recommends receiving the full HPV vaccine series.

14-10 The average age of menopause in the United States is A. 45 years. B. 48 years. C. 51 years. D. 53 years.

Answer C The average age of menopause in the United States is 51 years. Menopause is defined by the World Health Organization as the permanent cessation of menstruation resulting from loss of ovarian follicular activity and 12 months of amenorrhea at the time of midlife.

13-27 What is the most common cause of chronic renal failure (CRF)? A. Glomerulonephritis B. Hypertension C. Diabetic nephropathy D. A combination of urological diseases

Answer C The most common cause of chronic renal failure (CRF) is diabetic nephropathy. Diabetic nephropathy makes up about 34% of all cases of CRF, followed by hypertension (29%), glomerulonephritis (11%), and other urological diseases (6%).

13-29 What is the most common cause of death in dialysis clients? A. Infection B. Bleeding from the access site C. Cardiovascular failure D. Sepsis

Answer C The most common cause of death in dialysis clients is cardiovascular failure, with hypotension and diabetes as predisposing factors. Sepsis is the next leading cause of death, followed by bleeding complications, cerebrovascular accidents (brain attacks and strokes), pericardial effusion with tamponade, and trauma. The mortality in dialysis clients remains significant, about 15% in the first year.

14-45 What is the most common cause of male infertility? A. Azoospermia B. A problem with sperm motility C. A varicocele D. Antisperm antibodies

Answer C The most common cause of male infertility is a varicocele. Other causes include oligospermia or azoospermia, problems with sperm function or motility, abnormalities of sperm morphology, and, rarely, an antisperm antibody.

151 In teaching your client about birth control, you tell her that the highest failure rate is with which birth control method? A. Spermicide B. Withdrawal C. Cervical cap D. Condoms

Answer C The spermicide failure rate is 2% to 30%, the failure rate for withdrawal is 18%, the failure rate for the cervical cap is 2% to 20%, and the condom failure rate is 3% to 12%.

14-8 The nurse practitioner has just treated a patient's condylomata acuminata with podophyllum in benzoin. What instructions should be given to him as part of patient teaching? A. "Refrain from sexual relations for 48 hours." B. "Don't take a shower until tomorrow morning." C. "Wash the medication off within 1 to 2 hours." D. "Go into the bathroom now and wash the medication off."

Answer C The treatment of choice for the client with condylomata acuminata (warts) on the external genitalia is to "paint" them with podophyllum in benzoin. The client should wash the medication off in 1 to 2 hours because normal tissue may be destroyed along with the warts. Sometimes a repeat treatment is necessary. Carbon dioxide laser treatment might be more effective.

13-50 Transient urinary incontinence in women may be caused by A. hypertrophy of the vaginal or urethral walls. B. diarrhea. C. use of anticholinergic agents. D. hypoglycemia.

Answer C The use of anticholinergic agents may be responsible for transient urinary incontinence in women. Etiologies for transient urinary incontinence may be stated using a DIAPERS mnemonic: D for drugs, such as hypnotics, sedatives, anticholinergic agents, diuretics, and adrenergic agents, and delirium or altered mental status; I for infection; A for atrophy of the vagina or urethra; P for psychological disorders, such as functional depression; E for endocrine disorders, such as hyperglycemia or hypercalcemia; R for restricted mobility; and S for stool impaction.

129 Marsha, age 60, is asking about whether she should have cervical cancer testing with a Pap test or not. How do you respond? A. "No, now that you're over 60, you no longer need Pap tests." B. "No, but you still need an HPV test every 5 years until the age of 65." C. "You still need a Pap and an HPV test until the age of 75." D. "You should have one more Pap and HPV test."

Answer D Cancer.org states that women ages 30 to 65 should get a Pap test and an HPV test every 5 years, or get a Pap test every 3 years. When over the age of 66, if the previous Paps done on a regular basis are normal, they may stop testing. If women have had a serious cervical precancer previously, they should be tested for at least 20 years after the diagnosis.

14-11 Which type of urinary incontinence results from Parkinson's disease and multiple sclerosis? A. Overflow incontinence B. Stress incontinence C. Urge incontinence D. Functional incontinence

Answer C There are four types of urinary incontinence that are considered established—overflow, stress, urge, and functional—and one type that is considered transient or potentially reversible. Overflow incontinence is caused by detrusor underactivity. There is frequent leakage of urine from the failure to fully empty the bladder, such as occurs with prostatic hypertrophy. Stress incontinence is a failure to store urine related to urethral incompetence. It may be caused by weak pelvic musculature or intrinsic or neurogenic sphincter deficiency and is commonly seen in men after radical prostatectomy. Urge incontinence caused by detrusor overactivity results in the failure to store urine and can coexist with urethral obstruction from benign prostatic hypertrophy or be due to conditions such as Parkinson's disease, multiple sclerosis, urinary tract infection, bladder stones, or tumors. Functional incontinence is caused by the effects of medications, fecal impaction, and immobility. Transient incontinence is characterized by a sudden onset. Its causes in older males can include delirium, infection, pharmacological agents, psychological factors, excess urinary output, restricted mobility, and stool impaction in hospitalized or immobile persons.

71 Specific conditions demonstrate fluid in the scrotum on transillumination. These include A. testicular cancer. B. a varicocele. C. a hydrocele. D. testicular torsion.

Answer C To assess for hydrocele, the scrotum is transilluminated with a penlight in a darkened room. The trapped fluid may appear light pink, yellow, or red. The other conditions listed do not have fluid, so they will not be able to be transilluminated.

49 Marge comes for a pelvic examination complaining of vaginal burning and itching and painful intercourse. You note strawberry spots on her cervix, along with a greenish-yellow, frothy, foul-smelling vaginal discharge. What is your diagnosis? A. Candida albicans infection B. Gardnerella vaginalis infection C. Trichomonas vaginalis infection D. Atrophic vaginitis

Answer C Trichomonas vaginalis is characterized by greenish-yellow, frothy vaginal discharge with a strong odor; painful urination; vaginal itching and irritation; and occasionally lower abdominal pain. Vaginal examination would reveal vulvovaginal erythema and a strawberry appearance of the cervix plus microscopic hemorrhages in the surface tissue of the cervix.

13-28 The most common cause of urinary tract obstruction is A. edema resulting from trauma. B. a tumor. C. ureterolithiasis. D. a vascular problem.

Answer C Ureterolithiasis—a stone in the ureter—is the most common cause of urinary tract obstruction. Pain with ureteral obstruction is compounded by the stretching of the renal capsule by edema and swelling. Edema does not usually cause complete obstruction. Although a tumor may cause an obstruction, it is not as common as an obstruction caused by ureterolithiasis. Vascular problems may aggravate or potentiate a problem leading to an obstruction but may not be the actual cause.

13-12 The most common factor predisposing a woman to a urinary tract infection is A. the use of an oral contraceptive. B. the use of a diaphragm. C. urinary stasis. D. calculi.

Answer C Urinary stasis is the most common factor predisposing a woman to a urinary tract infection (UTI). It is followed by calculi and the presence of catheters, stents, and other foreign bodies. The use of an oral contraceptive (if the male partner is not using a condom) also predisposes a woman to a UTI because sexual intercourse increases the likelihood of developing a UTI. The use of a diaphragm also increases the risk of a UTI because of the properties of the spermicidal agents used in conjunction with the diaphragm. Women should be encouraged to empty their bladders when they get the urge to go and not to "hold" it.

14-86 Ursula, age 19, is going to begin taking birth control pills. She asks you if she is "safe" immediately. How do you respond? A. "Yes, you should not get pregnant once you start taking the pill. However, it doesn't protect you from STIs." B. "For the first month, you need to be on a backup birth control method. However, the pill doesn't protect you from STIs." C. "A second birth control method needs to be used during intercourse for the first 7 days while taking the pill. However, the pill doesn't protect you from STIs." D. "Until you have your second period (cycle) with the pill, you are not considered safe."

Answer C When a client is first starting to take birth control pills, she should be instructed to use a second birth control method during intercourse, such as condoms or a diaphragm used with spermicide, for the first 7 days. Another contraceptive method should always be kept on hand to use in case of missed pills; when taking another medication that might interfere with pill effectiveness, such as an antibiotic; or when vomiting or diarrhea occurs.

14-38 Human papillomavirus (HPV) may lead to A. pelvic inflammatory disease. B. molluscum contagiosum. C. cervical dysplasia. D. genital herpes.

Answer C When human papillomavirus (Condylomata acuminata) causes genital warts, it may lead to cervical dysplasia and cervical cancer. Pelvic inflammatory disease is usually secondary to gonorrhea or Chlamydia infection. Molluscum contagiosum is a sexually transmitted disease that causes a benign viral skin infection. Genital herpes is caused by herpes simplex virus.

14-57 Beth is breastfeeding her 3-month-old infant with no supplementation. She says she has heard that she cannot get pregnant during this time. What do you tell her? A. "It's highly likely that you may become pregnant, so you should use another method of birth control." B. "Yes, you're safe for as long as you breastfeed." C. "For the first 6 months, if you breastfeed and have very little supplementation, your chances are less than 2% that you'll get pregnant." D. "You're more at risk for getting pregnant now because of your fluctuating hormone levels."

Answer C Women have less than a 2% chance of getting pregnant as long as they are amenorrheic for 2 months' postpartum; are fully breastfeeding, with a supplementation not exceeding 15%; and are less than 6 months' postpartum. If any of these conditions is not present, the woman needs to begin to use another form of contraception.

19-48 In performing a urinalysis on a patient, you note urine that is foamy. This indicates the presence of which of the following? A. Blood, hemoglobin, myoglobin, or having eaten beets or food coloring B. A urinary tract infection, blood, bilirubin, or mucus C. Diabetes insipidus, diuretic agents, or fluid overload D. Protein or bile salts

Answer D Foamy urine may indicate the presence of protein or bile salts. Pink/red urine may indicate the presence of blood, hemoglobin, myoglobin, or having eaten beets or something with red food coloring. Cloudy urine may indicate the presence of a urinary tract infection, blood, bilirubin, or mucus. Colorless urine may indicate diabetes insipidus, the use of diuretic agents, or fluid overload.

14-78 "Emergency contraception" refers to A. an induced abortion in an emergency department. B. quickly starting on birth control pills in anticipation of sexual intercourse. C. having a medroxyprogesterone (Depo-Provera) injection in the ER every 12 weeks. D. methods of contraception that can be used to prevent pregnancy in the first few days after unprotected intercourse.

Answer D "Emergency contraception" (or "emergency birth control") refers to keeping a woman from getting pregnant when she has had unprotected sex. Depending on when they are taken, the pills work by keeping the egg from leaving the ovary, by keeping the sperm from meeting the egg, or by keeping a fertilized egg from attaching to the uterus. The pills must be taken within 5 days of unprotected sex in order to work. There are two types of emergency contraception pills. Plan B is a progestin-only pill taken for the purpose of emergency contraception. One pill is taken immediately; the next is taken 12 hours later. In the second type, a higher dose of regular birth control pills is taken. The number of pills is different for each brand; again, one dose is taken immediately, and the second dose is taken 12 hours later. Plan B is sold over the counter without a prescription. The copper IUD also can be used for emergency contraception. The FDA has not labeled it to be used in this manner, but it is highly effective if placed within 5 days of sexual intercourse. IUDs containing levonorgestrel are currently being investigated for use as emergency contraception. The abortion pill—Mifeprex—is for use after a woman becomes pregnant (after a fertilized egg has already attached to the uterus). Mifeprex is used with misoprostol and can end an early pregnancy within 49 days of the start of a woman's last menstrual period. The regimen for a medical abortion through day 49 of a pregnancy is on day 1 to take Mifeprex three tabs of 200 mg together, and then on day 3 to take misoprostol two tabs of 200 mcg together. On day 14 the client should see her health-care provider to confirm that a complete termination of the pregnancy has occurred.

14-76 Janice, age 26, who has genital herpes, asks if her partner has to use a condom during sexual intercourse even if she does not have a visible lesion. How do you respond? A. "Yes, we're not sure if it's still transmitted when the lesions are not visible, so it's better to be on the safe side." B. "No, you're not contagious when the lesions are not visible." C. "No, use of a spermicidal agent is all that is required." D. "Yes, shedding of the herpes simplex virus is a primary mode of transmission even in the absence of a lesion."

Answer D A condom should be worn during sexual intercourse when one partner has genital herpes, even though there may not be a visible lesion. Shedding of the herpes simplex virus from mucocutaneous surfaces in the absence of visible lesions is a primary mode of transmission both horizontally (to sexual partners) and vertically (to the fetus).

14-77 The initial diagnostic and/or laboratory testing that a nurse practitioner should order to rule out organic causes of erectile dysfunction in men includes A. color Doppler sonography. B. CBC, blood chemistry profile, TSH, and PSA. C. nocturnal penile tumescence and rigidity test (NPTR). D. FBS, lipid profile, TSH, and testosterone.

Answer D A fasting blood sugar test to rule out diabetes mellitus, a lipid profile to rule out dyslipidemia, and a determination of thyroid-stimulating hormone and testosterone levels are the initial laboratory tests that should be done to rule out causes of erectile dysfunction (ED). If the testosterone level is below 300 ng/dL, then a serum prolactin level is warranted. In a male with established ED, a CBC, blood chemistry profile including fasting glucose or glycosylated hemoglobin level, TSH, and PSA are frequently recommended. Specialized tests, such as the color Doppler sonogram or NPRT, can be done if the cause of ED is not apparent from the above standard tests.

14-82 How long can the vaginal contraceptive ring (NuvaRing) be out of the vagina before an additional form of contraception is necessary? A. 30 minutes B. 1 hour C. 2 hours D. 3 hours

Answer D A vaginal contraceptive ring (NuvaRing) is a soft, transparent, flexible ring of ethylene vinyl acetate copolymer. If the ring is accidentally expelled from the vagina for less than 3 hours, instruct the woman to rinse it with lukewarm water and reinsert it into her vagina. No backup method is needed. If the ring is expelled from the vagina for 3 hours or more, instruct the woman to discard the ring and (1) insert a new ring immediately and use a backup method of birth control for an additional 7 days or (2) insert a ring no later than 7 days from when the last ring was removed and use backup methods for 7 days. Caution the patient that a withdrawal bleed may occur if the ring was not in place for several days.

14-61 The most likely cause of amenorrhea is A. an anatomical deviation. B. a genetic factor. C. an endocrine abnormality. D. pregnancy.

Answer D Although pregnancy seems like an obvious choice as the most likely cause of amenorrhea, it is sometimes overlooked, especially if the client denies the possibility of pregnancy and is seeking a pathological reason for the amenorrhea. The most likely physiological causes of amenorrhea that should be considered are pregnancy, lactation, and menopause, if appropriate. Among women who are in the childbearing years, amenorrhea unrelated to pregnancy may signal stress or a life-threatening disease. These conditions may include anatomical deviations, genetic factors, endocrine abnormalities or imbalances, defective enzyme systems, autoimmune diseases, tumors, eating disorders, excessive exercise, and medications. With amenorrhea, a pregnancy test should always be done first to rule out pregnancy or its related complications, such as ectopic pregnancies, complete or incomplete abortions, and trophoblastic neoplasms. The most accurate test for pregnancy is the serum beta human chorionic gonadotropin test.

13-5 Among the following genetic diseases, which is the most common cause of chronic renal disease? A. Cystic fibrosis B. Sickle cell disease C. Huntington's chorea D. Polycystic kidney disease

Answer D Autosomal dominant polycystic kidney disease (ADPKD) is the most common genetic cause of chronic renal disease. It is 20 times more common than Huntington's chorea, 15 times more common than cystic fibrosis, and 10 times more common than sickle cell disease as a cause of chronic renal disease.

14-80 The most common type of vaginal infection in women of childbearing age is A. candidiasis. B. trichomoniasis. C. gonorrhea. D. bacterial vaginosis.

Answer D Bacterial vaginosis (BV) is the most common vaginal infection (about 40% of all cases). The infecting organisms are identified as Gardnerella vaginalis, Mobiluncus species, and other anaerobes. Bacterial vaginosis results in an overgrowth condition within the vagina for as-yet-unknown reasons. The incubation period is 5 to 10 days. About half of all clients with BV are asymptomatic. Those with symptoms typically describe a gray-white, malodorous or fishy-smelling pruritic discharge that is accompanied by burning. It may be scant to profuse and adhere to the vaginal walls. The differential diagnoses include any other known cause for vaginitis (such as trichomoniasis or candidiasis) and cervicitis (such as gonorrhea or Chlamydia infection). Diagnosis is made through microscopic examination of the specimen by wet mount. The practitioner should look for clue cells; the presence of these cells, which look like pepper on the surface of cells, is diagnostic of BV.

148. what recommendation from the american cancer society for screening for prostate cancer in an african american male who is 50 years old? a. PSA screening now if desired b. PSA screening should have taken place earlier c. DRE only d. no screening recommendation at this time

a. PSA screening now if desired

14-46 Characteristics of polycystic ovary syndrome (PCOS) include A. hirsutism, thinness, and hypoinsulinemia. B. menopausal onset, vitiligo, and hyperinsulinemia. C. alopecia, thinness, and abdominal cramping. D. premenarchal onset, obesity, and hyperinsulinemia.

Answer D Characteristics of polycystic ovary syndrome (PCOS) include premenarchal onset, obesity, hyperinsulinemia, hyperandrogenism (hirsutism, seborrhea, acne, and alopecia), menstrual disturbances, and infertility. Visible signs of the syndrome are obesity, acne, and hirsutism. Clients with PCOS typically present with complaints of amenorrhea or irregular menstrual cycles, but some have the initial complaint of infertility. About 10% to 20% of clients with PCOS are symptomatic. PCOS should be considered in all clients presenting with amenorrhea, infertility, or hirsutism.

13-51 Which of the following is the likeliest outcome in clients with chronic kidney disease (CKD)? A. End-stage renal disease (ESRD) B. Dialysis C. Hypertension D. Death

Answer D Clients with chronic kidney disease (CKD) are more likely to die from the disease than to start dialysis or progress to end-stage renal disease (ESRD), as most people would think is the likeliest outcome. It is now evident that CKD is a major risk factor for death from cardiovascular disease (CVD) and that this risk increases with worsening CKD. Clinicians need to focus their attention on preventing CKD progression, maximizing CVD risk reduction, and aggressively treating comorbidities associated with CKD. These comorbid conditions include anemia, volume overload, abnormal calcium phosphate homeostasis, hypertension, dyslipidemia, and metabolic syndrome, many of which predispose to CVD.

20-26 Your client brings her 18-year-old daughter in for an initial visit. She is sexually active and seeking a Pap smear with HPV testing. You explain to them that clinical guidelines recommend the initial Pap smear be done A. soon after sexual activity begins. B. within 1 year after menarche. C. at age 18. D. at age 21.

Answer D Clinical guidelines (ACOG and USPSTF) recommend the initial Pap be performed at age 21. According to AAFP, most observed abnormalities in adolescents regress spontaneously, therefore screening Pap smears done in this age group can lead to unnecessary anxiety, additional testing, and cost.

13-26 Janice, age 54, presents with clinical manifestations including microscopic hematuria, a palpable abdominal mass, fever, and flank pain. What do you suspect? A. Pancreatic tumor B. Liver tumor C. Colon mass D. Renal tumor

Answer D Clinical manifestations of renal tumors include microscopic or gross hematuria, a palpable abdominal mass, fever, flank pain, fatigue, weight loss, and anemia or polycythemia. The key terms here are flank pain and hematuria This is specific to renal complaints. Pancreatic tumor symptoms would show up only when the disease progresses. They would also include weight loss as well as jaundice, loss of appetite, pain in the upper abdomen and back, and digestive problems, including abnormal stools, nausea, or vomiting. A liver tumor may include many of those symptoms as well, such as weight loss, nausea and vomiting, abdominal pain, as well as a large mass that can be felt in the upper right part of the abdomen. With a colon mass, besides palpating an abdominal mass, there probably would be a change in bowel habits, including diarrhea or constipation, as well as a change in the consistency of the stool, with possibly narrow stools. There will probably be blood in the stool.

14-70 Urinary stone disease, or urolithiasis, afflicts a large number of clients every year, exceeded in frequency as a urinary tract disorder only by infections and prostate disease. This disorder affects men more frequently than women, with a ratio of 3 to 1. Common presenting signs and symptoms include which of the following? A. Guarding of the abdomen B. Fever C. Pain that is present during the daytime hours D. Nausea and vomiting

Answer D Colic is the main presenting symptom of urinary stone disease and usually occurs suddenly, even awakening clients from sleep. The pain is sudden and severe and often accompanied by nausea and vomiting. These clients are constantly moving in contrast to those with acute abdominal pain who "guard" their abdomen and try not to move, each movement eliciting increased pain. Although the pain may occur episodically, it is not a chronic pain.

14-68 Gerri, age 33, complains of external vaginal irritation after adding new fabric softener to her laundry. You have diagnosed her with reactive vaginitis. The treatment of choice for this condition includes A. metronidazole (Flagyl) 500 mg PO twice a day for 7 days. B. conjugated vaginal estrogen cream externally every day for 1 week. C. rewashing undergarments without fabric softener and applying petroleum jelly to the affected area. D. rewashing undergarments without fabric softener and applying corticosteroids to the affected area.

Answer D Elimination of the offensive agent and use of corticosteroids is the treatment of choice for reactive vaginitis. An antifungal agent needs to be administered for candidiasis, and topical estrogen is used for atrophic vaginitis. No treatment or povidone-iodine is used for normal cervical or vaginal-discharge vaginitis.

14-50 When performing a newborn assessment of a male infant, the nurse practitioner notes that the urethral opening is on the dorsal side of the glans. This is referred to as A. hypospadias. B. Peyronie's disease. C. priapism. D. epispadias.

Answer D Epispadias means that the urethral meatus opens on the dorsal side of the glans. Hypospadias means that the urethral meatus opens on the ventral side of the glans. Both must be reported to the physician because a circumcision should not be performed until these conditions are corrected. Peyronie's disease is a condition of penile curvature that occurs with erection. Priapism is a continuous and pathological erection of the penis.

13-57 Which of the following drugs is(are) not associated with acute renal failure? A. Angiotensin-converting enzyme (ACE) inhibitors B. Cimetidine (Tagamet) C. NSAIDs D. Erythromycin (E-Mycin)

Answer D Erythromycin (E-Mycin) is not associated with acute renal failure (ARF). It is one of the safest antibiotics, even in pregnancy. NSAIDs and ACE inhibitors can cause the prerenal type of ARF by causing a decreased intrarenal arteriolar resistance. Cimetidine (Tagamet) can cause an elevation in serum creatinine levels without a change in glomerular filtration. Although this is not a problem in clients with normal renal function, there is a more profound effect on serum creatinine concentrations in clients with borderline renal dysfunction.

13-33 Which of the following conditions does not cause flank pain? A. Pyelonephritis B. Ureterolithiasis C. Vascular occlusion of the kidney (renal vein thrombosis) D. Renal cysts

Answer D Renal cysts are usually asymptomatic and usually do not cause flank pain. Pyelonephritis, ureterolithiasis, and vascular occlusion of the kidney (renal vein thrombosis) all usually cause flank pain.

14-31 Which of the following conditions is a contraindication to using the copper intrauterine device (IUD)? A. History of ectopic pregnancy B. Nulliparity C. Treated cervical dysplasia D. Heart disease

Answer D Heart disease is considered a contraindication to the use of a copper intrauterine device (IUD) because the client may be susceptible to bacterial endocarditis. The following conditions were previously believed to preclude the use of IUDs, including the copper IUD, but they are no longer contraindications: history of ectopic pregnancy (remains a contraindication to use with the progesterone-containing IUDs); nulliparity; treated cervical dysplasia; diabetes mellitus; valvular heart disease; irregular menses as a result of anovulation, breastfeeding; corticosteroid use; and age under 25. The conditions that preclude systemic hormonal methods (breast cancer, venous thromboembolism or phlebitis, arterial vascular disease, active liver disease, and age older than 35 combined with smoking) do not preclude intrauterine contraception.

119 When doing a pelvic examination on Tara, age 19, you note painful, raised, reddened lesions filled with fluid around the labia. You diagnose these as A. lesions associated with syphilis. B. a vaginal infection. C. condyloma. D. herpetic vesicles.

Answer D Herpetic vesicles are fluid-filled blisters caused by herpes. Condyloma is a wart-like growth around the anus or vulva. Syphilitic lesions usually begin as solitary, raised, firm, red papules. A vaginal infection will probably involve a discharge.

67 Timmy, age 5, comes for a preschool physical. You note that his urethral meatus opens on the ventral surface of the penis and document this as A. epispadias. B. paraphimosis. C. phimosis. D. hypospadias.

Answer D Hypospadias is a birth defect of the urethra in the male where the urinary opening is not in the usual location on the head of the penis. Epispadias is where the urethra opens on the upper aspect of the penis. Paraphimosis is the inability to return the foreskin back to its normal position over the tip of the penis. Phimosis is where the foreskin of the penis cannot be pulled back past the glans.

14-51 Infertility is best defined as the A. inability to conceive when having intercourse during ovulation. B. inability to conceive for 9 months of unprotected intercourse when both partners are younger than 30 years of age. C. choice to be childless. D. inability to conceive after 1 full year of unprotected intercourse.

Answer D Infertility affects approximately 15% of reproductive-age couples (ages 15-44) in the United States. Although pregnancies do occur outside this age range, they are less frequent. Infertility is defined as the failure of a couple to conceive after 12 months of frequent, unprotected intercourse.

14-73 Barry, a 32-year-old patient, has a history of unprotected intercourse with numerous sexual partners. He has been diagnosed with epididymitis. Initial treatment for this inflammatory condition includes A. use of a scrotal support and warm compresses. B. epididymectomy. C. a spermatic cord block with local anesthetics. D. bedrest with scrotal elevation and antibiotics.

Answer D Initial treatment of epididymitis includes bedrest with scrotal elevation and icepacks, along with antibiotics. A spermatic cord block with local anesthetics may be necessary to relieve pain in severe cases. Drainage of an abscess or referral for surgical intervention may also be needed depending on severity of the case.

43 Menstruating women lose approximately how much iron per day? A. 5 mg B. 10 mg C. 0.1 to 1.6 mg D. 2 mg

Answer D Menstruating women lose about 2 mg of iron per day.

13-23 Eric, age 5, has enuresis. The probable cause is A. anatomical disease. B. neurological disease. C. a psychological problem. D. maturational delay.

Answer D Most children with voiding disturbances do not have an anatomical or neurological disease or a psychological problem causing their enuresis. Although the cause of primary nocturnal enuresis has not been clearly established, it appears to be related to maturational delay of mechanisms involved in sleep and arousal or to a delay in the development of increased bladder capacity. Most children can be helped through parental involvement, such as restricting fluids after dinner, encouraging bladder training, awakening the child during the night to void, and using electronic devices that establish a conditioned reflex response to waken the child the moment urination starts. (Such devices are only mildly successful.) Imipramine (Tofranil) is effective (50 mg PO at bedtime), as well as desmopressin acetate (DDAVP) nasal spray (one spray in each nostril at bedtime).

13-53 Which of the following medications is least likely to damage your kidneys? A. Vancomycin B. Ramipril C. Jardiance D. Acetaminophen

Answer D NSAIDS, vancomycin, iodinated radiocontrast dyes, ACE inhibitors (e.g., ramipril), Jardiance (a diabetic medication), and certain HIV and antiretroviral medications can all lead to kidney injury. Acetaminophen is safe regarding kidney insult.

13-4 Marcy, age 24, is having extreme dysuria with her urinary tract infection (UTI). Which of the following drugs would you prescribe as an analgesic to treat her dysuria? A. Hyoscyamine (Levsin) B. Oxybutynin (Ditropan) C. Propantheline (Pro-Banthine) D. Phenazopyridine (Pyridium)

Answer D Phenazopyridine (Pyridium) is the only analgesic listed. For the first few days of a urinary tract infection (UTI), all of the drugs listed may be prescribed, in addition to antibiotics, to decrease the pain and discomfort of a UTI. Use of these agents should not be prolonged, however, given their significant side-effect profile. Analgesics such as phenazopyridine (Pyridium) may be prescribed, but this alters the color of urine to orange and may cause urinary leakage secondary to anesthetization of the urethra and sphincter. Anticholinergics, including atropine (Donnatal), hyoscyamine (Levsin, Cystospaz), propantheline (Pro-Banthine), or oxybutynin (Ditropan), produce an antispasmodic effect, thus relieving pain. However, anticholinergics may also contribute to urinary retention (especially in the elderly), which is a clear risk factor for UTI and should thus be used with caution.

44 Jack had a routine screening and was found to have proteinuria. Your next action would be to A. schedule a renal ultrasound. B. refer him to a urologist. C. ask the physician to schedule a renal biopsy. D. collect a 24-hour urine sample for quantification.

Answer D Proteinuria is usually an indicator of renal pathology. Any persistent proteinuria that is not classified as functional proteinuria requires further work-up, beginning with a 24-hour measurement of urine protein and creatinine level. It is too early to decide on a renal ultrasound or a biopsy. Depending on the results of the 24-hour urine test, a referral to a urologist may be indicated.

169. which of the following drugs is recommended for postherpeetic neuralgia? a. TCAs b. SSRIs c. atypical antidepressants d. benzodiazepines

a. TCAs TCAs and anticonvulsants are recommended. help with neuropathic pain.

14-75 In a patient diagnosed with cervical gonococcal infection, you would also suspect a coinfection with A. candidiasis. B. syphilis. C. trichomoniasis. D. chlamydia.

Answer D Simultaneous chlamydial infections are present in 30% to 50% of clients who have cervical gonococcal infections. Treatment should automatically be done for both when one has been diagnosed. The most common therapies are azithromycin (Zithromax) 1 g PO for one dose for chlamydia infection and ceftriaxone (Rocephin) 125 mg IM for one dose for gonorrhea. Candidiasis is a fungal infection that may affect the mouth, throat, vagina, or bloodstream. Syphilis is a sexually transmitted infection caused by a type of bacteria known as Treponema pallidum. Trichomoniasis is a sexually transmitted infection caused by a parasite.

6-5 Jane, age 72, is very upset about her stress incontinence. She asks you for advice. What do you tell her? A. "Unfortunately, stress incontinence is part of the normal aging process." B. "There are many new pads on the market that provide an efficacious approach to this common problem, thus preventing social isolation." C. "A diet that incorporates cranberry juice may be effective in curbing this problem." D. "Kegel exercises may help."

Answer D Stress urinary incontinence is not a normal part of the aging process. It may be controlled or alleviated in many women. The diagnosis must be confirmed first to rule out other causes of incontinence. Nonsurgical treatment options include drug therapy with estrogen vaginal cream, pelvic floor electrical stimulation, imipramine, Kegel exercises with biofeedback, pessaries, and occlusive devices. Very often, a combination of several of these will benefit the client. Surgery may be indicated, depending on which structure needs repairing. For example, if a cystocele is producing the symptoms, repair of the anterior vaginal wall prolapse is indicated. Pads may prove helpful, but the underlying causes must first be investigated. There is no evidence that cranberry juice helps curb stress incontinence.

14-9 What differentiates prostate cancer symptoms from benign prostatic hypertrophy (BPH) symptoms? A. Urinary frequency, hesitancy, and intermittency are much worse with prostate cancer. B. Nocturia is worse with BPH. C. Dribbling and a weak stream are more indicative of BPH. D. Symptoms of prostate cancer in general tend to progress more rapidly than those of BPH.

Answer D Symptoms of prostate cancer can mimic the symptoms of benign prostatic hypertrophy (BPH); however, with prostate cancer in general, the symptoms tend to progress more rapidly as compared with those of BPH. Symptoms of both prostate cancer and BPH include urinary frequency, hesitancy, intermittency, nocturia, dribbling, and a weak urinary stream.

147 James is a high school senior and cannot understand why you will not prescribe testosterone for him. He states that all his bodybuilding friends take anabolic androgenic steroids to increase their muscle mass. You tell him that you have seen it ordered A. for weight loss. B. for decreased immune function. C. for fibrocystic breast disease. D. to increase energy and appetite in older adults.

Answer D Testosterone increases muscle mass, bone density, strength, and bone maturation. It is also used to increase energy and appetite in older adults. Teenagers should not be taking steroids to increase their muscle mass

6-31 As the nurse practitioner in an outpatient clinic, you will be performing a history and physical on Maria, age 15, whose mother comes to the clinic with her. How would you approach the topic of sexuality with Maria? A. Ask Maria's mother if you can discuss this issue with Maria. B. Ask Maria, with her mother present, if it is okay for you to discuss this issue with them both. C. Ask Maria's mother to leave the room. D. When you are alone with Maria, tell her that you want to talk to her about this issue and ask her if she wants her mother to be present.

Answer D The U.S. Preventive Services Task Force and other groups advise health-care providers to take a complete sexual history on all adolescent and adult clients. Because of the sensitive nature of the topic, the provider needs to emphasize its importance to the client and state the relevance to a total history and physical. The client who is present with another in the room needs to be asked in private if he or she would be more comfortable addressing this topic alone or with the other person present.

13-59 The best index of kidney function is A. a midstream urinalysis. B. serum creatinine levels. C. urine protein level. D. glomerular filtration rate (GFR).

Answer D The best index of kidney function is the glomerular filtration rate (GFR). Kidney function used to be assessed by serum creatinine level. Both diet and muscle mass influence generation of creatinine, however, making it an inaccurate indicator of renal function. Lower serum creatinine levels are typically observed with older age, female gender, vegetarian diet, and muscle-wasting states, while higher values are associated with muscular habitus and a high-protein diet. Serum creatinine levels can underestimate kidney disease. The National Kidney Foundation defines chronic kidney disease (CKD) as the presence of kidney damage and/or reduced GFR for 3 or more months. The most precise method to measure the GFR is to measure iothalamate or insulin clearance. This is the gold standard for investigational studies, but it is not practical in clinical practice. The 24-hour urine creatinine clearance test is an alternative in clinical practice, and that measurement is required for the calculation of the GFR. Clients often have difficulty conducting this 24-hour test. Recently, the Modification of Diet in Renal Disease (MDRD) equation has been recommended by the National Kidney Foundation to better estimate the GFR. Besides serum creatinine, the MDRD GFR requires input of client age, gender, and ethnicity.

13-43 What amount of urine output in a 24-hour period represents oliguria in an adult? A. No urine output B. Urine output less than 50 mL C. Urine output less than 100 mL D. Urine output less than 500 mL

Answer D While sources differ and are sometimes arbitrary, oliguria (diminished urination) in an adult is typically defined as less than 500 mL urine output per day. Anuria, although it technically means "without urine," is an output of less than 100 mL per day. The body makes 1 mL of urine per minute, or 1,440 mL per day. In the hospital setting, the least acceptable amount of urine is 30 mL per hour, but adults actually produce 60 mL per hour.

14-56 Endometrial cancer, hirsutism, acne, breast cancer, increased risk of diabetes, infertility, menstrual bleeding problems, and an increased risk of cardiovascular disease are clinical consequences of A. mastalgia. B. abnormal uterine bleeding. C. endometriosis. D. persistent anovulation.

Answer D The clinical consequences of persistent anovulation include infertility; menstrual bleeding problems, ranging from amenorrhea to dysfunctional uterine bleeding; an increased risk of cardiovascular disease; hirsutism and acne; an increased risk of endometrial cancer and breast cancer; and an increased risk of diabetes mellitus in clients with hyperinsulinemia. Therapy depends on the client. If the client wants to get pregnant, she is a candidate for the medical induction of ovulation. For the client who does not wish to become pregnant and does not complain of hirsutism but is anovulatory and has irregular bleeding, therapy is directed toward interruption of the steady-state effect on the endometrium and breast. Mastalgia is breast pain. The breast is a complex organ that is sensitive to hormones. Estradiol and progesterone stimulate breast tissue. Breast pain that positively correlates with menses is cyclic mastalgia. Cyclic mastalgia is the most common breast-related complaint seen in women's health practice. Abnormal uterine bleeding (acute) (AUB) is an episode of bleeding in a woman of reproductive age, who is not pregnant, of sufficient quantity to require immediate intervention to prevent further blood loss but at the normal time during the month. AUB (chronic) is bleeding from the uterus that is abnormal in duration, volume, and/or frequency and has been present for the majority of the last 6 months. Endometriosis is extrauterine growth of the endometrial glands or stroma. It is believed to occur through retrograde menstruation, differentiation of totipotential cells, or both.

19-24 Macy, age 81, had the shingles vaccine (Zostavax [zoster vaccine live]) 10 years ago and heard that there is a new vaccine that will protect her better. She has very mild renal impairment. Which of the following is true about Shingrix (recombinant zoster vaccine)? A. Since she already had the Zostavax vaccine, she does not need Shingrix. B. With Macy's renal impairment, she is not a candidate. C. Since she had the previous vaccine, she will need only one dose. D. Shingrix is administered as a two-dose series at 0 and 2 to 6 months.

Answer D The recombinant zoster vaccine, Shingrix, is administered IM as a two-dose series at 0 and 2 to 6 months for adults over the age of 50. Shingrix may be administered after someone has received the Zostavax vaccine. There are no dosage adjustments related to renal or hepatic impairment.

14-14 Abnormalities of the scrotum are usually painless or nontender. Which of the following is an exception and is usually tender? A. Hydrocele B. Tumor of the testis C. Spermatocele D. Tuberculous epididymitis

Answer D Tuberculous epididymitis is a chronic inflammation of tuberculosis. It produces a firm enlargement of the epididymis, which is usually tender, and thickening or beading of the vas deferens. A hydrocele is a nontender, fluid-filled mass that is in the space within the tunica vaginalis. A spermatocele is a painless, movable cystic mass just above the testis. A tumor of the testis is usually a painless nodule.

5-88 The mother of your 12-year-old female client is concerned because all her daughter's friends have started their periods, but her daughter has not. Upon physical exam, you note that the daughter's pubic hair and breast development are both at Tanner stage III. Your next step should be to A. refer to an endocrinologist. B. start the girl on birth control pills to encourage her menses to start. C. order bone age x-rays. D. explain to the mother and child that this appears to be normal for this child.

Answer D Typically, menarche occurs 2 years after the onset of thelarche, at the end of Tanner stage III. The average age for girls to reach menarche is 12.5 years. There is no need for this 12-year-old to be referred, to have bone x-rays, or to be started on birth control pills to encourage her menses to start.

13-31 Joe, age 72, is coming home after his transurethral resection of the prostate (TURP). He has heard about frequent urinary infections caused by indwelling catheters. You tell him that urine tends to become colonized when indwelling urinary catheters are left in place for more than A. 24 hours. B. 36 hours. C. 48 hours. D. 72 hours.

Answer D Urine tends to become colonized with bacteria when indwelling urinary catheters are left in place for more than 72 hours. If catheters are left in place only temporarily and removed quickly when the client can void, infection will usually not result. Antibacterial coverage is warranted if catheters are left in place for 5 to 10 days.

13-46 You should be concerned about the use of diuretics, chronic renal failure, a high-protein diet, gout, leukemia, or lymphoma when a serum uric acid level is A. 4.0 mg/dL. B. 5.3 mg/dL. C. 6.0 mg/dL. D. greater than 7.2 mg/dL.

Answer D When a client has an elevated serum uric acid level (greater than 7.2 mg/dL), the following should be considered as possible causes: the use of diuretics, chronic renal failure (CRF), a high-protein diet, gout, leukemia, alcoholism, and lymphoma. To specifically look for CRF, a serum creatinine level test and a 24-hour urinary uric acid test need to be performed.

6-74 Your client, whom you recently treated for a sexually transmitted disease, has returned with a reinfection. At this point, you A. re-educate. B. stress the importance of "safe sex" in more emphatic terms. C. instruct the client to avoid all sexual contacts. D. expect that the client will need to come back for more than one visit.

Answer D When counseling clients, practice the "art of the possible." Your goal is not to increase the client's knowledge but to effect behavioral change. Be realistic. For example, a client should not be asked to stop all intercourse. Clients who are not adequately helped the first time will be back again and again. It is practically impossible to counsel clients in this area adequately in one session. Personal connection, repeated messages, praise of progress, and negotiation are the effective strategies to use. Even commercial sex workers, for example, might learn to be more restrictive by insisting on practicing only safe sex or by avoiding high-risk situations such as working with belligerent or drugged clients.

14-71 What is the position of the uterus when the cervix is on the anterior vaginal wall? A. Midposition B. Retroverted C. Retroflexed D. Anteverted

Answer D When the uterus is anteverted, the position of the cervix is on the anterior vaginal wall. When the position of the uterus is midposition, the cervix is at the apex of the vagina. When it is retroverted, the cervix is on the posterior vaginal wall. When the uterus is retroflexed, the position of the cervix may be on the anterior or posterior vaginal wall or the apex.

28. irregular bleeding associated with depo-provera can be minimized with the use of all of the following except: a. acetaminophen b. ibuprofen c. naproxen sodium d. estrogen supplements

a. acetaminophen

14-48 According to the American Urological Association (AUA) Guideline on the Management of Benign Prostatic Hyperplasia: Diagnosis and Treatment Recommendations, when is referral for invasive surgery automatically warranted? A. With an AUA symptom index of 7 or lower B. With an AUA symptom index of 8 or greater C. With irritative symptoms such as urgency, frequency, or nocturia D. With presence of refractory retention and bladder stones

Answer D With mild symptoms of benign prostatic hyperplasia (BPH) and an AUA index of 7 or lower, watchful waiting is recommended. If a client has moderate to severe symptoms and an AUA index of 8 or higher, noninvasive medical therapy is still an option, as is minimally invasive therapies or surgery. The presence of refractory retention or any of the following clearly related to BPH require referral to surgery: persistent gross hematuria, bladder stones, recurrent urinary tract infections, and/or renal insufficiency. Irritative symptoms are usually what the client will present with for further assessment to diagnosis BPH or other conditions.

9. which of the following results in a clinically INSIGNIFICANT increase in the PSA? a. DRE b. ejaculation c. prostatitis d. prostate biopsy

a. DRE leads to .26-.4 for about 48-72 hrs afterward. prostate biopsy increases PSA about 8 for up to 4wks following. prostate infection and ejaculation can significantly increase PSA levels in varying amounts

17. chancroid is considered a cofactor for transmission of: a. HIV b. gonorrhea c. chlamydia d. trichomoniasis

a. HIV

5. 644 The nurse practitioner is assessing a 13-year-old male who presents with back pain and nausea and vomiting for the past 24 hours. The patient reports feeling a dull pain and muscle spasms in the back after a direct hit to the side during football practice. His vital signs are blood pressure of 140/84 mmHg, temperature of 99.1°F, pulse of 98 beats/minute, and respirations of 26 breaths/minute. Which diagnosis is most likely? A. Appendicitis B. Bruised kidney C. Cholecystitis D. Ruptured vertebral disc

B) Bruised kidney. The findings (back pain and muscle spasms, nausea with vomiting, and a direct hit to the side of the body) indicate a potential bruised kidney. Appendicitis typically begins with anorexia, nausea, and vomiting for the first 12 to 24 hours. Abdominal pain, a late sign, is usually diffuse at first and gradually localizes to the right lower quadrant. Cholecystitis may cause radiating pain to the back but would not be due to a direct hit and would not manifest as muscle spasms. A ruptured vertebral disc would manifest as pain and muscle spasm but without nausea and vomiting.

126. The mother of a 16-year-old boy is concerned that her son is not developing normally. On physical exam, the patient is noted to have small testes with no pubic or facial hair. What is the most appropriate statement to the mother? A. Her son is developing normally B. Her son's physical development is delayed and should be evaluated by a pediatric endocrinologist C. Her son should be rechecked in 3 months; if he still does not have secondary sexual characteristics, a thorough hormonal workup should be initiated D. Her son's physiologic development is slower than normal but is within the lower limit of normal for his age group

B) Her son's physical development is delayed and should be evaluated by a pediatric endocrinologist. Puberty may be delayed for several years and still occur normally, in which case it is considered constitutional delay, a variation of healthy physical development. Delay of puberty may also occur due to malnutrition, many forms of systemic disease, or defects of the reproductive system (hypogonadism) or the body's responsiveness to sex hormones. Hypogonadism occurs when the sex glands produce little or no hormones. In men, these glands (gonads) are the testes. A 16-year-old male without secondary sexual characteristics should be referred to an endocrinologist. If there is no testicular development by 14 years of age, an endocrinology consult is warranted.

93. A 17-year-old female patient tells the nurse practitioner that she is sexually active with her long-term boyfriend and asks for contraception information and a prescription. What is the responsibility of the nurse practitioner? A. Call the parent and obtain consent B. Prescribe the contraceptive after educating the patient about the medication C. Counsel the patient on the dangers of early sexual activity D. Ask the patient to include the boyfriend in the contraceptive decision

B) Prescribe the contraceptive after educating the patient about the medication. A 17-year-old patient has the right to consent for the use of contraceptives. She may give full consent without the involvement of her parents or boyfriend. The nurse practitioner should provide all necessary education regarding safe sexual practices and choice of contraception. The patient is due all confidentiality in this matter.

102. 751 High-grade squamous interepithelial lesions (HSILs) were found in a Pap specimen for a 26-year-old patient. The nurse practitioner will: A. Test the specimen with a potassium hydroxide slide B. Refer the patient for loop electrosurgical excision procedure C. Refer the patient for colposcopy with cervical biopsy D. Administer vaccination for human papillomavirus (HPV)

B) Refer the patient for loop electrosurgical excision procedure. HSILs are likely to be associated with precancer and cancer. If they are found in patients aged 25 years or older, the patient should be referred for immediate excisional treatment by LEEP or cervical conization surgery. A potassium hydroxide slide is useful in the diagnosis of fungal infections. Colposcopy is appropriate for HSIL found in younger women (aged 21-24). Quadrivalent recombinant vaccination can prevent HPV, which is associated with cervical cancer, but once the patient has HSIL, administration of the vaccination would not be useful.

160. What effect does finasteride (Proscar) have on prostate-specific antigen (PSA) levels in patients with prostate cancer? A. Finasteride does not have an acute effect on the PSA B. The chronic effect of finasteride is the stabilization of the PSA C. When using finasteride for long-term management, the PSA will be doubled D. An increase in PSA within the normal range may occur with the use of finasteride

B) The chronic effect of finasteride is the stabilization of the PSA. The chronic effect of using finasteride to treat prostate cancer is the stabilization or gradual decline of the patient's PSA. An acute effect of the finasteride on PSA is a reduction, rather than an increase, of approximately 50%. Any increase in PSA is a concern, even if the value is within a normal range.

128. Which of the following is not an absolute contraindication for use of oral contraceptive pills? A. Active hepatitis A infection B. Thrombosis related to an intravenous (IV) needle C. Undiagnosed vaginal bleeding D. Transient ischemic attack (TIA)

B) Thrombosis related to an intravenous (IV) needle. Thrombosis related to either a known trauma or an IV needle does not represent a contraindication for use of oral contraceptives.

65. The preferred method to identify the location of small renal stones is A. x-ray B. abdominal ultrasound C. CT scan D. radionuclide scan

C. CT scan

40. You see a 70-year-old woman in a walk-in center with a chief complaint of increased urinary frequency and dysuria. Urinalysis reveals pyuria and positive nitrites. She mentions she has a "bit of kidney trouble, not too bad." Recent evaluation of renal status is unavailable. In considering antimicrobial therapy for this patient, you prescribe: A. nitrofurantoin. B. fosfomycin. C. ciprofloxacin. D. doxycycline.

C. ciprofloxacin.

58. You see an 82 year old woman with early onset dementia and urge incontinence. Which of the following medications is least likely to contribute to worsening mental status? A. oxybutynin (Ditropan) B. tolterodine (Detrol) C. derifenacin (Enablex) D. solifenacin (VESIcare)

C. derifenacin (Enablex)

44. In a person diagnosed with superficial bladder cancer without evidence of metastases, you realize that: A. the prognosis for 2-year survival is poor. B. a cystectomy is indicated. C. despite successful initial therapy, local recurrence is common. D. systemic chemotherapy is the treatment of choice.

C. despite successful initial therapy, local recurrence is common.

8. You see a 63-year-old man with a suspected upper gastrointestinal bleed who has no renal health issues. Expected laboratory findings would include: A. elevated BUN; elevated serum creatinine. B. normal BUN; elevated serum creatinine. C. elevated BUN; normal serum creatinine. D. lowered BUN; elevated serum creatinine.

C. elevated BUN; normal serum creatinine.

14. Guidelines recommend considering initiating treatment with an erythropoiesis-stimulating agent (ESA) for patients with chronic renal failure and a hemoglobin (Hg) level: A. less than 8.5 mg/dL. B. less than 9.0 mg/dL. C. less than 10 mg/dL. D. less than 11.5 mg/dL.

C. less than 10 mg/dL.

the Shingrix vaccine: who gets it? what is the dosing schedule?

CDC recommends 2 doses of Shingrix separated by 2-6 months for immunocompetent adults aged 50+yrs -does not matter if they report a prior episode of HZ -does not matter if they had prior dose of Zostavax (no longer available in US) -does not matter if this have chronic medical conditions -can be used in those taking low dose immunosuppressive therapy, are anticipating immunosuppression, have recovered from immunocompromising illness, are getting other routine vaccines at visit such as influenza and PNA vax, just use different arms -there is no specification for the amount of waiting time before administering shingrix after having HZ, however do not give if pt is experiencing acute episode, it is not necessary to screen for evidence of prior varicella infection

Complications of COVID-19

Complications of COVID-19 include pneumonia, acute respiratory distress syndrome, cardiac injury, arrhythmia, septic shock, liver dysfunction, acute kidney injury, and multi-organ failure, among others. Long COVID

How is cutaenous anthrax commonly manifested?

Cutaneous anthrax develops 2-5 days (range, 1-7 days) postexposure. Lesions most commonly develop at lacerations, abrasions, or insect bites on exposed areas of skin; most commonly affects upper extremities but may arise anywhere on the body. Infection begins as a pruritic papule that enlarges within 24-48 hours to form a 1-cm vesicle; this then becomes an ulcer surrounded by an edematous halo. [1] Resultant lesions are usually 2-3 cm in diameter with a round, regular, raised edge. Lesions may become edematous and necrotic but are usually not purulent. They are painless but on occasion are slightly pruritic. Regional lymphadenopathy may occur and may be painful. The ulcer and edema evolve into a black eschar within 7-10 days and then last for 7-14 days before separating and leaving a scar. Lymphadenopathy may be persistent. With neck lesions, edema and lymphadenopathy may impinge on the airway and cause stridor and respiratory compromise.

23. The results of a Pap test performed on a 21-year-old woman state, "Atypical squamous epithelial cells present. No endocervical cells present." The nurse practitioner will: A. Test for human papillomavirus (HPV) B. Perform an endometrial biopsy C. Refer for colposcopy D. Repeat the test as soon as possible

D) Repeat the test as soon as possible. A specimen is satisfactory only if both squamous epithelial cells and endocervical cells are present. If lacking either type of cell, the specimen is incomplete, and the test needs to be repeated. Atypical squamous cells are cause for concern in a complete specimen. When atypical squamous cells of undetermined significance are found in women age 25 to 29, a HPV test should be performed. An endometrial biopsy is indicated if there are atypical glandular cells in the presence of endometrial cells. A referral for colposcopy is needed only if atypical cells are found in an acceptable specimen in women age 30 years or older. A colposcopy would be recommended for a 21-year-old woman only if high-grade squamous intraepithelial lesions were found.

How does inhalational anthrax manifest?

Inhalational anthrax begins abruptly 1-3 days (range, hours to 60 days) postexposure and follows a biphasic course Typically begins as fever with nonproductive cough and may feature myalgia, fatigue, or retrosternal chest pain Transient clinical improvement may occur after the first few days, followed by rapid progression and clinical deterioration including the following:High-grade fever Symptoms of respiratory failure: severe dyspnea, tachypnea, hypoxemia Hematemesis or hemoptysis Chest pain, which may be severe enough to mimic acute coronary syndrome Decreased level of consciousness, meningismus, and coma (with meningeal involvement)

16. a 27yo asymptomatic male presents with generalized lymphadenopathy. he has multiple sexual partners infrequently uses condoms. of the following choices, which test should be performed? a. HIV test b. RPR c. lymph node biopsy d. ESR

a. HIV test

37. During a routine digital rectal exam, a hard, fixed nodule is detected on the prostate gland of a 55-year-old male patient. The patient's prostate-specific antigen (PSA) is 4.1 ng/mL. Which of the following will the nurse practitioner order? A. CT of the abdomen B. Radionuclide bone scan C. Measurement of serum acid phosphatase D. Transrectal ultrasound-guided needle biopsy

D) Transrectal ultrasound-guided needle biopsy. A PSA level of ≥4 ng/mL is considered an indication for biopsy in men >50 years. Although very high levels are significant (suggesting extracapsular extension of the tumor or metastases) and the likelihood of cancer increases with higher PSA levels, there is no cutoff below which there is no risk. CT or MRI of the abdomen and pelvis is commonly done to assess pelvic and retroperitoneal lymph nodes if the Gleason score is 8 to 10 and the PSA is >10 ng/mL. Radionuclide bone scans are rarely helpful for finding bone metastases (they are frequently abnormal because of the trauma of arthritic changes) until the PSA is >20 ng/mL. Elevated serum acid phosphatase correlates well with the presence of metastases, particularly in lymph nodes. However, this enzyme may also be elevated in benign prostatic hyperplasia (BPH) and is slightly elevated after vigorous prostatic massage.

24. Compared with UTI in younger women, uncomplicated UTI in an elderly woman is more likely to be associated with each of the following signs and symptoms except: A. new-onset urinary incontinence. B. delirium. C. weakness. D. hematuria.

D. hematuria.

10. Objective findings in patients with glomerulonephritis include all of the following except: A. edema. B. urinary red blood cell (RBC) casts. C. proteinuria. D. hypotension.

D. hypotension.

69. You see a 58-year-old woman who is being treated for a renal stone. Analysis of a stone passed in the urine reveals that it is composed of calcium oxalate. In counseling the patient about preventing future stones, you consider all of the following except: A. reducing sodium in her diet. B. limiting consumption of beets, rhubarb, nuts, and chocolate. C. encouraging her to get her daily calcium requirements from food. D. if calcium supplements are needed, this medication should be taken on an empty stomach.

D. if calcium supplements are needed, this medication should be taken on an empty stomach.

19. Diagnostic confirmation of glomerulonephritis typically requires A. urinalysis plus a CBC with differential B. abdominal CT scan C. kidney ultrasound D. kidney biopsy

D. kidney biopsy

28. You see a 34-year-old woman with an uncomplicated UTI. She is otherwise healthy but reports having a sulfa allergy. Appropriate therapy would include: A. TMP-SMX. B. amoxicillin. C. azithromycin. D. nitrofurantoin.

D. nitrofurantoin.

5. A 78-year-old man presents with fatigue and difficulty with bladder emptying. Examination reveals a distended bladder but is otherwise unremarkable. The blood urea nitrogen (BUN) is 88 mg/dL (31.4 mmol/L); the creatinine is 2.8 mg/dL (247.5 μmol/L). This clinical assessment is most consistent with: A. prerenal azotemia. B. acute glomerulonephritis. C. acute tubular necrosis. D. postrenal azotemia.

D. postrenal azotemia.

3. When counseling a woman about COC use, you advise that: A. long-term use of COC is discouraged because the body needs a "rest" from birth control pills from time to time. B. fertility is often delayed for many months after discontinuation of COC. C. there is an increase in the rate of breast cancer after protracted use of COC. D. premenstrual syndrome symptoms are often improved with use of COC.

D. premenstrual syndrome symptoms are often improved with use of COC.

S/s of COVID

Fever or chills Cough Shortness of breath or difficulty breathing Fatigue Muscle or body aches Headache New loss of taste or smell Sore throat Congestion or runny nose Nausea or vomiting Diarrhea Other reported symptoms have included the following: Sputum production Malaise Respiratory distress Neurologic (eg, headache, altered mentality) The most common serious manifestation of COVID-19 appears to be pneumonia.

What is the Treatment for Ebola?

General principles of care are as follows: Supportive therapy with attention to intravascular volume, electrolytes, nutrition, and comfort care is of benefit to the patient. Such therapy must be administered with strict attention to barrier isolation; all body fluids contain infectious virions and should be handled with great care. No specific therapy is available that has demonstrated efficacy in the treatment of Ebola hemorrhagic fever. Ebola Zaire vaccine is approved in Europe and the United States. The live recombinant vaccine has shown effectiveness of 97.5% in preventing infection among 90,000 individuals in an active Ebola virus outbreak in the Democratic Republic of Congo. The FDA approved atoltivimab/maftivimab/odesivimab (Inmazeb), a recombinant human monoclonal antibody combination. These antibodies target the glycoprotein (GP) on the Ebola virus surface, thereby blocking attachment and entry of the virus on host cell membranes.

89. a 40yo male has been dx with acute bacterial prostatitis. his PSA is elevated on dx. how soon should his PSA be rechecked? a. 4wks b. 2wks c. 1 wk d. 2-3d

a. 4wks

When is HIV most contagious and why do antivirals react with many other medications?

Though people living with HIV tend to be most infectious in the first few months after being infected, many are unaware of their status until the later stages. Seroconversion is the period when someone with HIV is at their most infectious. In the first few weeks after initial infection people may experience no symptoms or an influenza-like illness including fever, headache, rash or sore throat. ARVs used in the treatment of HIV are often prone to drug interactions because many of them are metabolized through the CYP450 system. Of the CYP450 isoenzymes, CYP3A4, CYP2D, and CYP2C9/19 are considered the primary isoenzymes involved in the drugs' metabolism. Examples of medications that interact with NNRTIs include azole antifungals (ketoconazole, itraconazole), rifamycins (eg, rifabutin, rifampin), benzodiazepines (eg, midazolam, triazolam), HMG-CoA reductase inhibitors (eg, lovastatin, simvastatin), and methadone.

38. a patient arrives at your clinic to discuss long-term options for contraception. in evaluating her circumstances you decide the implanon might work best for her. you might tell her all of the following regarding the implant EXCEPT: a. "odds are you will be able to maintain a regular cycle" b. "i would hold off on agreeing to implanon before i tell you everything about it" c. "this drug will be more expensive than the pill - at least, at first" d. "the implant may be visible, so take that into consideration"

a. "odds are you will be able to maintain a regular cycle" can lead to irregular or absent periods

61. anticipated organ survival exceeds 85% with testicular decompression within how many hours of torsion? a. 1 b. 6 c. 16 d. 24

a. 1

76. what is the approximate incubation period for Neisseria gonorrhoea? a. 1-5d b. 7-10d c. 18d d. 28d

a. 1-5d

81. during asymptomatic HSV-2 infection, genital shedding of the virus occurs during approximately _____ of days. a. 10% b. 25% c. 50% d. 100%

a. 10%

92. during asymptomatic HSV-2 infections, genital shedding of virus occurs during approximately ____ of days. a. 10% b. 25% c. 50% d. 100%

a. 10%

89. how long should a patient be treated with abx if he has prostatitis secondary to an STD? a. 14 days or longer b. 7-10d c. about 5 days d. 3 days

a. 14 days or longer prostate gland does not absorb abx readily. so abx must be given for 2-6wks to achieve high enough concentrations to tx effectively and eradicate organism.

37. implanon usually offers continuous birth control for how long? a. 3 yrs b. 4yrs c. 5yrs d. 6yrs

a. 3 yrs suppresses ovulation, altering viscosity of cervical mucus and preventing implantation of embryo in endrometrium through controlled release of etonogestrel

109. a patient with a primary case of scabies was probably infected: a. 3-4wks ago b. 2 weeks ago c. 1wk ago d. 1-3 d ago

a. 3-4wks ago this is the incubation period.

14. a patient was exposed to HIV through sexual intercourse last night. he was screened and found to be negative for HIV. when should he be screened again? a. 4-6wks b. 8wks c. 3mos d. 6mos

a. 4-6wks

46. a patient is diagnosed with Bell's palsy and prescribed high-dose steroids for 10 days with minimal improvement. during a follow-up visit, the NP suspects the pt has HSV-1. which med will the NP prescribe? a. acyclovir b. bactrim DS c. penicillin d. ciprofloxacin

a. acyclovir bell's palsy is caused by inflammation in CN VII. HSV is a virus and acyclovir is an antiviral med that should be given. the rest are abx and are not effective against virus.

27. your pt's DEXA scan reveals a value of -2.7. based on this finding, your best course of action at this time would be to recommend which of these therapies? a. alendronate b. increase in dietary calcium c. coumadin therapy once daily d. calcium supplements QD

a. alendronate indicates fully developed osteoporosis; so need a bisphosphonate to tx. dietary changes are not helpful at this point.

15. a 74yo woman is diagnosed with shingles. the NP is deciding how to best manage her care. what should be prescribed? a. an oral antiviral agent b. an oral antiviral agent plus an oral steroid c. an oral antiviral agent plus a topical steroid d. topical steroid only

a. an oral antiviral agent

37. you are treating 4 female pts who each exhibit predisposing factors for osteoporosis. which pt is LEAST likely to be at risk for osteoporosis? a. an overweight african american female with a family hx of uterine cancer b. a hispanic woman living a predominantly sedentary lifestyle c. an underweight asian woman with hx of tobacco use d. a caucasian woman experiencing early onset of menopause

a. an overweight african american female with a family hx of uterine cancer risk factors are predominantly sedentary lifestyle, caucasian and asian descent, underweight, tobacco use, early onset menopause

72. sequelae of genital HPV infection in a man can include: a. anorectal carcinoma b. low sperm count c. paraphimosis d. reiter syndrome

a. anorectal carcinoma

82. the most appropriate time to begin screening for renal nephropathy in a pt with T2DM is: a. at diagnosis b. once a year after diagnosis c. 2-3 after dx d. 5 years after dx

a. at diagnosis

31. shaun, a 30yo male, presents to the clinic with yellow-green penile discharge, testicular pain, and NV. given the most likely diagnosis, which of these drugs should you include in the regimen? a. azithromycin b. penicillin c. erythromycin d. valacyclovir

a. azithromycin first-line for uncomplicated gonorrhea as indicated by yellow-green penile discharge, testicular pain and NV. would use ZPAK + ceftriaxone or doxycycline to co-treat potential chlamydia co-infection. penicillin and erythromycin are used for tx syphilis. valacyclovir common tx for herpes.

128. a 25yo female presents to the NP with new s/s of difficulty urinating, increased frequency of urination, and mild suprapubic discomfort. urine dipstick identifies moderate number of leukocytes and nitrates. the NP orders: a. bactrim DS BID x 3 days b. bactrim BID x 5 days c. bactrim BID x 7 days d. bactrim BID x 10 days

a. bactrim BID x 3 days healthy women age 18-65 dx with uncomplicated UTI may be tx with 3 day. 5-10 days would be excessive and result in abx resistance and other issues such as c. diff

74. a patient newly diagnosed with DM has BMI of 32.5, creatinine of 1.4, BUN of 20, and FBS of 186. pt also has hx of bladder CA. which is the most appropriate drug class for her? a. biguanides b. thiazolidinediones c. meglitinides d. sulfonylureas

a. biguanides moderately obese with BMI of 32.5. BUN and creatinine are normal but need monitoring. metformin (biguanide) is preferred for obesity because it helps weight loss. sulfonylureas stimulate beta cells but have long half life and there is a high risk for hypoglycemic events. thiazolidinediones should not be used with hx of bladder CA or HF. meglitinides stimulate pancreatic release of insulin and are indicated for postprandial hyperglycemia.

62. the most common renal stones are composed of: a. calcium b. uric acid c. sodium d. iron

a. calcium

33. which of the following is recommended by the CDC as single-dose therapy for uncomplicated urethritis caused by N. gonorrhoeae when an oral product is most appropriate? a. cefixime b. metronidazole c. TMP-SMX d. amoxicillin

a. cefixime

85. which of the following agents is active against N. gonorrhoeae? a. ceftriaxone b. metronidazole c. ketoconazole d. amoxicillin

a. ceftriaxone

32. the preferred treatment for uncomplicated gonococcal proctitis is: a. ceftriaxone 250mg IM as a single dose plus a single dose of azithromycin 1g PO b. oral erythromycin 500mg BID for 7 days c. oral norfloxacin 400mg BID with metronidazole 500mg BID for 3 days d. azithromycin 1g PO as a single dose plus single dose of injectable doxycycline 100mg

a. ceftriaxone 250mg IM as a single dose plus a single dose of azithromycin 1g PO

80. a 24-year-old female presents with abdominal pain. what additional finding supports a diagnosis of PID? a. cervical motion tenderness b. positive RPR c. vaginal discharge d. dysuria

a. cervical motion tenderness difficult to dx and often unrecognized bc of varied presenting s/s. delay in dx contributes to inflammatory sequelae in upper reproductive tract. CDC recommends having low treshold for diagnosis. presumptive tx should be initiated in sexually active women if experiencing s/s of pelvic or lower abd pain if one or more is on exam: CMT, uterine tenderness or adnexal tenderness

39. henry, a 44 year old male is hospitalized and has an STD that is characterized by hemiparesis, hemiplegia, and cardiac insufficiency. given the most likely stage of henry's condition, which of the following s/s would you LEAST expect to see? a. chancre b. aortic aneurysm c. leukoplakia d. meningitis

a. chancre often a first sign that person has primary stage of syphilis, typically heals in 6 wks. hemiparesis, hemiplegia, cardiac insufficiency all indicate tertiary syphilis, which is years after initial infection. leukoplakia and aortic aneurysms and meningitis are also potential s/s of tertiary stage of syphilis.

84. a 30-year-old woman presents without symptoms but states that her male partner has dysuria without penile discharge. exam reveals a friable cervix covered with thick yellow discharge. this description is most consistent with an infection caused by: a. chlamydia trachomatis b. neisseria gonorrhoeae c. HPV d. trichomonas vaginalis

a. chlamydia trachomatis

30. a patient taking oral contraceptives may be at increased risk for which of the following conditions as the patient's age, dose and length of therapy increase? a. hypertension b. T2DM c. abnormal menstrual bleeding d. hypercholesterolemia

a. hypertension

100. which assessment finding is associated with a diagnosis of proximal hypospadias? a. chordee b. hydrocele c. micropenis d. inguinal hernia

a. chordee presence of ventral shortening and curvature of penis, commonly associated with more proximal urethral defect. hydrocele, micropenis and inguinal hernia are not associated with proximal hypospadias.

93. when a positive HIV result is obtained, which of the following statements is most accurate in communicating the results to the patient? a. communication of positive results should be done only through face-to-face contact b. communication via a first degree relative is appropriate if the patient is not proficient in english language c. text messaging or email can be used to communicate results if the contact info is provided by the patient d. it is appropriate to inform the spouse of a positive test result if the patient is unavailable

a. communication of positive results should be done only through face-to-face contact

129. when a positive HIV result is obtained, which of the following statements is most accurate in communicating the results to the patient? a. communication of positive results should only be done through in-person contact b. communication via a first-degree relative is appropriate if the pt is not proficient in english language c. text message or email can be used to communicate results if contact info is provided by pt and pt agrees to this at the time of testing d. it is appropriate to inform the spouse of a positive test result if the pt is unavailable

a. communication of positive results should only be done through in-person contact

11. what is the recommendation from the american cancer society for assessment of the prostate gland in a man who is 45yrs old and average risk for development of prostate cancer? a. consider screening at 50+ yrs b. PSA now c. PSA and DRE now d. DRE only

a. consider screening at 50+ yrs high risk pts begin at age 40-45 (african american, men with first degree relative with prostate cancer diagnosed prior to age 65), should be done in men expected to live at least 10 more years. if PSA > 2.5, annual testing should take place. if < 2.5 test every 2 yrs. USPSTF does not recommend screening.

15. a male presents to the clinic complaining of a painful ulcer on his genitals. a close exam reveals that the ulcer is surrounded by an erythematous halo. given the most likely condition, which of the following abx would NOT be prescribed? a. doxycycline b. ceftriaxone c. azithromycin d. ciprofloxacin

a. doxycycline not normally used for chancroid, condition often presenting with painful ulcers surrounded by erythematous halos. doxy more likely used for chlamydia or LGV. chancroid often is given ceftriaxone, azithromycin or cipro.

6. a male pt presents to you with flu-like symptoms and a rash on his palmar surfaces. the rash itself is characterized by rough, reddish-brown dots. further exam shows enlarged lymph nodes. following a diagnostic, you want to prescribe him medication but his records show that he is allergic to penicillin. based on the most likely condition, which two abx would serve as suitable tx alternatives? a. doxycyline or erythromycin b. erythromycin or ciprofloxacin c. ciprofloxacin or doxycycline d. acyclovir or erythromycin

a. doxycyline or erythromycin for pts allergic to penicillin with secondary syphilis, (flu-like s/s, palmar skin rash, and lymphadenopathy), you can do doxycycline or erythromycin. cipro is good for genital ulcers but not syphilis. acyclovir is good for herpes.

5. hematuria is NOT a common clinical manifestation in: a. early prostate cancer b. BPH c. bladder cancer d. renal cancer

a. early prostate cancer rarely any symptoms

66. when reviewing the use of nutritional supplements for the management of menopausal symptoms, the NP considers that: a. few high quality studies support the use of these products b. the use of these products is consistently reported to be helpful c. the products can be safely used as long as blood hormone levels are carefully evaluated d. the use of these products is associated with a greater reduction in menopausal symptoms than with prescription HT

a. few high quality studies support the use of these products

30. "hot flashes" that occur during menopause are thought to be related to: a. fluctuating estrogen levels b. fluctuating progesterone levels c. low progesterone levels d. low estrogen levels

a. fluctuating estrogen levels

68. which of the following representative of the presentation of secondary syphilis? a. generalized rash b. chancre c. pupillary alterations d. aortic regurgitation

a. generalized rash

49. the most common causative organisms in chronic bacterial prostatitis include: a. gram-negative rods b. gram-positive cocci c. gram-negative cocci d. gram-positive coccobacilli

a. gram-negative rods

1. the practitioner knows that which of the following agents is LEAST effective in treating candida balanitis? a. griseofulvin b. clotrimazole c. steroids d. miconazole

a. griseofulvin mostly seen in skin infections such as tinea capitis, tinea corporis and tinea cruris and fungal infections of the scalp, fingernails and toenails. miconazole, clotrimazole and steroids are used to tx balanitis.

55. which behavioral intervention is used for functional incontinence?

a. having an assistant who is aware of voiding cues and helps with toileting activities

6. what is the recommendation of the american cancer society for screening average risk 40yo caucasian male for prostate cancer? a. he should be screened starting at age 50 years b. DRE and PSA c. serum PSA d. DRE

a. he should be screened starting at age 50 years

57. an elderly male presents with complaints of severe pain on his left back and a concentrated swath of vesicles. which dx is most likely? a. herpes zoster b. varicella c. tinea corporis d. plaque psoriasis

a. herpes zoster shingles is caused by infection with herpes varicella-zoster virus, the same virus that causes chickenpox. commonly causes severe, deep pain along a peripheral nerve on trunk of body and red, nodular skin legions. fever and malaise typically accompany. more frequent in geriatrics. chickenpox, also known as varicella, is a highly contagious infection caused by the varicella-zoster virus. there is a blister-like rash, which first appears on face & trunk & spreads throughout the body. plaque psoriasis is a chronic autoimmune condition. it appears on skin in patches of thick, red, scaly skin. tinea corporis is a dermatophytosis that causes pink-to-red annular (O-shaped) patches and plaques with raised, scaly borders that expand peripherally and tend to clear centrally.

28. ashley, 22yo female, is discussing various forms of contraception with you. she says she has heard some great things about IUDs but would like to be informed about the disadvantages of them. which of the following disadvantages is most commonly associated with IUDs? a. increased risk of PID after insertion b. annual need for maintenance and reinsertion c. increased risk of asherman's syndrome d. high levels of adverse estrogenic effects

a. increased risk of PID after insertion due to effect of IUD on microbiologic environment of vagina. some link IUDs with development of intrauterine adhesions characteristic of Asherman's syndrome. can remain in place for up to 10 years.

46. the first sign that a male child is experiencing sexual maturation is: a. increased testicular size b. enlargement of the scrotum c. increase in length of the penis d. scrotal and penile changes

a. increased testicular size tanner stage II, srotum begins to thin, redden and enlarge. penile length remains the same.

40. in contrast to progestin-only emergency contraception a possible mechanism of action of ulipristal (ella) is: a. inhibiting embryo implantation b. impairing sperm transport c. through spontaneous abortion d. impairing ovum transport

a. inhibiting embryo implantation

90. a patient is at increased risk of osteopenia if she uses which form of birth control? a. injectable progestin b. IUD c. oral contraceptives d. natural family planning

a. injectable progestin

106. a 75-year-old has isolated systolic hypertension. she started on amlodipine 4 weeks ago. she states that since then, she has developed urinary incontinence. what is the NP's assessment? a. it is probably related to amlodipine b. she may have underlying urinary tract pathology c. it is coincidental d. this is unrelated to her new medication

a. it is probably related to amlodipine calcium is responsible for muscle contraction, so sometimes CCBs worsen or produce urinary incontinence by impairing detrusor muscle contraction. lower dose can be tried or different class

12. mrs. jackson complains of urinary incontinence when she laughs or sneezes. what should be used first line to tx her s/s? a. kegal exercises b. prescribe oxybutinin c. avoid caffeine and alcohol d. minimize fluids at nighttime

a. kegal exercises

106. the NP has diagnosed a 78-year-old female pt with stress urinary incontinence. which of the following is most appropriate tx for this pt? a. kegels b. PT c. oxybutynin (ditropan) d. terazosin (hytrin)

a. kegels oxybutinin is appropriate for urge incontinence. another option is imipramine, TCA. therapy for gait training and strengthening is for functional incontinence. terazosin is for men with BPH.

67. a 51-year-old woman is using vaginal estrogen preparation with effect to manage localized menopause s/s. anticipated effects of this therapy include a potential for: a. lower rate of UTI b. endometrial atrophy c. fewer vasomotor s/s d. resumption of menses

a. lower rate of UTI

64. which of the following is a common finding in a man with varicocele? a. lower sperm count with increased number of abnormal forms b. increased rate of testicular cancer c. recurrent scrotal pain d. BPH

a. lower sperm count with increased number of abnormal forms

29. when using IUDs, which of the following mechanisms of action is typically caused by local foreign body inflammatory responses? a. lysis of the blastocyst b. thickening of cervical mucus c. atrophy of endometrial lining d. inhibition of sperm binding to egg

a. lysis of the blastocyst

68. which of the following statements is true? a. many OTC progesterone creams contain sterols that the human body is unable to use b. all progesterones are easily absorbed via the skin c. alfalfa is an example of phytoprogrestone d. progesterones, whether synthetic or plant based, should not be used by a woman who has undergone a hysterectomy

a. many OTC progesterone creams contain sterols that the human body is unable to use

45. which of the following statements regarding perimenopause is false? a. menstruation ceases during perimenopause b. hot flashes and flushes are common during the week before menses c. pregnancy is still possible during perimenopause d. ovulation becomes more erratic during perimenopause

a. menstruation ceases during perimenopause

53. the NP refers a 12 year old boy with a swollen, red scrotum and ascending testicle to the ED with a diagnosis of testicular torsion. which additional objective finding led the NP to this diagnosis? a. missing cremasteric reflex b. present cremasteric reflex c. hematuria d. pyuria

a. missing cremasteric reflex torsion presents with sudden onset of pain in scrotum/testicles and initially may be felt in the abdomen and groin. the scrotum will be swollen & in severe cases acute hydrocele is present, and the affected testicle is higher than the unaffected one. severe NV are common. cremasteric reflex is missing, not present. if not corrected within 24 hours, 100% of testicles become gangrenous and must be surgically removed. UA usually normal.

26. a patient with lesions presents to the clinic. a diagnosis of syphilis would most likely be confirmed if the lesions are: a. mucous b. pearly-white c. soft d. painful

a. mucous rash may also show up after initial sore has healed. pearly-white papules are mostly molluscum contagiosum, lesions are often smooth & rounded. initial sores from syphilis are firm and painless, rather than soft & painful.

140. which of the following describes the most common clinical presentation of trichomonas in a male? a. no clinical symptoms b. urethritis c. burning with urination d. testicular pain

a. no clinical symptoms

130. a localized tumor in prostate gland associated with early-stage prostate cancer is likely to produce: a. no s/s b. urinary frequency c. low back pain d. urinary hesitancy

a. no s/s

81. a 65-year-old woman who has had T2DM for 20yrs is concerned about her kidneys. she has a hx of 3 UTIs within past 8mos but is currently asymptomatic. the NP will: a. recheck urine during visit, send a urine specimen for C&S, refer to nephrologist b. order UA dipstick to be repeated monthly c. order CT of kidneys d. provide empiric tx for UTI

a. recheck urine during visit, send a urine specimen for C&S, refer to nephrologist 3 UTIs in 8 months warrants testing in office. UTI = 100,000 > organisms per mL of asymptomatic pts or > 100 organisms with pyuria (> 7 WBCs) in symptomatic pt. diabetics are higher risk for UTI and may develop bladder damage (cystopathy) and nephropathy. nephrology consult needed.

47. The NP notes that an 82yo male pt's penile gland and foreskin are inflamed. A reddened, moist macular lesion is present on the prepuce. The pt is being tx for a UTI with amoxicillin. The NP will: a. obtain a swab of lesion for KOH prep b. prepare the client for needle biopsy of the lesion c. obtain serum sample for RPR test d. prescribe 0.05% betamethasone cream BID for 4 wks

a. obtain swab of lesion for KOH prep s/s associated with balanoposthitis. fungal infections especially candida infections, are commonly associated with use of abx and are most common identifiable infections etiology associated with balanoposthitis. therefore, NP will obtain KOH prep to confirm dx. there is no indication that pt requires biopsy. RPR test used to screen for syphilis, pt does not present with chancre-type lesion, or other findings consistent with syphilis. Prior to giving betamethasone cream, the NP will confirm causative agent associated with dx, can promote exacerbation of fungal infection.

105. which of the following is a treatment option for a 30yo woman with PID and a hx of severe hive-form reaction when taking a penicillin or cephalosporin? a. ofloxacin with metronidazole b. amoxicillin with gentamicin c. cefixime with vancomycin d. clindamycin with azithromycin

a. ofloxacin with metronidazole

63. a 25yo woman complains of dysuria, severe vaginal pruritis and malodorous vaginal discharge. pelvic exam reveals a strawberry-colored cervix and frothy yellow discharge. microscopic exam of discharge reveals mobile organisms that have flagella. the correct pharmacologic therapy for the condition is: a. oral metronidazole (flagyl) b. ceftriaxone sodium (rocephin) c. doxycycline hyclate (vibramycin) d. clotrimazole (gyne-lotrimin) cream or suppositories

a. oral metronidazole (flagyl) trichomoniasis s/s include dysuria, severe vaginal pruritus, and malodorous vaginal discharge. wet prep microscopic exam should show trichomonads that are pear shaped and have several flagella (whiplike tails) at one end. the CDC recommends tx is flagyl. single dose is effective for most cases.

63. a 21yo woman complains to you of a 1wk episode of dysuria, frequency, and strong odor to her urine. this is her second episode of the year. the previous UTI occurred 3 months ago. what is the most appropriate f/u for the pt? a. order a UA and urine C&S and tx pt with abx b. order a urine C&S and hold tx until you get the results from the lab c. tx pt with 7d course of abx and order urine C&S now and after she completes her abx d. tx pt with stronger drug as such olfloxacin for 10 days

a. order a UA and urine C&S and tx pt with abx since this is the 2nd UTI for the year and last episode was 3 months ago, best action to identify organism causing the UTI. start empiric tx with an abx for 7d duration. do not use 3 day regimen.

48. pt presents with painful urination, lower abdominal pain, incomplete bladder elimination, and cloudy urine. the NP will: a. order urine c/s with E. coli b. perform microscopic urine exam for C. diff c. order IV pyelogram d. perform routine urine dipstick for RBCs and bacteria

a. order urine C&S for e. coli pt has s/s of UTI, majority are caused by E. coli.

70. common sites of chlamydia trachomatis infection in women include all of the following except: a. ovaries b. cervix c. endometrium d. urethra

a. ovaries

20. which type of incontinence would NOT benefit from pelvic floor muscle training? a. overflow b. mixed c. urge d. stress

a. overflow

32. nancy, a 24yo was engaging in intercourse with her boyfriend 3 days ago when the condom broke. she seeks emergency contraception; however, in the past, she has experienced severe NV after using levonorgestrel products. which of the following products would be best suited for nancy at this time? a. paragard b. implanon c. mirena d. patch

a. paragard can be used at alternative emergency contraception within 5-6 days of intercourse.

118. first-line treatment options for primary syphilis include: a. penicillin b. ciprofloxacin c. erythromycin d. ceftriaxone

a. penicillin

71. first-line treatment options for primary syphilis include: a. penicillin b. ciprofloxacin c. erythromycin d. ceftriaxone

a. penicillin

125. a female pt complains of dysuria and vaginal discharge. how should she be managed? a. perform an abdominal exam, UA, pelvic b. perform a pelvic exam c. order a urine cx d. order a UA

a. perform an abdominal exam, UA, pelvic

14. Bactrim is given to AIDS pts as PPX for which disease? a. pneumocystitis pneumonia b. progressive multifocal leukocenphalopathy c. coccidiodomycosis d. histoplasmosis

a. pneumocystitis pneumonia PPX for pneumocystitis jiroveci. progressive multifocal leukocephalopathy caused by john cunningham virus, has no PPX. coccidiodomycosis is tx with antifungals. there is only rare cases where PPX is used, which is fluconazole. PPX rarely used for histoplasmosis, but would use itraconazole is first choice for immunocompromised pts.

3. you order a referral to further evaluate your 19yo pt who reported s/s of cramping, pain in her upper thighs, nausea and fatigue during the first few days of her menstrual cycle. for which of the following do you make a referral? a. primary dysmenorrhea b. secondary dysmenorrhea c. endometriosis d. premenstrual dysphoric disorder

a. primary dysmenorrhea needs to find out if based on her s/s if her body is producing too much prostaglandin. secondary dysmenorrhea occurs in women age 20 or older and is most often associated with pelvic disease. endometriosis may present with dysmenorrhea, but it typically presents with heavy or irregular bleeding, as opposed to regular flow. PDD may produce with pain, but begins prior to menses and ends when menses begins

3. when assessing a 78-year-old man with suspected BPH, the NP considers that: a. prostate size does not correlate well with severity of symptoms b. BPH affects less than 50% of men of this age c. he is at increased risk for prostate cancer d. limiting fluids is a helpful method of relieving severe symptoms

a. prostate size does not correlate well with severity of symptoms

23. which of the following would be most indicative of an initial herpes outbreak? a. pruritic ulcers lasting 12 days b. pruritic and exuding ulcers lasting 5 days c. ulcers surrounded by an erythematous halo d. painless ulcers

a. pruritic ulcers lasting 12 days recurrent herpes usually has pruritic ulcers that last for 5 days. do not typically exude. ulcers surrounded by erythematous halo are typical of chancroid, not herpes.

60. a health 32yo female has left flank pain, nausea, fever (101F). her UA demonstrates hematuria and presence of leukocytes, nitrites and WBC casts. what is the most likely diagnosis? a. pyelonephritis b. cholecystitis c. renal stone d. UTI

a. pyelonephritis

40. sharon, a 24yo, calls the clinic inquiring about her nuvaring. she says that her ring, which had been in place for 2wks, fell out about 4 hours prior. she is worried that this will throw off her contraceptive schedule. what instructions would be most effective in helping sharon continue on her contraceptive schedule? a. rinse the ring with cool water. reinsert and use a spermicide or barrier for 1wk b. reinsert the thing immediately c. discard the ring and insert a new ring immediately d. wait until the current 21 day period is over and start a new ring

a. rinse the ring with cool water. reinsert and use a spermicide or barrier for 1wk if nuvaring is displaced for more than 3 hours within first 2 weeks of use, it is best to rinse with cool water and reinsert ASAP and use spermicide or barrier method in conjunction for next 7 days. if < 3 hours displaced, replace as soon as possible. if > 3 hours in third week of use, ring should be discarded and new ring should be inserted immediately.

35. in gonococcal infection, which of the following statements is true? a. risk of transmission from an infected woman to a male sexual partner is about 20-30% with a single coital act b. most men have symptomatic infection c. the incubation period is about 2-3wks d. the organism rarely produces beta-lactamase

a. risk of transmission from an infected woman to a male sexual partner is about 20-30% with a single coital act

146. a female pt who takes oral contraceptives has just completed her morning exercise routine. she complains of pain in her RIGHT calf. her blood pressure and heart rate are normal. she is not short of breath. her calf is red and warm to touch touch. what is NOT part of the differential diagnosis? a. sciatica b. calf muscle strain c. cellulitis d. DVT

a. sciatica

78. the NP prescribes an ACEI for a pt with kidney disease. the NP orders which of the following testing to monitor the pt? a. serum K+ b. serum hgb c. US of kidney d. US of abd

a. serum potassium K may rise initially and begin to taper off 2-3 mos later, continued monitoring needed.

8. which finding from a patient's cervical cytology results would best confirm condyloma acuminata? a. squamous intraepithelial lesions b. multinucleated giant cells c. infection d. parabasal cells

a. squamous intraepithelial lesions multinucleated giant cells from cervical cytology smear generally indicates HSV. if indicated infection, pt should be further tested for trichomoniasis, vaginosis, herpes and chlamydia, rather than condyloma acuminata. parabasal cells are often confused with dysplastic cells, but parabasal cells are normal and usually are prior to menarche or after menopause.

34. a patient who has recently started on the patch mentions that she is also pursuing an herbal regimen for various health issues. which of the following herbs would be most likely to affect her tx with the patch? a. st. john's wort b. ginger c. echinacea d. ginseng

a. st. john's wort diminishes therapeutic effect of estrogens while decreased serum concentration of CYP3A4 substrates, thus creating contraceptive failure.

2. a 22-year-old woman is taking a 35-mcg ethinyl estradiol COC calls after forgetting to take her pills 2 consecutive days. she is 2 wks into the pack. you advise her to: a. take the last pill missed immediately, even if this means taking two pills today b. discard two pills and take two pills today c. discard the rest of the pack and start a new pack with the first day of her next menses d. continue taking one pill daily for the rest of the cycle

a. take the last pill missed immediately, even if this means taking two pills today

28. which of the following findings is NOT a common somatic symptom of PMS? a. tension b. headaches c. breast tenderness d. poor coordination

a. tension this is an emotional symptom which may be accompanied by irritability, confusion and mood swings. headaches, breast tenderness and poor coordination are somatic s/s.

18. which of these represents the standard of care for men with a diagnosis of symptomatic BPH? a. terazosin (hytrin) b. saw palmetto c. avanafil (stendra) d. dutasteride (avodart)

a. terazosin (hytrin) this is an alpha-blocker, which provides immediate relief of symptoms by relaxing muscles of bladder & prostate. dutasteride & other 5-alpha reductase inhibitors may also see use in shrinking enlarged prostates, however, these drugs are primarily effective in pt's with larger prostates who cannot tolerate alpha-blockers. saw palmetto is an herb that is inconclusive full efficacy. phosdiesterase-5 inhibitors, such as avanafil, may be used to tx concomitant ED and BPH but it is limited evidence.

10. what is the primary role of the PCR in the treatment of a patient with HIV? a. the PCR helps quantitate the viral count of HIV in an infected person b. the PCR confirms HIV-positive status following a positive result on an ELISA test c. the PCR helps determine the number of CD4 cells in a patient with confirmed HIV d. the PCR helps determine when a person will undergo seroconversion

a. the PCR helps quantitate the viral count of HIV in an infected person

55. during perimenopause, which of the following is likely to be noted? a. the length of menstrual cycle and duration of menstrual flow are often unpredictable b. the length of perimenopausal period is predictable c. symptoms are less severe in women who smoke d. hot flashes are uncommon

a. the length of menstrual cycle and duration of menstrual flow are often unpredictable

20. the presentation of acute epididymitis is an otherwise well 22yo man includes: a. the presence of a positive phren sign b. low back pain c. absent cresmasteric reflex d. diffuse abdominal pain

a. the presence of a positive phren sign

2. which of these statements is true regarding the use of condoms? a. there is a higher reported failure of female condoms than male condoms b. natural skin condoms give the most protection against STDs c. the failure rate of condoms is the lowest of all barrier contraceptives d. leaving empty space at the end of the condom increases the risk of breakage

a. there is a higher reported failure of female condoms than male condoms latex condoms provide greatest degree of protection, natural skin condoms do not protect against STDs. sponge has lowest possible failure rate of barrier contraceptives. leaving 1/2 inch of space at end of condom decreases risk of breakage.

33. all of the following keratolytic agents are routinely used to tx condyloma acuminata EXCEPT: a. podophyllin b. 5-fluorouracil c. trichloroacetic acid d. bichloroacetic acid

b. 5-fluorouracil used to tx condyloma acuminata, but it is no longer recommended for routine use due to risk of severe toxic reaction. podophyllin, trichloroacetic acid and chloracetic acid are all recommended tx for condyloma acuminata.

135. a 20yo male is being seen for physical exam by the NP. he complains of pruritic macerated areas in his groin that have been present for past 2wks. which of the following is most likely? a. tinea cruris b. tinea corporis c. tinea capitis d. tinea pedis

a. tinea cruris jock itch is a common skin infection that is caused by a type of fungus called tinea. fungus thrives in warm, moist areas of the body, and as a result, infection can affect the genitals, inner thighs, and buttocks. infections more often occur in summer or warm, wet climates. tinea cruris appears as red, itchy rash that is often ring shaped. tinea corporis involves body, capitis = head, pedis = feet.

107. treatment options for patients with condyloma acuminatum include all of the following except: a. topical acyclovir b. cryotherapy c. podofilox d. trichloroacetic acid

a. topical acyclovir

22. if sponge is left in place for too long, a patient typically is at serious risk for which of the following conditions? a. toxic shock syndrome b. trichomoniasis c. anemia d. amenorrhea

a. toxic shock syndrome may increase risk for candidiasis. IUD may increase risk of anemia d/t increased menstrual bleeding, depo-provera can cause amenorrhea.

102. a transvaginal US in the woman with PID will likely show: a. tubal thickening with or without free pelvic fluid b. cervical thickening c. endometrial thinning d. inflammation of the ovaries

a. tubal thickening with or without free pelvic fluid

24. a female pt has been diagnosed with chlamydia. how should this be managed? a. tx with azithromycin b. tx with ceftriaxone by injection c. tx with doxycycline d. tx for gonorrhea also

a. tx with azithromycin

50. absolute contraindications to postmenopausal HT include: a. unexplained vaginal bleeding b. seizure disorder c. dyslipidemia d. migraine headache

a. unexplained vaginal bleeding

23. which of the following is most likely to be part of the clinical presentation of an otherwise healthy 27yo woman with uncomplicated lower UTI? a. urinary frequency b. fever c. suprapubic tenderness d. lower GI upset

a. urinary frequency

65. an 8yo boy with T1DM is being seen for 3d hx of urinary frequency and nocturia. he denies flank pain and is afebrile. the UA result is negative for blood and nitrites but is positive for large amount of leukocytes and ketones. he has a trace amount of protein. which of following is the best test to order initially? a. urine C&S b. 24-hour urine for protein & creatinine clearance c. 24-hour urine for microalbumin d. IV pyelogram

a. urine C&S high risk for UTIs. large amount of leukocytes is abnormal, and he has been having s/s of frequency and nocturia for past 3 days. need to r/o UTI.

123. a 24-year-old female has been dx with an uncomplicated UTI. which assessment below is least important at this time? a. vaginal exam b. assessment of CVA tenderness c. abdominal exam d. body temperature

a. vaginal exam

40. all of the following tests would be used to dx PID EXCEPT a: a. venereal disease research lab test b. ESR test c. CRP test d. US

a. venereal disease research lab test you should perform cx for gonorrhea and chlamydia to confirm PID. venereal disease includes syphilis. increased ESR and CRP improve specificity of diagnosis, US helps isolate ovarian cysts.

73. which of the following best describes lesions associated with condyloma acuminatum? a. verruciform b. plaque-like c. vesicular d. bullous

a. verruciform

18. which risk factor has the greatest impact on HIV transmission? a. viral load b. type of sexual activity c. presence of other STDs d. patient gender

a. viral load

58. which body area has the greatest concentration of estrogen receptors? a. vulva b. vascular bed c. heart d. brain

a. vulva

21. your pt complains that she has "gotten shorter over the years" and noticed a "hump" on the back of her neck. as she walks over to the exam table, you notice that she stoops forward. based on the pt's condition, which of the following would you be most likely to recommend to help her manage her condition? a. yogurt and green leafy vegetables b. red meats and iron-fortified breads c. salmon and beans d. fruits and raw vegetables

a. yogurt and green leafy vegetables calcium supplementation is mainstay of tx for osteoporosis as indicated by complaints.

An elderly male resident in a long-term care facility has suddenly developed a fever of 102.3°F and is complaining of chills and pain that radiates from the perineal area to the rectum and back. The prostate is extremely tender, warm, and boggy upon digital rectal examination (DRE). Urinalysis reveals pyuria and hematuria. Which diagnosis is most likely?

acute prostatitis

21. you are meeting with a patient who is HIV-positive. the patient's CD4 lymphocyte percentage has fallen below 20%. what would you most likely say when discussing with this patient? a. "the lower lymphocyte percentage indicates seroconversion." b. "the risk of your HIV infection progressing to AIDS has increased." c. "as your CD4 lymphocyte percentage drops, your viral load usually drops." d. "now that your CD4 lymphocyte percentage is below 20%, you can start antiretroviral therapy."

b. "the risk of your HIV infection progressing to AIDS has increased." seroconversion is process of converting from HIV negative to HIV positive; which would have already occurred in pt & is not indicated by lower lymphocyte %. viral load results closely correlate with progression of HIV, but low CD4 % would result in high viral loads, not lower. when to start antiretroviral tx is usually according to CDC by the time CD4 count reaches 350/uL.

62. the typical HT regimen contains _____ or less of the estrogen dose of COC a. 1/8 b. 1/4 c. 1/2 d. 3/4

b. 1/4

78. approximately what % of sexually active adults has serological evidence of HSV-2? a. 5 b. 15 c. 25 d. 40

b. 15

91. a 73yo male pt presents with b/l edema in legs & feet, weight gain of 15lbs, decreased urine output. the NP will order: a. hgb testing b. 24-hr creatinine level c. urine C&S d. abd US

b. 24hr creatinine s/s acute renal failure. may also c/o lack of appetite, nausea, lethargy. his serum creatinine level will be elevated. hgb testing provides info on level of protein available to carry O2 in the blood. examine for ARF and then do MRI or IVP.

105. how long should a female patient with an uncomplicated UTI be treated with oral abx? a. 1 days b. 3 days c. 5 days d. 7 days

b. 3 days

16. which of the following is the most likely candidate to initiate dialysis resulting from chronic kidney disease? a. 46-year-old man with hypertension & GFR = 42 b. 64yo woman with T2DM & GFR = 28 c. a 76-year-old man with anemia & GFR = 55 d. 58yo woman with heart disease & GFR = 46

b. 64yo woman with T2DM & GFR = 28

11. which statement most accurately reflects the relationship between HIV and CD4 cells? a. HIV infects CD4 cells and maintains the compromised cell for virus spreading b. HIV uses CD4 cells as receptors and reservoirs c. HIV causes an excessive increase in CD4 cells d. HIV causes CD4 cells to attack the body

b. HIV uses CD4 cells as receptors and reservoirs subsequently destroys the cells, resulting in dramatic decline, not increase, of overall CD4 count. although compromised CD4 cells transmit the virus through the body, the primary means of transmission is destroying the compromised cell to produce virus particles, not maintaining the cell to aid transmission

46. appropriate antimicrobial tx for a 25yo man with acute bacterial prostatitis is: a. oral azithromycin b. IM ceftriaxone followed by oral doxycycline c. oral ofloxacin d. oral amoxicillin-clavulanate

b. IM ceftriaxone followed by oral

8. an initial pharmacologic approach to a patient who is diagnosed with primary dysmenorrhea could be: a. acetaminophen b. NSAIDs at the time symptoms begin or onset of menses c. NSAIDs prior to onset of menses d. combo of acetaminophen and NSAIDs

b. NSAIDs at the time symptoms begin or onset of menses pain associated with dysmenorrhea is likely due to prostaglandins, which can cause prolonged contraction of the uterus. this produces ischemia, sometimes termed uterine angina. NSAIDs are prostaglandin inhibitors. these are usually started at the onset of menses or onset of s/s and continued for 2-3d depending on symptom pattern of pt.

23. a 26-year-old mother who breastfeeds her 4-month-old child queries about hormonal contraceptives. in counseling her on the useof progestin-only pill, you mention all of the following except: a. the pill is taken every day b. POP is more effective than COC c. POP does not alter the quality of quantity of breast milk d. POP is associated with bleeding irregularity, ranging from prolonged flow to amenorrhea

b. POP is more effective than COC

118. a sexually active adult female presents to the clinic c/o genital ulcers. upon exam, white wartlike papules are found in the inguinal area and a maculopapular rash is noted on the palms and soles of the feet. the NP will order: a. gram stain b. rapid plasma reagin test c. NAAT d. HIV-1/HIV-2 antibody and P24 antigen test

b. RPR these wartlike lesions condylomata lata and rash on palms and soles of feet indicate a probable syphilis infection. they do 2 different types of tests: treponemal and nontreponemal, needed to dx. step 1 - order screening, nontreponemal - RPR or VDRL if reactive, step 2 - order confirmatory, treponemal tests: fluorescent treponemal antibody absorption (FTA-ABS), microhemagglutination test for antibodies to treponema pallidum (MHA-TP). a gram stain is to look for gram negative diplococci cx not used in primary care, it is useful for gonorrhea. NAATs are good for gonorrhea and chlamydia.

5. a male pt complains of dysuria. his UA is positive for nitrites, leukocyte esterase and bacteria. what med should be given and for how many days? a. doxycycline for 7d b. TMPS for 7-10d c. ciprofloxacin for 3d d. nitrofurantoin for 14d

b. TMPS for 7-10d

42. a 14yo male pt has an acute, painless groin swelling. which tool would yield the most info to identify etiology of the swelling? a. MRI of scrotum b. US of scrotum c. US of abd d. abd radiographs

b. US of scrotum

24. by using a diaphragm with spermicide nonoxynol-9 during sexual intercourse, a woman is likely at increased risk for: a. cervical stenosis b. UTI c. increased perivaginal lactobacilli colonization d. ovarian malignancy

b. UTI

107. a female pt with a complaint of dysuria has a urine specimen that is positive for leukocytes and nitrites. there is blood in the specimen. the most appropriate diagnosis is: a. UTI or chlamydia b. UTI with hematuria c. asymptomatic bacteriuria d. UTI

b. UTI with hematuria leukocytes and nitrites most likely infection in bladder. blood is common when pt has UTI. cannot dx chlamydia from this.

45. a 30-year-old man with prostatitis presents with a fever of 102.3F. What would be the expected CBC findings from this pt? a. WBC = 15000; neutrophils 4000 b. WBC = 18000; neutrophils 11,500 c. WBC = 7200; neutrophils 3200 d. WBC = 4000; neutrophils 1200

b. WBC = 18000; neutrophils 11,500

8. due to an increased risk of blood clots, an alternative to the contraceptive ring or patch is preferred in all of the following women except: a. a 42yo nulliparous woman with T2DM and high LDL b. a 31yo woman with hx of naturally occurring multiple gestation pregnancy c. a 28yo who smokes > 15 cigs per day d. a 33yo with a family hx of venous thrombosis

b. a 31yo woman with hx of naturally occurring multiple gestation pregnancy

2. of the following, who is most likely to experience symptoms of PMS/premenstrual dysphoric disorder? a. a 17yo athlete on a strict vegan diet b. a fit, healthy 33yo with a high-stress job c. a 45yo who regularly practices yoga d. a 24yo artist who drinks caffeine

b. a fit, healthy 33yo with a high-stress job combination of age and occupational stress. incidence of PMS and PDD slowly increased during adolescence, peaks during 30s, and declines in 40s.

23. high amounts of estrogen may cause several adverse effects associated with oral contraceptive use. which of the following adverse effects is NOT typically caused by high amounts of estrogen? a. nausea b. acne c. edema d. breast tenderness

b. acne worsening facial acne usually result of excess androgens not higher estrogen. may cause nausea, edema and breast tenderness.

26. the reduction in free androgens noted in a woman using COC can yield an improvement in: a. cycle control b. acne vulgaris c. breast tenderness d. rheumatoid arthritis

b. acne vulgaris

38. which of the following is likely not among the proposed mechanisms of action of all forms of oral emergency contraception? a. inhibits ovulation b. acts as an abortifacient c. slows sperm transport d. slows ovum transport

b. acts as an abortifacient

103. which of the following effects is seen in women using depo-provera (medroxyprogesterone injection) for more than 5 years? a. melasma b. amenorrhea c. weight loss d. headache

b. amenorrhea

2. when prescribing antihypertensive therapy for a man with BPH and HTN, the NP considers that: a. loop diuretics are the tx of choice b. an alpha1-antagonist should not be used as a solo or first-line therapeutic agent c. angiotensin receptor antagonist use is contraindicated d. beta-adrenergic antagonist use often enhances urinary flow

b. an alpha1-antagonist should not be used as a solo or first-line therapeutic agent

52. you perform a DRE on a 72yo man and find a lesion suspicious for prostate cancer. the findings are described as: a. a rubbery, enlarged prostatic lobe b. an area of prostatic induration c. a boggy gland d. prostatic tenderness

b. an area of prostatic induration

89. which assessment question will the NP ask a 68yo pt who has been receiving ongoing tx with gabapentin after a shingles outbreak? a. have you noticed any new blisters? b. are you experiencing pain anywhere? c. have you had a rash over the past month? d. are you experiencing problems falling asleep?

b. are you experiencing pain anywhere? outbreak that occurs with shingles lasts about 3-5wks. pt prescribed gabapentin 3 mos after an outbreak of shingles is being tx for postherpetic neuralgia. therefore the NP will inquire about pain, which is generally confined to the area in which dermatomal involvement occurred. s/s associated with postherpetic neuralgia include a sensation of burning, throbbing, sharp pain, aching at site of outbreak. gabapentin is not prescribed to prevent new lesions, rash or problems with falling asleep.

126. the NP diagnoses a pt with uncomplicated chlamydia. which tx is recommended? a. acyclovir 400mg PO TID x 7-10d b. azithromycin 1g PO in a single dose (directly observed tx preferred) c. benzathine penicillin G 2.4mU IM a single dose d. podofilox .5% gel or cream BID x 3 consecutive days

b. azithromycin 1 g PO single dose acyclovir is for genital herpes benzathine PCN G - primary syphilis (chancre), secondary syphilis of latent syphilis (< 1 yr) podofilox is for condylomata acuminata (genital warts) on vagina, external genitals, urethra, anus, penis, nasal mucosa, oropharynx or conjunctivae

6. clue cells are found in patients who have: a. leukemia b. bacterial vaginosis c. epidermal fungal infections d. pneumonia

b. bacterial vaginosis epithelial cells with adherent bacteria. will have unpleasant "fishy" smell more noticeable after sex. BV can produce cervicitis, risk factor for HIV acquisition and transmission. metronidazole is common for initial therapy. usually 500mg BID x 7 days. alcohol should be avoided

20. an NP is most likely to inform a menopausal patient starting HT that she may not need as much daily intake of which supplement? a. potassium b. calcium c. phosphorus d. iron

b. calcium this is due to increased levels of estrogen from HT

30. when prescribing the contraceptive patch or vaginal ring, the NP considers that: a. these are progestin-only products b. candidates include women who have difficulty remembering to take a daily pill c. there are significant drug interactions with both products d. contraceptive efficacy is less than with COC

b. candidates include women who have difficulty remembering to take a daily pill

7. patients are at increased risk for what fungal infection when using the sponge? a. toxic shock syndrome b. candidiasis c. skin irritation d. UTI

b. candidiasis toxic shock can occur from using sponge for too long, but this is bacterial in nature. none of them are fungal in nature although they can occur.

24. a middle-aged male pt being tx for BPH presents to the clinic for f/u appt. he reports that over past few days, he has experienced painful ejaculation and pain during urination. the pt's prostate is enlarged and boggy, and pt experiences discomfort during the exam. which tx would the NP rx? a. cefixime (suprax) 400mg single dose PO b. ceftriaxone (rocephin) 250mg single dose IM c. doxycycline 100mg PO BID x 10 days d. ciprofloxacin (cipro) 500mg PO BID x 10 days

b. ceftriaxone (rocephin) 250mg single dose IM s/s consistent with acute bacterial prostatitis. no indication that its related to STI; so the other meds listed are not indicated.

22. a 26yo female patient has been dx with gonorrhea. how should she be managed? a. ceftriaxone only b. ceftriaxone and azithromycin c. cefixime and azithromycin d. penicillin G

b. ceftriaxone and azithromycin

48. a 29-year-old female patient presents to the clinic with complaints of painful sexual intercourse and pain when urinating. the pt's temp is 100.1F. upon PE, the NP notes healthy appearance of internal & external genitalia. the pt has lower abdominal and CMT in response to palpation. what will the NP order to confirm dx and determine appropriate tx? a. blood cx b. cervical cx c. vaginal cx d. US imaging

b. cervical culture findings of endometritis. infection of uterus, can be caused by different organisms. most commonly associated with PID or invasive gynecological procedures in non-OBGYN population. to determine appropriate tx, NP will obtain cervical cx to identify type of bacteria causing infection. vaginal cx are easily contaminated and thus may mislead provider to rx wrong med. blood cx only obtained in setting of sepsis or bacteremia. US imaging useful if pt postpartum, can rule out endometritis caused by retained products of conception, uterine abscesses or infected hematoma.

115. which of the following is not representative of the presentation of secondary syphilis? a. generalized rash b. chancre c. arthralgia d. lymphadenopathy

b. chancre

26. A MALE PT PRESENTS WITH DYSURIA AND PENILE DISCHARGE. he states that his female partner has an STD, but he is not sure which one. which of these should be part of the differential? a. BV and trichomonas b. chlamydia and gonorrhea c. HIV and herpes d. syphilis and chlamydia

b. chlamydia and gonorrhea

80. treatment of vulvovaginitis caused by candida albicans includes: a. metronidazole gel b. clotrimazole cream c. hydrocortisone ointment d. clindamycin cream

b. clotrimazole cream

46. in advising a woman about menopause, the NP considers that: a. the average age at last menstrual period for north american woman is 47-48yrs b. hot flashes and night sweats occur in about 60% to 90% of women c. women with surgical menopause usually have milder symptoms d. FSH and LH levels are suppressed

b. hot flashes and night sweats occur in about 60% to 90% of women

111. which of the following terms describes the mechanism of action of imiquimod (Aldara)? a. keratolytic b. immune modulator c. cryogenic d. cytolytic

b. immune modulator

149. a 68yo female presents at primary care for complaints of whitish-looking papules in the vulvar region and nonpruritic maculopapular rash on soles of the feet. the pt is also experiencing alopecia and lymphadenopathy. RPR is 1:8. which dx is most likely? a. lichen planus b. condylomata lata c. molluscum contagiosum d. condylomata acuminata

b. condylomata lata s/s of secondary syphilis with infectious white papules in vulvar area that look like white warts (condylomata lata). may be skin-colored, white, pink or yellowish. additional data for secondary syphilis includes alopecia, lymphadenopathy, maculopapular rash on soles of feet and RPR of 1:8 (normal = 1:1). condylomata acuminata are genital warts caused by HPV, usually skin-colored with tan or bluish hues. molluscum cantagiosum is a poxvirus infection that can present anywhere on the skin in children, but adults, it is usually located in genital area and inner thighs of young adults. they have central umbilication with a white core. lichen planus causes red patches and plaques, but intact tissue may appear white or gray with a lacy pattern.

31. katie, a 17yo female, has a history of PMS. she comes to your clinic complaining of PMS s/s that she says have gotten worse over the past few months. you consider pharmacological management for katie's s/s. which of the following would NOT be effective in managing her concerns at this time? a. SSRI b. corticosteroids c. hormonal birth control d. vitamin E

b. corticosteroids may worsen the symptoms of mood swings.

25. you are teaching an NP class about a recurrent, viral STD that can either be associated with infection of the face or infection of the genitalia. there is no cure for this condition and it typically presents with painful lesions. what is the most definitive test for this condition? a. ELISA b. culture c. papanicolaou stain d. tzanck stain

b. culture this is the most definitive test for herpes, characterized by lesions on face or genitals, incurable, viral culture. can be detected in pap smear or tzanck stain, tests are not definitive. there is an ELISA for herpes but has low sensitivity for HSV2.

4. a 70-year-old male pt is dx with renal disease. which activity will help preserve kidney function? a. 20 mins of daily exercise b. daily increase in water consumption c. daily weights d. avoidance of red meat

b. daily increase in water consumption volume depletion decreases renal perfusion, as GFR improves so does renal function. water is critical for properly functioning kidneys.

82. which of the following is associated with male aging? a. increased levels of estrogen b. decreased sperm production c. increased production of semen d. decreased concentration of sperm

b. decreased sperm production result of decreased testosterone levels in aging men. secretions from seminal vesicles and prostate gland decrease, resulting in overall decrease in semen produced. low levels of estrogen are associated with low levels of testosterone in geriatric males.

43. Which of the following lab tests is sensitive for evaluating renal function? a. electrolyte panel b. eGFR c. creatinine d. BUN

b. eGFR GFR can be estimated from serum creatinine. eGFR uses serum creatinine along with age and values assigned for gender and race.

98. while reviewing lab results for a pt with suspected recreational drug use, the NP notes an elevated BUN. which additional test will the NP order to confirm or r/o renal failure in this pt? a. BUN:serum Cr ratio b. eGFR c. intravenous pyelogram d. MRI

b. eGFR when BUN is abnormally high, eGFR should be performed to r/o acute renal dysfunction or failure. if results are normal, then renal function is likely also normal. BUN:Cr determines level or stage of renal failure as well as dehydration or hypovolemia. an IVP is a test to evaluate the urine system and would be used to determine the cause of renal failure, such as tumor or scarring. MRI not required at this time.

10. a 22-year-old female has been diagnosed with PCOS. what is a common finding? a. low testosterone level b. elevated insulin levels c. positive pregnancy test d. elevated BP

b. elevated insulin levels multiple cysts on the ovaries. testosterone levels tend to be elevated since PCOS produces state of hyperandrogenemia.

56. a 48-year-old woman complains of increased frequency and severity of hot flashes. her last menses occurred 6 months ago. you would expect all of the following lab findings except: a. increased levels of LH b. elevated levels of testosterone c. reduced levels of estradiol d. reduced levels of progesterone

b. elevated levels of testosterone

2. sharyl, 25yo, says that she experiences pain with urinating, bleeding after intercourse and has vaginal discharge. which dx test would help confirm most likely diagnosis? a. ESR b. enzyme immunoassay c. western blot d. viral load

b. enzyme immunoassay chlamydia is mostly diagnosed based on pain during urination, bleeding after intercourse and vaginal discharge, often diagnosed through an enzyme immunoassay because it is quicker and less expensive than a chlamydia culture. western blot and viral load are for assessing HIV and not chlamydia.

53. which behavioral intervention is used for urge incontinence?

b. establishing a voiding schedule and gentle bladder stretching

6. which of these patients would most likely need to have her diaphragm refitted, given that all of these pts already have a diaphragm and want to continue to use one? a. a pt who experiences an allergic reaction b. gained approximately 25lbs c. lost ~8lbs d. contracted herpes

b. gained approximately 25lbs should refit diaphragm of gain or lose weight > 20lbs.

88. a pt presents with a complaint of dark-colored urine and fatigue. UA indicates hematoma, protein, RBC casts. what is the most likely explanation? a. UTI b. glomerulonephritis c. chronic renal insufficiency d. acute renal failure

b. glomerulonephritis

22. you are talking to your male patient about the importance of practicing safer sex. you mention that certain STDs can cause urethritis and, in rare cases, this can lead to epididymitis and male infertility. which of the following STDs is most likely to predispose a male patient to urethritis and infertility? a. genital warts b. gonorrhea c. molluscum contagiosum d. chancroid

b. gonorrhea in rare cases urethritis may lead to epididymitis and male infertility. the others do not cause this.

59. the greatest risk of transmitting HIV is during: a. late infection phase b. high viral load periods c. the time that detectable antibody is present d. the acute phase

b. high viral load periods

5. which of the following is LEAST likely undesirable effect to expect in a female pt using spermicides? a. incomplete dissolution of suppositories b. increased risk for candidiasis c. vaginal skin irritation d. unpleasant taste

b. increased risk for candidiasis can increase risk but is not significant compared to other undesirable effects. increases risk for UTIs. can also have vaginal or penile skin irritation, incomplete dissolution of suppositories and unpleasant taste.

136. a 22yo male who is otherwise healthy complains of scrotal pain. his pain has developed over the past 4 days. he is diagnosed with epididymitis. what is the most likely etiology? a. phimosis b. infection with chlamydia c. underlying hydrocele d. UTI

b. infection with chlamydia

23. a 55yo pt going through menopause comes to your office with complaints of vaginal dryness. she also says that she feels depressed and frequently experiences hot flashes. she then inquires about HT. as you go over her medical history, you know that which of the following conditions would NOT be a contraindication to HT? a. breast cancer b. infertility c. MI d. uterine cancer

b. infertility the others are all contraindications

24. what are the two most common methods for diagnosing molluscum contagiosum? a. serologic tests and CBC b. inspection and microsopic exam c. CBC and inspection d. microscopic exam and serologic tests

b. inspection and microsopic exam histologic exams may also be helpful. serologic tests are used for syphilis, CBC more useful in gonorrhea.

23. which of the these would NOT be considered a first-line tx for inflammation of glans penis caused by candida albicans? a. miconazole b. ketoconazole c. clotrimazole d. fluconazole

b. ketoconazole these pills should not be prescribed to tx any fungal infection d/t severe liver injury. miconazole, clotrimazole and fluconazole are antifungals that can be used

97. women with PID typically present with all of the following except: a. dysuria b. leukopenia c. cervical motion tenderness d. abdominal pain

b. leukopenia

110. an 81yo pt is brought to primary care clinic for dysuria, urinary frequency, nocturia and fever of 100.6F. pt is diagnosed with complicated cystitis. which abx will NP rx? a. fosfomycin (monurol) 3g x one dose b. levofloxacin (levaquin) 750mg once daily x 5-7d c. bactrim BID x 3 days d. nitrofurantoin BID x 5 days

b. levaquin 750mg once daily x 5-7 days elderly pt with complicated cystitis needs a fluoroquinolone. if high risk of multi-drug resistant organisms, macrobid can be used.

50. a 55yo male presents with swollen, painful right testicle and burning on urination. exam reveals edematous scrotum with tenderness and a positive Phren's sign. pt states he is heterosexual and has been in a monogamous relationship for the last 5 yrs. denies practicing anal intercourse. the NAAT is negative. which med will the NP prescribe? a. ceftriaxone 250mg IM b. levofloxacin 500mg PO x 10 days c. doxycycline 100mg PO BID x 10 days d. ceftriaxone 250mg IM and ofloxacin 300mg PO BID x 10 days

b. levofloxacin 500mg PO x 10 days NAAT negative for chlamydia and gonorrhea. in older male, acute epididymitis (generally gram negative E. coli), is the probable dx for most gram negative E. coli. levaquin or floxin QD for 10 days is preferred. ceftriaxone and doxycycline given for pts with epididymitis caused by chlamydia infection and gonorrhea. ofloxacin and ceftriaxone would be given for epididymitis usually caused by STDs and also organisms in anal sex.

70. a 72-year-old female patient has a complicated UTI. which regimen will the NP rx? a. medication tx for minimum of 3 days b. med tx for min of 7 days c. med tx for min of 10 days d. med tx for min of 14 days

b. meds for a min of 7 days complicated UTI in males, uncontrolled diabetics, pregnant or elderly women and pts who are immunocompromised require med tx for a minimum of 7 days. 3 days is not long enough for any bacteria, 10 or 14 days would be too long and could cause complications such as c-diff

53. the progestin component of HT is given to: a. counteract the negative lipid effects of estrogen b. minimize endometrial hyperplasia c. help with vaginal atrophy symptoms d. prolong ovarian activity

b. minimize endometrial hyperplasia

7. upon examining a 23yo pt, you identify pearly-white papules around her anogenital region. the papules are about 2mm wide and are round, smooth, and firm. the pt does not exhibit any other s/s. you would most strongly suspect which of the following conditions? a. genital warts b. molluscum contagiosum c. chancroid d. herpes

b. molluscum contagiosum small, round, white lesions on anogenital region, suggestive of molluscum contagiosum. genital warts are also growths, but these are typically soft, fleshy and keratinized. s/s chancroid include painful ulcers surrounded by any erythematous halo, if there are any s/s at all. HSV-1 generally indicated on face & lips, HSV-2 usually genitalia and painful ulcers.

6. a physically independent 75yo was dx with mild cognitive impairment 6mos ago. she resides in assisted living. she is in the clinic today for a scheduled visit. her adult daughter reports that about 2wks ago her mother had an episode of urinary incontinence, but no known episodes since. she is found to have asymptomatic bacteriuria. how should she be managed? a. repeat UA in 7d b. tx her today with one day dose of abx b. monitor her for s/s of UTI d. repeat UA in 4wks

b. monitor her for s/s of UTI

87. when discussing ED with a 70yo man, the NP considers that: a. it is a normal consequence of aging b. most cases have an underlying contributing cause c. although depression is common in older men, it is usually not correlated with increased rates of ED d. treatment options for younger men are seldom effective in older men

b. most cases have an underlying contributing cause

87. which of the following statements is true gonococcal infection? a. the risk of transmission from an infected woman to a male sexual partner is about 80% b. most men have asymptomatic infection c. the incubation period is about 2-3wks d. the organism rarely produces beta-lactamase

b. most men have asymptomatic infection

149. a 26yo pregnant female complains of dysuria. her UA is positive for nitrites, leukocytes esterase, bacteria. which statement below is true? a. the pt has a UTI and staph saprophyticus is the most likely pathogen b. the pt has a UTI and e. coli is the most likely pathogen c. this pt does not have a UTI d. this pt has a contaminated specimen

b. the pt has a UTI and e. coli is the most likely pathogen

38. a patient comes to your practice c/o intense itching in her vaginal area and yellowish-green discharge that "smells like old garbage". inspecting the pt's vulvovaginal region, you find erythema and red spots around the vagina. based on your observations, you order a wet prep test to confirm the pt's condition and expect which of the following results? a. clue cells b. motile trichomonads c. psuedo hyphae d. WBC

b. motile trichomonads characterized by vaginal erythema, "strawberry patches" on cervix or vagina, vaginal erythema, pruritus, and malodorous yellowish-green discharge. confirmed on wet prep. clue cells turn up if BV is cause, candidiasis produces psuedo hyphae. WBC are not diagnostic.

41. the most causative organisms of acute bacterial prostatitis in men < 35 yrs are: a. e. coli and klebsiella pneumoniae b. n gonorrhoeae and c. trachomatis c. psuedomonas and acinetobacter species d. enterococci

b. n gonorrhoeae and c. trachomatis

90. a female should be told to take her oral contraceptive pill at bedtime if she experiences: a. spotting b. nausea c. headaches d. weight gain

b. nausea

27. with depo-provera, the recommended length of use is usually: a. less than 1 year b. no more than 2 years c. as long as the woman desires this form of contraception d. as determined by her lipid response to the med

b. no more than 2 years

27. which cervical cancer screening guideline would the NP follow when designing a health promotion program for women > 65yrs? a. HPV testing every 3 years b. no screening for women with low risk c. pap smear with cytology every 5 years d. screen throughout life span for women with cervix

b. no screening for women with low risk women with no hx of high-grade precancerous lesions or those who have had a hysterectomy with removal of cervix, do not require screening. USPSTF recommends that women aged 21-29yrs to be screened q3yrs with cervical cytology alone and that women aged 30-65yrs be screened every 3 years with cervical cytology alone, every 5 years with high-risk HPV testing alone, or every 5 years with combination of hrHPV and cytology.

114. the NP is seeing a 35yo male pt with HIV for f/u of initial antiretroviral tx regimen of 600mg dolutegravir/50mg abacavir/300 mg lamivudine. his plasma viral load is 25000 copies/mL, a decrease from 35,000; CD4 is 400 cells up from 250 cells and assessment reveals pt is in no acute distress. all of the following interventions are indicated for this pt, except: a. order liver enzyme testing b. order a f/u PVL in 2 wks c. refer pt to a nutritionist for low-fat diet counseling d. order a f/u CD4 in 3 months

b. order a f/u PVL in 2 wks antiretroviral regimen of 600mg dolutegravir/50 mg abacavir/300mg lamivudine has a black box warning for severe hepatomegaly and HBV exacerbation, so monitoring liver enzymes is crucial. likewise, steatosis is common with hepatomegaly

116. the NP suspects possible kidney stones in an older adult male. an US is ordered with no evidence of kidney stones. the pt returns to the clinic with intermittent right-sided flank pain that is severe in intensity. the NP will: a. repeat the US b. order a noncontrast CT scan c. order CBC d. perform mcburney's test

b. order a noncontrast CT highest sensitivity/specificity for kidney stones. first test is renal US; if needed CT is ordered. CBC and UA will not assist in dx.

84. which of the following increases the risk of cryptorchidism? a. family hx of hearing problems b. premature birth c. maternal iron deficiency anemia d. constipation

b. premature birth

11. a patient diagnosed with abnormal uterine bleeding is undergoing further evaluation to determine the etiology. which condition is LEAST likely to be the cause of the pt's abnormal uterine bleeding? a. polycystic ovarian disease b. primary dysmenorrhea c. perimenopause d. immature hypothalamic-pituitary-ovarian axis

b. primary dysmenorrhea not known to cause abnormal uterine bleeding. secondary dysmenorrhea may result in heavy or irregular bleeding, but does not usually affect uterine bleeding. the other choices can cause abnormal uterine bleeding.

57. an elderly patient is diagnosed with UTI. which drug class may put this pt at risk for achilles tendon rupture? a. sulfonamides b. quinolones c. cephalosporins d. penicillins

b. quinolones

105. a sexually active male patient presents with epididymitis. what finding is likely? a. scrotal edema b. recent history of heavy physical exercise c. dysuria d. hematuria

b. recent history of heavy physical exercise

80. an adolescent has suspected varicocele. he has dull scrotal pain that is relieved by: a. standing b. recumbency c. having a bowel movement d. elevation of the testicle

b. recumbency

65. in postmenopausal women, a major benefit from the use of topical or local estrogen is: a. decreased rate of breast cancer b. reduced risk of recurrent UTIs c. reduced risk of type 2 diabetes d. increased levels of androgens

b. reduced risk of recurrent UTIs

92. a 6-month-old male has a palpable cystic mass in his scrotum. his mother states that sometimes the size of his scrotum varies. what should the NP do next? a. referral to urology as soon as possible b. referral to urology if this has not resolved in 6 months c. scrotal US should be ordered d. order a urine specimen to r/o infection

b. referral to urology if this has not resolved in 6 months

104. a 19yo female presents with temp of 100.8F and lower abd pain that began about 12 hours ago. she denies vaginal discharge. which choice below is the least likely cause of her s/s? a. ruptured ovarian cyst b. renal stone c. UTI d. appendicitis

b. renal stone

94. a 32-year-old injection drug user is tested for HIV and has a negative test result. in addition to recommending HIV-prevention counseling, you advise: a. repeat testing in 2 months b. repeat testing within 1 year c. repeat testing in 2 years d. that no repeat testing is needed unless new-onset symptoms develop

b. repeat testing within 1 year

130. a 32-year-old injection drug user is tested for HIV and has a negative test result. in addition to recommending HIV prevention counseling, you advise: a. repeat testing in 6 mos b. repeat testing within 1 yr c. repeat testing in 2 yrs d. no repeat testing is needed unless new onset symptoms develop

b. repeat testing within 1 yr

52. postmenopausal HT use usually results in: a. reduction in rate of cardiovascular disease b. an increase in the rate of RA c. a reduction in the frequency and severity of vasomotor symptoms d. a disturbance in sleep patterns

c. a reduction in the frequency and severity of vasomotor symptoms

9. you are seeing a 24-year-old female who states that she has been experiencing severe cramping pain during her last two menstrual cycles. which of the following conditions is the patient experiencing? a. PID b. secondary dysmenorrhea c. PMS d. primary dysmenorrhea

b. secondary dysmenorrhea most likely to occur in women 20+yrs, should undergo further evaluation to determine etiology of dysmenorrhea. PID and PMS can cause secondary dysmenorrhea however severe cramping during menses is not found in either condition. primary dysmenorrhea may occur in adolescents with excess prostaglandin secretion and would not be considered since she is not an adolescent.

39. a hypertensive middle-aged male american indian has recently been diagnosed with renal insufficiency. he has been on lisinopril (accupril) for many years. which the following lab values should be carefully monitored? a. hgb, hct and MCV b. serum creatinine and eGFR c. AST and ALT serum sodium, potassium and magnesium

b. serum creatinine and eGFR best tests for renal function is GFR, requires creatinine to be calculated (cockcroft-gault equation). american indians and alaska natives have highest rate of DM in any group in US.

73. an annual screening for c. trachomatis infection is recommended for: a. all sexually active women b. sexually active women ≤ 25yrs c. sexually active women who have had ≥ 2 partners in past 12 mos d. sexually active men ≤ 25yrs

b. sexually active women ≤ 25yrs

92. a 20-year-old student has an MMR titer that demonstrates unprotective titer for rubella. she is HIV positive. her CD4 cell count is unknown. which statement is true? a. she should receive this. the immunization is not alive. b. she is at risk for MMR but should not be immunized c. MMR is safe to give but she does not need this d. she should not receive the MMR immunization because she is low risk for the disease

b. she is at risk for MMR but should not be immunized

8. an inguinal hernia is palpated on a male pt by an examiner. which word below best describes what the hernia feels like when touched by the examiner? a. nodular b. silky c. firm d. bumpy

b. silky

17. what finding is most characteristic of shingles? a. pain, burning, and itching b. single dermatome affected c. presence of grouped vesicles d. presence of rash and crusting

b. single dermatome affected

21. which of these methods of natural family planning uses both the basal body temp graph and cervical mucus test? a. lactational amenorrhea method b. symptothermal method c. calendar method d. billings test

b. symptothermal method billings test is another name for cervical mucus test

34. which is the third most commonly reported STD of bacterial etiology in the US? a. chlamydia b. syphilis c. HPV d. gonorrhea

b. syphilis chlamydia is most common bacterial STD, gonorrhea is 2nd most common. HPV is most commonly reported viral STD in the US.

12. a patient made an appointment with your clinic to be evaluated for a possible STI. when the pt misses the appt, you follow-up and learn that the pt has been admitted to the hospital for a case of meningitis. which of the following STDs would you most likely expect the pt to have? a. gonorrhea b. syphilis c. chancroid d. chlamydia

b. syphilis syphilitic aseptic meningitis is a type of meningitis that can result from tertiary syphilis. cardiac insufficiency, aortic aneurysm, hemiplegia + other signs of CNS involvement may also present. gonorrhea may also cause cardiovascular or CNS issues in rare cases, but meningitis from gonorrhea is very rare. untreated chlamydia will not produce meningitis, but may cause PID, reiter syndrome or infertility in women. complications of chancroid are typically limited to genitals - including phimosis, balanoposthitis and superinfection of ulcers

52. the NP is evaluating a 72 year old male pt newly dx with BPH. the pt states that he is experiencing difficulty starting a urine stream, and urinary retention is noted upon exam. the pt is taking SL NTG PRN for angina. which med will the NP prescribe? a. tadalafil (cialis) 5mg PO once daily b. tamsulosin (flomax) 0.4mg PO once daily c. oxybutynin (ditropan) 5mg PO once daily d. vardenafil (levitra) 10mg PO once daily

b. tamsulosin (flomax) 0.4mg PO once daily alpha-adrenergic antagonist, it is first-line for BPH. tadafil and verdenafil are PDE5 inhibitors and tx erectile dysfunction. oxybutinin is an anticholinergic for urinary incontinence.

117. which med can be rx to a pt to manage both HTN and BPH? a. silodosin (rapaflo) b. terazosin (hytrin) c. finasteride (proscar) d. phenoxybenzamine (dibenzyline)

b. terazosin (hytrin) quinazoline used to tx BPH and HTN. silodosin manages s/s BPH. finasteride tx symptomatic BPH. phenoxybenzamine is nonselective alpha-adrenergic blocker used to improve urinary flow, but since it is nonselective, high incidence of adverse effects.

25. the HCP was stuck with a needle from a patient suspected to be infected with HIV. a rapid HIV test was performed on the pt and found to be positive. this means that:

b. the HIV status of the patient requires further testing

48. a 53-year-old woman who is taking hormone therapy with conjugated estrogen, .45mg/d with medroxyprogesterone acetate (MPA), 1.5mg, has bothersome atrophic vaginitis s/s. you advise that: a. her oral estrogen dose should be increased b. the addition of topical estrogen can be helpful c. the MPA component should be discontinued d. baking soda douche should be tried

b. the addition of topical estrogen can be helpful

136. the following statements are true regarding herpes zoster, except: a. it is due to a reactivation of latent varicella virus b. typical lesions are bullae c. it is usually more severe in immunocompromised individuals d. infection of trigeminal nerve ophthalmic branch can cause corneal blindness

b. the lesions are typically bullae shingles occurs secondary to reactivation of the varicella-zoster virus. this infection can be more severe in immunocompromised pts because of their inability to fight infection. shingles rash starts as small blisters on a red base, with new blisters continuing to form for 3-5 days. rash is very painful and commonly appears in clusters, following one dermatome on one side of the body. when trigeminal nerve is involved, there is an increased risk of corneal blindness.

14. a patient who is diagnosed today with diabetes has an elevated urinary albumin to creatinine ratio. what can be concluded about this finding? a. the patient might have been diabetic for a long time before diagnosis b. the patient should have repeat test in 3-6 months c. there is renal damage d. the patient has diabetic neuropathy

b. the patient should have repeat test in 3-6 months

35. which of the following is the most accurate statement regarding the biological effects during menopause? a. the cervical mucus thickens, the vagina becomes redder in tone b. the vagina experiences decreased stimulation as the epithelium atrophies c. vaginal pH levels and vaginal secretions decrease d. the decreased pH in the vaginal lactic acid increases susceptibility to urinary tract pathogens

b. the vagina experiences decreased stimulation as the epithelium atrophies other findings include thinning cervical mucus & increased pH levels, vaginal lactic acid which may increase susceptibility to vaginitis and UTIs.

33. when considering the use of self-administered depo-subQ the NP acknowledges all of the following except: a. a self-injection is given every 3 months b. there is no risk of bone loss with long-term use of this DMPA formulation c. can be started immediately postpartum d. contains a smaller amount of active ingredient than the IM formulation

b. there is no risk of bone loss with long-term use of this DMPA formulation

23. a female pt and her male partner are diagnosed with trichomonas. she has complaints of vulval itching and discharge. he is asymptomatic. how should they be tx? a. she should receive metronidazole; he does not need tx b. they both should receive metronidazole c. she should be tx with ceftriaxone, he should receive ciprofloxacin d. they both should be tx with azithromycin and doxycycline

b. they both should receive metronidazole

14. the dx of osteoporosis involves the DEXA test to measure bone mineral density in four different parts of the skeletal system. the practitioner knows that all of the following types of bone tissue are typically measured by a DEXA EXCEPT: a. hip b. tibia c. spine d. ankle

b. tibia hip, spine, wrist and ankle are used in BMD scans to predict osteoporotic fractures

12. a 79yo male presents to your practice stating,"I'm back today because I forgot to tell you last week that I'm having trouble with my urine. I go several times a night, feel like i have to go all of the time, and when I pee, it starts, stops and even dribbles sometimes." at his physical exam 1 wk ago, his complete blood count, UA and VS including temp were normal. his PSA was noted to be 6.0ng/mL. the NP considers a referral for which of the the following tests at this time? a. MRI b. transrectal US c. CT of pelvis d. x-ray of kidney, ureters and bladder

b. transrectal US The pt's symptoms all indicate prostate cancer or BPH; pt should be referred for transrectal US to aid in indentification or solid nodules to utilize needle biopsy. MRI and CT are more useful in the staging of prostate cancer after confirmation of disease. x-ray is not helpful.

81. a 24-year-old woman presents with a 1wk hx of thin, green-yellow vaginal discharge with perivaginal irritation. PE findings include vaginal erythema with petechial hemorrhages on the cervix, numerous WBCs, and motile organisms on microscopic examination. these findings most likely represent: a. motile sperm with irritative vaginitis b. trichomoniasis c. BV d. condyloma acuminatum

b. trichomoniasis

3. noninfectious epididymitis is common in: a. soccer players b. truck drivers c. marathon runners d. men who wear boxer-style underwear

b. truck drivers occurs with reflux of urine into epididymis from ejaculatory ducts and vas deferens. this can cause ductal obstruction and acute inflammation without infection. this can occur if men spend a lot of time sitting. other risk factors are vigorous exercise that involve heavy lifting or upper body workouts; especially situps or abdominal crunches.

41. you see a 34-year-old woman who reports having unprotected sexual intercourse 5 days ago and requests emergency contraception, she has a current history of mucopurulent cervicitis. the most acceptable and effective option in this clinical scenario is: a. progestin-only emergency contraception b. ulipristal c. cu-IUD d. none, as 5 days is too long to wait for emergency contraception to be effective

b. ulipristal

16. a sexually active adolescent male reports that he has dysuria. how should this be managed? a. collect a urethral culture and schedule a return appt b. urethral/urine cx should be collected and screening done for STDs c. prescribe azithromycin and ceftriaxone d. prescribe azithromycin

b. urethral/urine cx should be collected and screening done for STDs

66. which assessment finding is expected in a 3yo male with penopubic epispadias? a. fused pelvic bones b. urine leakage with stress c. narrow penis that curves down d. urinary meatus found along shaft of penis

b. urine leakage with stress in penopubic epispadias, the urethral meatus is found near the pubic bone. position of meatus helps predict how well the bladder stores urine. when bladder sphincter is shaped more like horseshoe than a ring, it does not close all the way. because of this, urine leaks out. most boys with penopubic epispadias and about two of three with penile epispadias leak urine with stress. in most cases, the bones of pelvis do not come together in the front. epispadias tends to make penis broad, short and curved up. the penile epispadias urinary meatus is found along the shaft. in penopubic epispadias it is found on or near the pubic bone.

8. a 76-year-old male presents with urethral irritation after voiding. if STDs and UTI are ruled out, what is another etiology? a. acute bacterial prostatitis b. chronic prostatitis c. epididymitis d. asymptomatic bacteriuria

b.chronic

22. as you prescribe COC containing the progestin drospirenone (Loryna, ocella, vestura, yasmin, yaz) you offer the following advice: a. "always take this pill on a full stomach" b. "you should not take acetaminophen when using this birth control pill." c. "avoid using potassium-containing salt substitutes" d. "you will likely notice that PMS s/s might become worse"

c. "avoid using potassium-containing salt substitutes"

11. which of the following is the most appropriate response to a 27yo who is taking phenytoin (dilantin) for the tx of a seizure disorder and is requesting hormonal contraception? a. "a barrier method would be the preferable choice" b. "COC is the best option" c. "depo-provera use will likely not interact with your seizure medication" d. "Cu-IUD or LNG-IUD use is contraindicated"

c. "depo-provera use will likely not interact with your seizure medication"

93. diagnostic testing of a person with primary HSV-2 infection would likely show: a. negative virological and serological test results b. a negative virological test result and positive serological test result c. a positive virological test result and negative serological test result d. positive virological and serological test results

c. a positive virological test result and negative serological test result

31. jennifer, an 18yo female, arrives at clinic seeking emergency contraception. she claims she was engaging in intercourse with boyfriend last night and condom broke during the act. you believe that levonorgestrel would best address her concerns. which of the following statements would be most accurate? a. "you will likely need a prescription to get this drug" b. "this pill works by terminating an implanted fertilized egg" c. "you may experience menstrual irregularities during your next cycle" d. "the drug should work up to 4 days after intercourse"

c. "you may experience menstrual irregularities during your next cycle"

109. which HPV types are most often associated with cervical and anogenital cancer? a. 1, 2 and 3 b. 6 and 11 c. 16 and 18 d. 22 and 24

c. 16 and 18

113. how long after contact do clinical manifestations of syphilis typically occur? a. < 1 wk b. 1-3wks c. 2-4wks d. 4-6 wks

c. 2-4wks

66. how long after contact does the onset of clinical manifestations of syphilis typically occur? a. less than 1 wk b. 1-3wks c. 2-4wks d. 4-6wks

c. 2-4wks

53. the NP is evaluating pts who are at high risk for complication due to UTIs. which of the following pts does not belong in this category? a. 38yo diabetic paytient with an A1C 7.5% b. pt with RA who is being tx with methotrexate and low-dose steroids c. a 21yo pt who has a hx of IBS d. a pregnant pt

c. 21yo with hx of IBS risk factors for UTI are female, pregnancy, spermicide use during past year, mother with hx of UTIs, new sex partner during past year, urinary incontinence, cystocele

43. a woman who has used any form of oral emergency contraception should be advised that if she does not have a normal menstrual period within _____ wks, a pregnancy test should be obtained a. 1-2 b. 2-3 c. 3-4 d. 4-5

c. 3-4

24. what proportion of osteoporotic bone loss stems from hypoestrogenic states? a. one-fourth b. 1/2 c. 3/4 d. all bone loss

c. 3/4 occurs rather than aging process itself

127. HIV screening is recommended for all of the following except: a. 17yo female with no risk factors for HIV but who requests testing b. 47yo female seeking tx for syphilis c. 36yo female who was diagnosed with HIV infection 4yrs d. a 24yo female who is an injection drug user

c. 36yo female who was diagnosed with HIV infection 4yrs

50. a patient who is scheduled for pelvic exam with PAP smear should be advised to avoid douching, sexual intercourse, and tampon use before her exam. for how long should she be advised to avoid these activities for optimal evaluation? a. 1 wk b. 36hrs c. 48hrs d. 24hrs

c. 48hrs

112. about _____ of patients with genital warts have spontaneous regression of the lesions? a. 10% b. 25% c. 50% d. 75%

c. 50%

91. HIV testing is recommended for all of the following except: a. 17yo male with no risk factors for HIV but who requests testing b. 47yo male seeking tx for syphilis c. 75yo who reports no HIV associated risk factors d. 24yo male who is an injection drug user

c. 75yo who reports no HIV associated risk factors

7. a 72-year-old male pt has early renal insufficiency. what lab test best assesses his kidney function? a. serum creatinine b. presence of proteinuria c. GFR d. quantity of urine produced in 24 hours

c. GFR

44. a middle-aged man who is homeless reports to the local public health clinic c/o painless and shallow ulcer on penile shaft for past 2 weeks. he is sexually active and had unprotected intercourse with two male partners over past few months. the pt is tested for HIV, syphilis, gonorrhea and HBV and HSV1 and 2. the syphilis and HIV tests are both positive. the gonorrhea, HBV and herpes tests are negative. the NP is aware of the nationally notifiable infectious conditions. which of the following is true regarding the reporting of any of these STIs? a. HCPs must obtain the pt's permission before reporting the +HIV and syphilis test results to the local public health department. b. the NP should obtain the pt's and sexual partner's permission before reporting the +test to local health department c. HCPs are mandated by law to report certain types of diseases to the local health department even if the pt does not give permission d. the NP should consult with the supervising physician about this issue.

c. HCPs are mandated by law to report certain types of diseases to the local health department even if the pt does not give permission STDs, HIV/AIDS, gonorrhea, syphilis must be reported to the local health department even if the pt does not give permission. partner tracing and notification are done by the local health department. CDC mandates reporting of TB, diphtheria, HAV, HBV, HCV, measles, mumps, pertussis, lyme disease, rocky mountain spotted fever.

24. appropriate treatment of acute epididymitis for a 32yo male who has sex with men is: a. IM ceftriaxone b. PO azithromycin c. PO levofloxacin d. IV TMP-SMX

c. PO levofloxacin

5. which of the following women is the best candidate for progestin only pill (POP) use? a. an 18yo woman who frequently forgets to take prescribed medications b. a 28yo woman with multiple sexual partners c. a 32yo woman with adequately controlled hypertension d. a 26yo woman who wants to use the pill to help "regulate" her menstrual cycle

c. a 32yo woman with adequately controlled hypertension

7. which patient has the most substantial signs for developing osteoporosis? a. a 5'2" african american female weighing 115lbs b. a 5'9" caucasian male weighing 155lbs c. a 5'4" asian female weighing 90lbs d. a 5'10" latin female weighing 200lbs

c. a 5'4" asian female weighing 90lbs females of white or asian ethnicity with small frames are high risk for developing osteoporosis.

98. which of the following would you expect to find on a wet-mount slide of a patient diagnosed with BV? a. tzanck cells b. a large number of leukocytes and squamous epithelial cells c. a large number of squamous epithelial cells whose surfaces and edges are coated with large numbers of bacteria along with a few leukocytes d. epithelial cells and a small amount of blood

c. a large number of squamous epithelial cells whose surfaces and edges are coated with large numbers of bacteria along with a few leukocytes dx of BV includes 3/4 amsel criteria, (1) white, thick adherent discharge, (2) pH > 4.5, (3) positive whiff test, (4) clue cells > 20 % on a wet mount (epithelial cells dotted with large # of bacteria that obscure cell borders)

30. which of the following is most accurate information for a 40-year-old man with cystitis? a. this is a common condition of men of this age b. a gram-positive organism is the likely causative pathogen c. a urological evaluation should be considered d. pyuria is rarely found

c. a urological evaluation should be considered

13. a 22-year-old male pt has s/s of burning with urination. which assessment below is least important at this time? a. body temp b. full genital exam c. abdominal exam d. assessment of CVA tenderness

c. abdominal exam

59. a 32-year-old sexually active female is diagnosed with BV based on findings of copious milk-like vaginal discharge. speculum exam reveals gray/white discharge on the vaginal walls and fishlike odor based on the whiff test. the patient will be treated with a 5-day course of metronidazole vaginal gel, one application at bedtime, and advised to do which of the following? a. douche after intercourse b. ensure that sexual partner is seen & treated c. abstain from sexual intercourse during treatment d. use a condom during intercourse

c. abstain from sexual intercourse during treatment the pt would be advised to abstain until tx is complete. douching can alter normal vaginal pH and be root cause of overgrowth of bacteria. BV is not an STI so sexual partners do not need treatment.

9. which of the following statements is true concerning vaginal diaphragm use? a. when the device is in place, the woman is aware that the diaphragm fits snugly against the vaginal walls b. this is a suitable form of contraception for women with recurrent UTI c. after the device is inserted, the cervix should be smoothly covered d. the device should be removed within 2 hours of coitus to minimize the risk of infection

c. after the device is inserted, the cervix should be smoothly covered

50. an older adult pt has been diagnosed with shingles on the right lateral aspect of her trunk. it initially appeared yesterday. it is very painful. how should she be managed? a. tx with topical lidocaine patch only b. an oral antiviral agent and NSAIDs c. an oral antiviral agent and pain medication d. an oral antiviral agent, pain meds and oral steroids

c. an oral antiviral agent and pain medication

36. an exam of a patient dx with an advanced case of lymphogranuloma venereum reveals buboes in the vulvovaginal area. which tx is recommended to prevent ulcerations? a. drying agents b. electrocautery c. aspiration d. cryotherapy

c. aspiration may relieve pain & prevent formation of ulcers. drying agents are recommended for symptomatic tx of HSV to keep sores from weeping. electrocautery as well as cryotherapy, laser therapy and excision are recommended for removal of genital warts. cryoanesthesia with liquid nitrogen is preferred for removing lesions produced by molluscum contagiosum.

47. patients with urge incontinence often report urine loss: a. with exercise b. at night c. associated with strong sensation of needing to void d. as dribbling after voiding

c. associated with strong sensation of needing to void

117. syphilis is most contagious during which of the following? a. before onset of s/s b. at the primary stage c. at secondary stage d. at tertiary stage

c. at secondary stage

86. which of the following agents is most active against c. trachomatis? a. amoxicillin b. metronidazole c. azithromycin d. ceftriaxone

c. azithromycin

139. the clinical syndrome resulting from replacement of normal vaginal flora with anaerobic bacteria: a. herpes simplex b. PID c. bacterial vaginosis d. candidal vaginitis

c. bacterial vaginosis

15. all of the following are definitive reasons to re-examine and possibly replace a diaphragm as a contraceptive method EXCEPT: a. use of oil-based lubricants b. wear & tear c. being diagnosed with vulvovaginitis d. gaining weight > 20lbs

c. being diagnosed with vulvovaginitis although use of diaphragm increases the risk of vulvovaginitis, contraction of disease is not an absolute reason to re-examine or replace it.

38. what is the typical window of time in which an infected person most commonly converts from HIV negative to HIV positive? a. between 9-12 months b. between 5 days and 6 weeks c. between 3 weeks and 6 months d. between 12 to 24 months

c. between 3 weeks and 6 months seroconversion is process in which HIV antibodies develop & become more detectable, generally taking 3-6 months after initial infection. during this process individual may have flu-like s/s. it can occur sooner than 3wks, but it does not occur in 5 days. period of 9-12 months is abnormal.

12. all of following are typical findings for a patient with chancroid except: a. multiple lesions b. spontaneous rupture of affected nodes c. blood-tinged penile discharge d. dense, matted lymphadenopathy on the ipsilateral side of the lesion

c. blood-tinged penile discharge

104. which of the following is a treatment option for a 28-year-old woman with PID who has no hx of medication allergy and has undergone a bilateral tubal ligation? a. ofloxacin with metronidazole b. gentamicin with cefpodoxime c. ceftriaxone with doxycycline d. clindamycin with azithromycin

c. ceftriaxone with doxycycline

39. which of the following drugs is NOT a phosphodiesterase inhibitor? a. sildenafil b. vardenafil c. chlorophyllin d. tadalafil

c. chlorophyllin this is an internal deodorant used to improve breath and body fluid odors.

42. when choosing an antimicrobial agent for the tx of chronic bacterial prostatitis the NP considers that: a. gram-positive organisms are the most likely cause of infection b. cephalosporins are the first-line choice of therapy c. choosing an abx with gram-negative coverage is critical d. length of antimicrobial therapy is typically 5 days

c. choosing an abx with gram-negative coverage is critical

148. the following are absolute contraindications for use of oral contraceptives except: a. sexually active pt with amenorrhea b. hx of emboli that resolved with heparin therapy 15 years ago c. cigarette smoking at the age of 30 years d. hepatitis C infection

c. cigarette smoking at the age of 30 years relative not absolute contraindication, however smoking aged 35 or older is absolute.

76. a 79-year-old female pt with urinary frequency is found to have a UTI. what med could produce arrhythmias for her? a. doxycycline b. amoxicillin c. ciprofloxacin d. macrodantin

c. ciprofloxacin

99. a 4-year-old female is brought in to the clinic by her mother, who reports that she is constantly scratching "her private part". the pt states that it itches. on exam, the vagina is red and irritated. how should the NP proceed? a. call child protection for suspected sexual abuse b. prescribe a cortisone cream c. collect a vaginal swab of the external vagina for microscopic evaluation d. prescribe topical antifungal

c. collect a vaginal swab of the external vagina for microscopic evaluation

40. a 16-year-old female is diagnosed with primary dysmenorrhea. she has taken OTC ibuprofen in 800mg increments every 8 hours during menses for the past 3 months, with minimal relief of symptoms. which intervention will provide greatest relief of dysmenorrhea symptoms? a. 30mins of regular exercise daily b. daily multivitamin with B12 supplementation c. combined oral contraceptives d. flurbiprofen during menses

c. combined oral contraceptives

1. a patient using calendar method for contraception has determined that her shortest cycle lasts 28 days, her longest cycle lasts 32 days and that her cycle starts on day 5 of the month. during which range of days of the month should she abstain from intercourse to best avoid pregnancy? a. day 14-47 b. day 21-27 c. day 15-28 d. day 19-28

c. day 15-28 when using calendar method, subtract 18 days from shortest cycle and 11 days from longest cycle. both totals should be added to day of month her cycle begins, counting that day as part of totals, thus determining fertile window

22. you are performing a health check-up on a 47yo female. she tells you that she's recently been waking up at night to go to the bathroom. she mentions whenever she coughs or sneezes, she pees a little. she also says that she had to buy a bra with a smaller cup size the month prior and was surprised that her size had changed. based on the pt's complaints, which of the following skin findings would you LEAST expect to see? a. thinning of the vagina b. decreased skin elasticity c. decreased melanin synthesis d. dry skin

c. decreased melanin synthesis menopause can present with these symptoms but this can cause an increase not decrease in melanin synthesis, which becomes less regulated by lack of estrogen, resulting in brown spots. other common signs include thinning of vagina, dry skin and decreased skin elasticity.

87. a 25-year-old male patient with subacute bacterial epididymitis should be treated initially with: a. ice and scrotal support b. NSAIDs only c. doxycycline d. an oral quinolone

c. doxycycline

70. syphilis is most contagious at which of the following times? a. before onset of s/s b. during primary stage c. during secondary stage d. during tertiary stage

c. during secondary stage

9. a 17-year-old presents with complaints of dysmenorrhea. which finding below suggests that this is a secondary dysmenorrhea? a. nausea with menses b. unpredictable menses c. dysmenorrhea is not limited to menses d. normal pelvic exam

c. dysmenorrhea is not limited to menses primary dysmenorrhea is attributed to prolonged uterine contractions that cause ischemia in myometrium. some females with secondary dysmenorrhea may have nl pelvic exam.

101. expected lab findings for a woman with PID include all of the following except: a. elevated ESR b. elevated CRP c. elevated CrCl d. leukocytosis

c. elevated CrCl

13. although progestin-only contraceptive pills are not as effective in suppressing unscheduled bleeding, these are a more viable option for pts with certain conditions that are exacerbated by estrogen. which of these conditions does NOT usually warrant the need for progestin-only pills? a. migraine headaches b. hypertension c. endometriosis d. obesity

c. endometriosis

26. the most likely causative organism in community-acquired UTI in women during the reproductive years is: a. klebsiella species b. proteus mirabilis c. escherichia coli d. staphylococcus saprophyticus

c. escherichia coli

19. george, a 46yo male, says that he feels a burning sensation every time he urinates. he also states that he urinates more frequently than usual, and that he often has to get up in the middle of the night to do so. which of the following methods would yield the most sensitive but not specific lab results for the pt's suspected condition? a. visual exam of the pt's urine sample b. presence of nitrate by dipstick c. esterase detection by dipstick d. blood testing by dipstick

c. esterase detection by dipstick would confirm lower UTI with these symptoms. dipstick UA shows infection and considered to be very sensitive but not specific. visual exam is neither highly sensitive or specific, since appearance can be strange due to medication or other factors. UA can detect nitrates but is not sensitive enough for bacteriuria. blood test is not required unless suspecting kidney damage or blood disorders

141. a pt has been tx for HIV infection with antiretroviral therapy. he is stable. how often should CD4 counts be repeated? a. every 1-2wks b. every 1-2 months c. every 3-6months d. annually

c. every 3-6months

18. undesirable side effects of oral contraceptives such as depression, fatigue and decreased libido usually primarily result from: a. excessive estrogen b. estrogen deficiency c. excessive progesterone d. progesterone deficiency

c. excessive progesterone through androgenic properties. undesirable effects related to estrogen are nausea, HTN, increased clotting. both hormones can cause breast tenderness, headaches, HTN.

114. which of the following is not representative of the presentation of primary syphilis? a. painless ulcer b. localized lymphadenopathy c. flu-like symptoms d. spontaneously healing lesion

c. flu-like symptoms

67. which of the following is not representative of the presentation of primary syphilis? a. a painless ulcer b. localized lymphadenopathy c. flu-like symptoms d. a spontaneously-healing lesion

c. flu-like symptoms

9. a patient starting the contraceptive patch places her first patch on the first sunday of the month. assuming she maintains the recommended schedule, she would replace this patch with a new one on each of the following sundays except: a. second sunday b. third c. fourth d. fifth

c. fourth proper use of patch includes changing patch out once per week on same day of week, on fourth, "change day" patch is removed and not replaced until 1 week later. fifth starts a few cycle.

32. a patient presents with a complaint of dark colored urine and fatigue. UA indicates hematuria, protein and RBC casts. what is the most likely explanation? a. acute renal failure b. chronic renal insufficiency c. glomerulonephritis d. UTI

c. glomerulonephritis

37. a pt presents to the clinic with purulent white discharge and six maculopapular lesions. he also complains of pain in his testicles and frequent nausea. which of these is most likely diagnosis? a. herpes b. chancroid c. gonorrhea d. syphilis

c. gonorrhea pustular lesions located peripherally to site of infection are most indicative of gonorrhea. gonorrhea does not always have lesions, however when present, often painful and may be between 4-50 lesions. although herpes may also present with ulcers on genitals, ulcers are more often pruritic. additionally, purulent penile discharge is not commonly a sign of herpes. chancroid often manifests with painful ulcers that are surrounded by erythematous halo. syphilis also commonly presents with flu-like s/s and painless chancre on genitalia. however, purulent penile discharge not common.

31. neisseria gonorrhoeae are best described as: a. gram positive cocci b. gram positive rods c. gram negative diplococci d. gram negative bacilli

c. gram negative diplococci

1. a patient has urinary burning, frequency, and urgency. her UA is positive for leukocytes and negative for nitrites. a likely explanation for this is that the pt: a. has a kidney stone b. could be pregnant c. has an STD d. has DM

c. has an STD classic s/s of UTI but this is not a possible answer.

96. an adolescent male has had a sudden onset of severe scrotal pain following a kick in the groin earlier in the day during a soccer game. how should this be managed? a. a urine specimen should be collected b. a scrotal US should be ordered c. he should be referred to the ED d. he should be examined carefully for a low riding testicle

c. he should be referred to the ED

79. what is the american cancer society's recommendation for prostate screening in a 70-year-old male? a. he should be screened annually with PSA only b. he should be screened annually with PSA and DRE c. he should be screened until he has a life expectancy of < 10 yrs d. screening can stop at age 75yrs

c. he should be screened until he has a life expectancy of < 10 yrs

131. a 35-year-old female tests positive for HIV. it has been 18 months since her last HIV test and she states that she has had four partners since that time. you advise that: a. it is her legal responsibility to notify all partners at risk of HIV within 48hrs b. the clinic will contact each partner and inform them of her positive test result c. healthcare agencies are available to assist in notifying partners without disclosing her identity d. all partners for the past 5 years should be notified of the test result

c. healthcare agencies are available to assist in notifying partners without disclosing her identity

95. a 35-year-old male who has sex with men tests positive for HIV. it has been 18 months since his last HIV test and he states that he has had four partner since that time. you advise that: a. it is his legal responsibility to notify all partners at risk of HIV within 48hours b. the clinic will contact each partner and inform them of his positive test result c. healthcare agencies are available to assist in notifying partners without disclosing his identity d. all partners for the past 5 years should be notified of the test result

c. healthcare agencies are available to assist in notifying partners without disclosing his identity

15. which of the following hemograms would be expected for a 75-year-old woman with chronic renal failure and related anemia? a. hg = 9.7, MCV = 69, reticulocytes 0.8% b. hg = 10.2, MCV = 104, reticulocytes 1.2% c. hg = 9.4, MCV = 83, reticulocytes = 0.7% d. hg = 10.4, MCV = 94, reticulocytes = 2.6%

c. hg = 9.4, MCV = 83, reticulocytes = 0.7%

78. physical examination of a 19yo woman with a 3d hx of vaginal itch reveals moderate perineal excoriation, vaginal erythema, and white, clumping discharge. expected microscopic exam findings include: a. pH > 6 b. increased number of lactobacilli c. hyphae d. an abundance of WBCs

c. hyphae

4. a nurse practitioner identifies filamentous structures and many uniform, oval-shaped structures during a microscopic exam of vaginal discharge. these are probably: a. bacteria b. sperm c. hyphae d. candida albicans

c. hyphae the mechanism taht allows fungal growth. yeast may look uniform oval-shaped structures. these can range in size but are usually unicellular. candida albicans is common

73. the term that describes the urethral opening on the ventral surface of the penis is: a. hydrocele b. inguinal hernia c. hypospadias d. cryptorchidism

c. hypospadias

3. common causes of chronic renal failure include all of the following except: a. T2DM b. recurrent pyelonephritis c. hypotension d. polycystic kidney disease

c. hypotension

18. a 50yo male reports that he has a sensation of scrotal heaviness. he reports that the sensation is worse at the end of the day. he denies pain. what is the likely etiology of these s/s? a. hydrocele b. epididymitis c. inguinal hernia d. strangulated hernia

c. inguinal hernia

90. for patients with ED who fail therapy with a PDE-5 inhibitor, alternative approaches include all of the following except: a. alprostadil injection into the penis b. mechanical vacuum devices c. insertion of NTG pellet into urethra d. implantation of prosthetic device

c. insertion of NTG pellet into urethra

132. while examining the genitalia of a male pt, the NP observes distended scrotal veins. the NP will: a. examine the pt in a supine position b. examine pt in a sitting position c. instruct the pt to perform a valsalva maneuver d. assess the testicular size with an orchidometer

c. instruct the pt to perform the valsalva maneuver usually while standing. will cause distention of veins of pampiniform plexus and assist in evaluation for possible scrotal abnormalities such as varicocele. orchidometer is used to evaluate size of testes, not evaluate for scrotal abnormalities.

54. which behavioral intervention is used for stress incontinence?

c. kegal exercises in and pelvic floor rehab with biofeedback

48. symptoms in chronic bacterial prostatitis often include: a. fever b. GI upset c. low back pain d. penile discharge

c. low back pain

8. a patient has a UTI. what findings on urine dipstick best describe a typical UTI? a. positive leukocytes b. positive nitrates c. positive leukocytes, positive nitrites d. positive nitrates and hematuria

c. positive leukocytes, positive nitrites

33. a patient asks the NP's advice about an herb to help with her hot flashes. the NP knows these: a. help prevent osteoporosis b. substances have a mild estrogenic effect and will halt hot flashes c. may be contraindicated in patients with history of breast cancer d. are safe to use in all patients

c. may be contraindicated in patients with history of breast cancer probably black cohosh, actaea racemosa. it is a phytoestrogen. which means it provides estrogen from a plant source. if CI in pt, then it does not a matter where it comes from. estrogen can affect breasts. has not been shown in studies to help with hot flashes.

53. which test will the NP order for a pt with s/s of chronic prostatitis? a. UA b. PSA c. meares-stamey 2-glass test d. gram stain of urethral swab

c. meares-stamey 2-glass test used to assess for inflammation & presence of bacteria in lower urinary tract, findings that are associated with s/s of chronic prostatitis. PSA is not a specific test for inflammation or presence of bacteria associated with chronic prostatitis. gram stain of urethral swab can be used to evaluate pt for STI associated with acute bacterial prostatitis.

56. a 26-year-old HIV positive pt presents with photophobia and temp of 103.2ºF. he complains of a headache. on exam, he is unable to demonstrate full extension of the knee when his hip is flexed. which choice is the most likely diagnosis? a. septic arthritis b. septic bursitis c. meningitis d. pneumocystis infection

c. meningitis

57. which of the following is likely to be noted with short-term (less than 1-2yrs) HT use in a postmenopausal woman? a. reduction in dementia risk b. significant increase in breast cancer risk c. minimized hot flashes d. increased cardiovascular risk

c. minimized hot flashes

41. an 84yo female pt is a resident in an assisted living facility. she has early dementia. she walks daily and has had urinary incontinence for years. she has loss of urine with coughing, sneezing and if she unable to get to the bathroom quickly enough. her urinary incontinence is likely to be: a. urge b. stress c. mixed d. overflow

c. mixed

25. which of these conditions is most likely associated with CIN 2? a. carcinoma in situ b. atypical squamous cells of undermined significance c. moderate dysplasia d. HPV

c. moderate dysplasia indicates moderate cervical dysplasia under bethesda system (TBS). TBS is a standardized measure of degree of abnormal cells in pap test results ranges from least amount of dysplasia to highest (CIN 1 to CIN 3). HPV and mild dysplasia which cause low-grade dysplasia are classified under CIN 1, and severe dysplasia and carcinoma in situ are under CIN 3. atypical squamous cells of undetermined significance are under ASC-US.

5. you have just diagnosed your male pt with chancroid. you order a second battery of tests knowing that other conditions are commonly associated with the presentation of chancroid. which of the following conditions would you LEAST expect to find? a. syphilis b. HSV c. molluscum contagiosum d. HIV

c. molluscum contagiosum no known correlation between prevalence of chancroid and molluscum contagiosum. chancroid is usually painful ulcers, molluscum contagiosum often presents with painless keratinized growths. chancroid is also often a co-factor for HIV transmission.

98. a 22-year-old woman complains of pelvic pain. PE reveals cervical motion tenderness and uterine tenderness. which of the following would further support a diagnosis of PID? a. a temp < 100ºF b. absence of WBC in vaginal fluid c. mucopurulent vaginal discharge d. lab documentation of cervical infection with E. coli

c. mucopurulent vaginal discharge

145. a patient with testicular torsion will have a: a. negative cremasteric reflex b. positive cremasteric reflex c. negative cremasteric reflex on the affected side d. positive cremasteric reflex on the affected side.

c. negative cremasteric reflex on the affected side also high riding testis, profound swelling and acute onset of scrotal pain

26. a patient presents to your clinic with complaints of fever & chills. while discussing the pt's hx, she mentions that both urination and sex have been painful in the past few weeks, and that she keeps getting up in the middle of the night to urinate. you suspect that the pt may have PID; however, which of her complaints is NOT associated with PID? a. chills b. dysuria c. nocturia d. dyspareunia

c. nocturia this is a symptom of UTI.

50. which of the following is the best choice of therapy in chronic bacterial prostatitis? a. oral TMP-SMX for 2 wks b. parenteral ampicillin for 4wks c. oral ciprofloxacin d. injectable gentamicin for 2wks

c. oral ciprofloxacin

34. you see a 42-year-old man with uncomplicated urogenital gonorrhea. his medical records indicate a severe allergic reaction to penicillin that includes difficulty breathing and diffuse urticaria. you recommend tx with: a. oral cefixime b. parenteral levofloxacin c. oral zithromycin plus oral gemifloxacin d. parenteral tigecycline plus oral metronidazole

c. oral zithromycin plus oral gemifloxacin

62. to prevent a recurrence of testicular torsion, which of the following is recommended? a. use of a scrotal support b. avoidance of testicular trauma c. orchiopexy d. limiting the number of sexual partners

c. orchiopexy

121. a 42-year-old female patient presents with difficulty conceiving despite following ovulation indicators and timing intercourse accordingly. the NP suspects "false-positive" ovulation results and refers the patient to a fertility specialist for evaluation of all the following conditions except: a. PCOS b. ovarian insufficiency c. ovarian cancer d. endometriosis

c. ovarian cancer false positive tests that detect luteinizing hormone can occur in women who are menopausal or have PCOS, endometriosis or ovarian insufficiency. ovarian cancer would involve complaints of abdominal pain, backache, fatigue, or changes in BMs.

25. which of the following contraceptive methods should NOT typically be suggested to a woman who weighs > 90kg? a. depo-provera b. implanon c. patch d. mirena

c. patch related to pharmacokinetic differences associated with increased adipose tissue. predisposing factor for development of DVT.

43. all of the following are likely to be reported by patients with acute bacterial prostatitis except: a. perineal pain b. irritative voiding symptoms c. penile discharge d. fever

c. penile discharge

19. a 21-year-old female presents with six .5cm HPV lesions on her vulva. an appropriate tx option for this pt would be: a. acetic acid b. colposcopy c. podophyllin d. acyclovir

c. podophyllin

82. diagnostic testing of a person with primary HSV-2 infection would likely show: a. negative virological and serological test results b. negative virological test result and positive serological test result c. positive virological test result and negative serological test result d. positive virological and serological test results

c. positive virological test result and negative serological test result

1. which of the following is a contraindication to estrogen/progestin-containing methods (combined oral contraception, patch or nuvaring)? a. mother with hx of breast ca b. personal hx of HAV at age 10 years c. presence of factor V leiden mutation d. cigarette smoking one pack per day in a 22yo

c. presence of factor V leiden mutation

13. during a periodic health exam, your 19yo pt tells you that she normally experiences pain during the first few days of her menstrual cycle. she describes the pain as a cramp that radiates to her back and upper thighs, and states that the pain is usually accompanied by nausea and fatigue. based on the pt's presentation, high levels of which hormone are most likely responsible for her symptoms? a. estrogen b. progesterone c. prostaglandin d. dopamine

c. prostaglandin symptoms indicate primary dysmenorrhea, which occurs in younger women d/t high levels of prostaglandin. estrogen or progesterone is found in oral BCPs, which may be useful in treating primary dysmenorrhea. dopamine levels may be increased through use of antidepressants, such as selegiline, which may produce dysmenorrhea as an adverse effect.

3. a 31-year-old female patient presents with fatigue, fever (101F), worsening unilateral low back pain for past 5 days. her pain is 5/10 on the pain scale & has been unresponsive to ibuprofen. she denies abd pain, but is anorexic and nauseas. she denies vaginal discharge. UA demonstrates hematuria, the presence of WBC casts, leukocytes and nitrites. what should be included in the differential diagnosis? a. PID b. renal stone c. pyelonephritis d. UTI

c. pyelonephritis

9. three days after delivering a healthy baby girl, a mother calls the office with concerns that her daughter's vagina is swollen with a small amount of bleeding. the NP will: a. refer to the ED b. instruct the mom to bring newborn into office for eval c. reassure mom this is normal d. instruct mother to cleanse newborn's genitals with soap and warm water

c. reassure mom this is normal at approx 2-3 days of age, newborn female may have bleeding from vagina, labia may be swollen. normal and caused by withdrawal of hormone exposure in the womb. no need for office or ER eval unless bleeding does not stop. gently cleanse genitals but do not use soap, can irritate the vaginal mucosa.

13. your pt has an L3 immunotype of chlamydia trachomatis. which sign or symptom would be most expected from the suspected condition? a. painful ulcers b. green-yellow discharge c. regional adenopathy d. dysmenorrhea

c. regional adenopathy most commonly associated with LGV. usually has ulcers, often painless, painful ulcers are hallmark of chancroid. discharge in advanced cases of LGV is bloody and purulent, not green-yellow. dysmenorrhea is often a symptom of gonorrhea.

128. a 32yo female reports that she believes she was exposed to HIV 4wks ago and requests testing. she is given a rapid antibody test and the results are negative. the NP recommends: a. no further testing is necessary b. repeat testing in 2wks c. repeat testing in 2mos d. repeat testing in 1yr

c. repeat testing in 2mos

92. a 26-year-old male reports that he was likely exposed to HIV via sexual activity 4wks ago and requests testing. he is given a rapid antibody test (oraquick HIV test) and the results are negative. the NP recommends: a. no further testing is necessary b. repeat testing in 2wks c. repeating testing in 2 months d. repeat testing in 1 year

c. repeating testing in 2 months

15. in a private NP clinic, a pt presents with trichomonas. state law requires reporting of STDs to the public health department. the pt asks the NP not to report it because her husband works in the public health department. how should this be managed by the NP? a. respect the pt's right to privacy and not report it b. tell the pt that it wont be reported, but report it anyway c. report it to public health required by law d. report it to public health but dont divulge all the details

c. report it to public health required by law

1. a 16yo pt experiencing primary amenorrhea should be referred to a specialist and evaluated for various causes, the following are common known causes of primary amenorrhea except for: a. turner syndrome b. vaginal agenesis c. resistant follicular stimulation d. endocrine imbalance

c. resistant follicular stimulation this is cause of abnormal heavy uterine bleeding. primary amenorrhea is characterized by lack of menarche by age 16. it may also cause an absence of sexual characteristics or abnormal growth and development. diagnosis often requires full exam from endocrinologist to determine proper etiology. possible causes of primary amenorrhea include chromosomal disorders such as : turner syndrome; anatomic abnormalities, such as vaginal agenesis, and hormonal imbalances due to endocrine abnormalities

31. a patient requests screening for HIV after a sexual exposure. what are CDC's recommendations for screening this pt? a. she only requires screening if she develops s/s of HIV b. she should be re-screened in 6-12mos c. she should be screened today, with repeat screening at 4-6wks and 3 months d. there are no recommendations for this screening

c. she should be screened today, with repeat screening at 4-6wks and 3 months

82. an elderly patient has a + PCR for varicella zoster virus and has recent hx of breast CA. all of the following interventions should be included in the tx plan except: a. acyclovir daily x 10 days b. lidocaine 5% patch c. shingles/zoster (shingrix) vaccine d. gabapentin TID

c. shingles/zoster (shingrix) tx plan for elderly male who is immunocompromised and dx with shingles is acyclovir x 10 days, lidocaine patch and gabapentin TID for post herpetic neuralgia. since pt is immunocompromised, she should not get the vaccine.

75. which of the following findings is most consistent with vaginal discharge during ovulation? a. dry and sticky b. milky and mucoid c. stringy and clear d. tenacious and odorless

c. stringy and clear

5. of the available agents to prescribe for erectile dysfunction, which has the fastest onset and longest duration? a. saw palmetto b. sildenafil c. tadalafil d. vardenafil

c. tadalafil fastest onset of 15mins and longest efficacy of 36hrs. sildenafil and vardenafil generally require 30mins to take effect and may show effects for up to 4 hours. saw palmetto may help with ED stemming from prostate issues, btu is less helpful in treating ED resulting from other cases such as stress or antidepressant use.

20. an exam of andrew, a 28yo male reveals a non-indurated ulcer on his prepuce that he claims appeared 3 days ago, he states that he did some research online and believes it may be either chancroid or lymphogranuloma venereum (LGV). while examining him, which finding would you tell him best distinguishes chancroid from LGV? a. only LGV typically starts as a small papule b. LGV is asymptomatic in men c. tender inguinal lymphadenopathy is generally indicative of LGV d. only LGV may present with buboes

c. tender inguinal lymphadenopathy is generally indicative of LGV more commonly feature in LGV. can present in chancroid. both usually have small papules that turn into ulcers. painful buboes may be alongside lymphadenopathy in cases of chancroid and secondary LGV. both chancroid and LGV may be asymptomatic in women, men typically experience s/s when infected with either.

a 10 year old boy complains of sudden onset of scrotal pain upon awakening that morning. he is also c/o severe NV. during the PE, the NP finds a tender, warm and swollen left scrotum. the cremasteric reflex is negative, the urine dipstick is negative for leukocytes, nitrites and blood. 36. the most likely diagnosis is: a. acute epididymitis b. severe salmonella infection c. testicular torsion d. acute orchitis 37. what type of f/u should this pt receive? a. refer to urologist within 48 hours b. Refer to ED as soon as possible C. Prescribe ibuprofen 600mg QID for pain D. Order a testicular torsion US for further evaluation

c. testicular torsion s/s include sudden onset unilateral scrotal pain, nausea, vomiting and abd pain. acute epididymitis causes fever, chills, nausea and unilateral pain and is most commonly seen in sexually active men. unilateral scrotal pain does not occur with salmonella infection. acute orchitis is often based on having a recent mumps infection (parotitis) with testicular edema. b. refer him to the ER ASAP prevent irreversible ischemia. success of tx is usually 100% if tx within first 6 hours and 0% if tx after 24 hours. dx of torsion is often made clinically but if in doubt, US helpful. need dx & tx in 4-6 hours to prevent necrosis.

20. a patient newly diagnosed with diabetes reports severe hives and swollen lips after taking bactrim for a bladder infection 2 months ago. which of the following statements is correct? a. the pt cannot take any pills in the sulfonylurea class b. the patient can take some pills in the sulfonylurea class c. the pt can take any of the pills in the sulfonylurea class d. none of the above

c. the patient can take any of the pills in the sulfonylurea class there is no evidence that cross-reactivity between the sulfonamide abx (sulfamethoxazole) and nonantimicrobial sulfonamides (furosemide, thiazides, sulfonylurea hypoglycemics, protease inhibitors, carbonic anhydrase inhibitors). only have to worry about sulfasalazine.

33. how should a spermicide be applied in conjunction with use of a diaphragm for maximum efficacy? a. the spermicide should be applied to the vagina immediately following intercourse while leaving the diaphragm in place b. the spermicide should be placed around the outside of the diaphragm, which is then removed immediately after intercourse c. the spermicide should be placed inside the diaphragm, which is then removed at least 6 hours after intercourse d. the diaphragm should be removed immediately following intercourse so that spermicide can be applied

c. the spermicide should be placed inside the diaphragm, which is then removed at least 6 hours after intercourse spermicide can be applied inside the vagina without removing the diaphragm but this is only recommended for repeated intercourse, not first encounters

31. when considering the use of etonogestrel subdermal implant (nexplanon) for contraception, the NP realizes that: a. three rods are placed under the skin b. this method provides effective contraception for up to 2 years c. this method can lose effectiveness over time in overweight or obese women d. this method is considered acceptable for select women with a history of thrombophilia forms

c. this method can lose effectiveness over time in overweight or obese women

67. a male patient presents with reports of urethritis, migratory arthritis in the large joints, and ulcers on the glans penis secondary to a chlamydial infection. NP will implement all of the following interventions except: a. treat with NSAIDs b. prescribe abx to tx infection c. treat with alpha-linolenic acid (LNA) d. recommend rest and comfort measures

c. treat with LNA reiter's syndrome is an immune-mediated reaction secondary to chlamydia that causes red, swollen joints; ulcers on the skin of the glans of the penis; and urethritis. it resolves on its own. TX is NSAIDs and comfort measures. tx with abx will tx underlying infection which is chlamydia. LNA is used in treatment of ankylosing spondylosis.

67. the NP knows that she is HIV positive. she is employed in a private clinic and performs wellness exams on ambulatory adults. the NP is: a. obligated to inform her employer of her HIV status b. obligated to inform her patients of her HIV status c. under no obligation to inform anyone d. under obligation to inform the patient if she performs invasive procedures

c. under no obligation to inform anyone

4. a 64yo male presents to your practice with complaints regarding urination. he says that he has trouble "getting started" when he wishes to void and that the stream "keeps leaking" when he tries to terminate it. he also wakes frequently with the urge to urinate. upon further discussion, he mentions with some embarrassment that he "let go" while celebrating a victory for his sports team. which diagnostic test should the NP order initially? a. PSA b. gram stain c. urinalysis d. 24hr urine collection

c. urinalysis s/s indicate a UTI, BPH or prostate cancer. gram stain can be helpful to determine causative agents, but it is labor intensive. PSA is primarily useful in diagnosing BPH and prostate cancer, and typically comes after UA to r/o UTI. urine collection urine 24hrs is typically step to diagnosing kidney disorders, whereas s/s more strongly indicate urogenital involvement

28. which of the following lab results would best indicate that a patient's HIV is well controlled? a. viral load: detectable; CD4 lymphocyte count: 300 cells/uL b. WBC: 5,000 cells/mL c. viral load: undetectable; CD4 lymphocyte count: 700cells/uL d. WBC: 10,000 cells/mL

c. viral load: undetectable; CD4 lymphocyte count: 700cells/uL CD4 is at or near normal levels (>800cells/uL) & if viral load undetectable. CD4 of 300 indicates pt is progressing towards AIDS. lastly, even though WBC are within normal ranges, they are not a reliable indicator of pt's HIV status, counts are better at determining if pt has opportunistic infection.

34. which of the following is LEAST definitive test in diagnosing PID? a. STD screening b. ESR test c. US d. TSH test

d. TSH test most often stems from infection. dx include STD screening and ESR to rule out other conditions. US are used to see ovarian cysts and scarring in repro organs

5. a 15yo male has worked this summer as a lifeguard at a local swimming pool. he complains of itching in the groin area. he is diagnosed with tinea cruris. the NP is likely to identify: a. swelling of the scrotum b. macular lesions on the penis c. well marginated half-moon macules on the inner thigh d. maceration of the scrotal folds with erythema of the penis

c. well marginated half-moon macules on the inner thigh affects scrotum and inner thighs, but never affects penis and never shows scrotal swelling.

18. which of the following diagnostic tests is used to confirm HIV? a. ELISA b. CD4 lymphocyte count c. western blot d. viral load

c. western blot after initial screening with an ELISA test. high viral load and low CD5 lymphocyte count indicates high risk of progression to AIDS, but tests are not typically used in initial screening.

80. which diagnostic test is used to confirm a diagnosis of vaginal candida infection? a. pH b. whiff test c. wet mount d. tzanck smear

c. wet mount pH not specific enough, whiff test for BV, tzanck smear is for herpetic infection

86. patient education about the use of sildenafil (viagra) includes all of the following: a. a spontaneous erection occurs about 1 hour after taking the medication b. this medication helps regain erectile function in nearly all men who use it c. with the use of the medication, sexual stimulation also is needed to achieve erection d. nitrates can be safely used concurrently

c. with the use of the medication, sexual stimulation also is needed to achieve erection

59. a 35yo pt is HIV positive. which finding on an oral swab may be indicative of thrush? a. RBCs b. spores c. yeast d. epithelial cells

c. yeast

4. an 18-year-old patient comes to your office to inquire about spermicides, and asks about the failure rate of spermicides when used alone. as an NP, you tell her the typical first-year failure rate is: a. approx 11% b. ~16% c. ~21% d. ~32%

c. ~21% failure rate reduced when used with condoms or diaphragms.

110. what percentage of anogenital and cervical cancers can be attributed to HPV infection? a. < 30% b. ≥ 50% c. ≥ 70% d. ≥ 95%

c. ≥ 70%

8. the NP does not need to obtain parental consent from all the following pts except: a. 17yo who wants to be tx for STI b. 12yo who wants serum pregnancy test c. 15yo who wants BCP d. 14yo who wants to be tx for dysmenorrhea

d. 14yo who wants to be tx for dysmenorrhea do not need consent for STI tx, pregnancy testing or contraception counseling and tx. parental consent is required for any type of physical exam or for other problems that require more invasive testing.

94. a 3yo male was seen in office 8 days ago and given amoxicillin 200mg PO suspension TID x 10 days for strep throat. he presents for a f/u visit with fatigue, reduce appetite, and dark-colored urine. a urine dipstick test reveals protein, albumin, and blood in urine. which test will the NP order next? a. CT scan of kidneys b. fasting BMP with CBC with diff c. 24-hour urine collection for protein and creatinine clearance d. 24-hour urine collection for blood, protein and creatinine clearance

d. 24 hour urine collection for blood, protein and creatinine clearance more sensitive evaluation of renal function.

84. suppressive therapy reduces the frequency of genital herpes recurrences by: a. 5-10% b. 20-25% c. 40-50% d. 70-80%

d. 70-80%

95. suppressive therapy reduces the frequency of genital herpes recurrences by: a. 5-10% b. 20-25% c. 40-80% d. 70-80%

d. 70-80%

3. what is the estimated rate of male-to-female transmission of gonorrhea following initial exposure? a. a rate of 20-30% b. 40-50% c. 60-70% d. 80-90%

d. 80-90%

11. a 44-year-old female pt is dx with UTI. which bacteria count collected via midstream, clean catch supports a dx of UTI? a. > 10,000 bacteria b. > 25,000 bacteria c. > 50,000 bacteria d. > 100,000 bacteria

d. > 100,000 bacteria

114. an example of a drug that targets the renin-angiotensin-aldosterone system is an: a. diuretic b. calcium channel blocker c. beta blocker d. ACEI

d. ACEI

133. a female pt experiencing frequent UTIs and vaginal infections is being seen in the office for the third time in 7 months with complaints of urinary frequency and burning, vaginal discharge and severe itching. the NP orders testing for which of the possible underlying conditions? a. DI b. anemia c. H. pylori infection d. DM

d. DM persistent urinary and vaginal infections may indicate underlying glucose metabolism disorders and DM. DI is a rare metabolic disorder affecting fluid balance (excessive fluid intake and urination) in body and would not be screened for with the symptoms of this case.

33. a 64yo male presents to your practice with low back pain and fever and mentions that he wakes up at least 4x a night with an urgent need to urinate. an exam indicates that the pt's prostate is boggy and tender to palpation. a urine cx returns positive results for a causative agent. based on the pt's s/s and clinical findings, which pathogen would be the most likely cause of the pt's condition a. chlamydia trachomatis b. mycoplasma genitalium c. gardenerella vaginalis d. E. coli

d. E. coli this is acute bacterial prostatitis which matches the symptoms and exam. positive urine cx confirms this. chlamydia trachomatis is more causative of epididymitis in younger men but can cause bacterial prosatitis. the other two are not common but may cause nonbacterial prostatitis in men

8. which of the following types of contraceptives may be used to prevent the recurrence of asherman's syndrome? a. injected contraception b. cervical caps c. disposable barriers d. IUD

d. IUD this is because of the ability to be placed in the uterine cavity to create a barrier between walls of uterus. implantation after initial removal of scar tissue in uterus may help with healing and facilitate separation of tissues.

15. after performing an assessment on a 20yo female, you recommend that she significantly reduce her caffeine and salt intake. you also recommend that she eats more foods containing vitamins E and B6 to help reduce or manage symptoms associated with which condition? a. fibrocystic breast disease b. secondary dysmenorrhea c. PID d. PMS

d. PMS also similar in secondary dysmenorrhea, but they need further tx and do not need increased B6. PID is tx with abx.

71. a 23-year-old female who is homeless presents to the free clinic. she should be screened for: a. diabetes, HIV and neuropathy b. UTI and STDs c. pregnancy, headaches and STDs d. TB, HIV and hepatitis

d. TB, HIV and hepatitis

42. a 24-year-old female patient who is sexually active complains of vaginal itching. if she has BV, she might complain of: a. midcycle bleeding b. frothy, bloody vaginal discharge c. a truncal rash d. a "fishy" vaginal odor after coitus

d. a "fishy" vaginal odor after coitus clinical syndrome with high concentrations of anaerobic bacteria replace normal vaginal flora. also have itching and vulvovaginal pruritus and burning.

25. how should a patient with suspected syphilis be screened? a. ask about s/s in pt b. ask about s/s of sexual partners c. a urethral swab for cx d. a serum assessment

d. a serum assessment

100. the presence of an adnexal mass in the woman with PID most likely indicates presence of: a. uterine fibroids b. an ectopic pregnancy c. ovarian malignancy d. a tubo-ovarian abscess

d. a tubo-ovarian abscess

20. a 35-year-old man presents with edema of the face, hands and ankles along with hypertension (175/115). he reports urine that is darkly colored and foamy. you suspect acute glomerulonephritis and would expect urinalysis results to include all of the following findings except: a. elevated level of protein b. presence of RBCs c. presence of renal casts d. abnormally high glucose levels

d. abnormally high glucose levels

78. a 30-year-old male who is sexually active complains of pain during bowel movements. the DRE is negative for hemorrhoids, but the prostate gland is tender. what should be suspected? a. trichomoniasis b. BPH c. prostate cancer d. acute bacterial prostatitis

d. acute bacterial prostatitis

40. risk factors for acute bacterial prostatitis include all of the following except: a. having unprotected sex b. use of urinary catheter c. prior bladder infection d. age > 70 yrs

d. age > 70 yrs

43. a patient being treated for trichomoniasis receives a prescription for metronidazole. what instructions should she be given? a. take this med with food b. do not take this med if you are pregnant c. take this med on an empty stomach d. alcohol should be avoided when taking this med

d. alcohol should be avoided when taking this med

115. the NP finds multiple types of bacteria in a pt's urine sample. which of the following is most appropriate? a. order an IVP b. order an MRI c. perform a microsopic UA d. ask the pt to resubmit a clean catch specimen

d. ask pt to resubmit a clean catch specimen multiple bacteria in urine sample is indicative of a contaminated sample.

39. a 19yo pt is using a cervical cap as a contraceptive. she asks you how long must the cervical cap be left in the vagina following intercourse. you should tell her which of the following? a. two hours at most b. 3 hours at most c. at least 4 hours d. at least 6 hours

d. at least 6 hours

10. after a vaginal exam, a patient received a prescription for metronidazole 500mg BID for 7 days. what was her likely diagnosis? a. gonorrhea b. chlamydia c. syphilis d. bacterial vaginosis

d. bacterial vaginosis

150. a young female adult presents with vaginal discharge and itching. besides trichomoniasis and yeast, what else should be included in the differential? a. syphilis b. herpes genitalis c. chlamydia d. bacterial vaginosis

d. bacterial vaginosis

1. which of the following in inconsistent with the description of BPH? a. obliterated median sulcus b. size > 2.5 x 3 cm c. sensation of incomplete emptying d. boggy gland

d. boggy gland

8. a patient with PID seeks outpatient treatment for her condition. as she is intolerant of fluoroquinolones, you decide to put her on a regimen that focuses on doxycycline. according to the most recent CDC recommendations for treating PID, which of the following drugs would NOT be paired with doxycycline? a. cefoxitin b. probenecid c. metronidazole d. butoconazole

d. butoconazole more commonly used for candidiasis not part of PID tx. outpatient therapy for PID that does not use fluoroquinolones pairs 2g IM cefoxitin with 1g probenicid administered orally, followed by 100mg of doxycycline given PO BID for 14d. metronidazole is used for BV.

34. an NP sees a patient for an eval of STDs. Assessment reveals Fitz-Hugh-Curis syndrome. the NP will prescribe: a. benzanthine penicillin G IM weekly x 3 weeks b. doxycycline 100mg PO BID x 14 d c. ceftriaxone 500mg IM x one doses plus azithromycin 1g PO x 1 dose d. ceftriaxone 500mg IM x 1 dose plus doxycycline 100mg BID x 14 days plus metronidazole 500mg BID x 14d

d. ceftriaxone 500mg IM x 1 dose plus doxycycline 100mg BID x 14 days plus metronidazole 500mg BID x 14d fitz-hugh-curtis syndrome should be tx as complicated gonorrheal/chlamydial infection: ceftriaxone 500mg IM x1 dose plus doxycycline 100mg BID plus metronidazole 500mg BID x 14d for anaerobic coverage. benzathine penicillin G 2.4 mU IM weekly x 3weeks used to tx complicated syphilis infections. doxycycline appropriate for chlamydial infections. pregnant pt with confirmed uncomplicated gonorrheal and chlamydial infections would be RX ceftriaxone 500mg IM x 1 dose plus ZPAK 1g PO x 1 dose

36. a pt presents with lower abd pain and vomiting. her temp is 40ºC. you perform a cervical exam, and your patient tests positive for cervical motion tenderness. upon further testing, you identify chlamydia trachomatis. based on the likely dx, which regimen would you most likely prescribe? a. levofloxacin and probenicid b. ofloxacin and ceftriaxone c. cefoxitin and probenecid with or without ceftriaxone d. ceftriaxone and doxycycline with or without metronidazole

d. ceftriaxone and doxycycline with or without metronidazole based on s/s and positive test results, she is most likely experiencing PID and this is the course of tx. other regimens include ofloxacin or levofloxacin with or without metronidazole, or cefoxitin plus probenecid and doxycycline with or without metronidazole.

35. george, a 32yo male, comes to the clinic with swollen lymph nodes in his inguinal area. this finding is typically a sign of all the following conditions EXCEPT: a. lymphogranuloma venereum b. chancroid c. syphilis d. chlamydia

d. chlamydia although the same genus of bacteria causes chlamydia and LGV, chlamydia does not typically present with lymphadenopathy, whereas LGV does. chancroid and syphilis pts may also present with regional lymphadenopathy in inguinal lymph nodes.

146. a 20 year old pregnant woman has been diagnosed with gonorrhea. the NP should test the pt for: a. HSV-1 b. genital warts c. syphilis d. chlamydia

d. chlamydia high rates of co-infection especially in pregnancy.

9. which of these STDs is caused by a parasitic pathogen? a. chancroid b. molluscum contagiosum c. condyloma acuminata d. chlamydia

d. chlamydia small gram negative bacteria that is an obligate intracellular parasite. chancroid produces haemophilus ducreyi, facultative anaerobic bacillus that is not parasitic in nature. both molluscum contagiosum and condyloma acuminata are caused by viruses, with former producing from molluscum contagiosum virus and latter producing from HPV.

47. appropriate antimicrobial treatment for a 65yo man with acute bacterial prostatitis is a course of oral: a. erythromycin b. cefepime c. TMP-SMX d. ciprofloxacin

d. ciprofloxacin

49. for a woman with bothersome hot flashes who cannot take HT, alternative options with demonstrated efficacy and limited adverse effects include the use of all of the following except: a. venlafaxine b. sertraline c. gabapentin d. clonidine

d. clonidine

1. all of the following STDs are paired with a pathogen commonly responsible for the condition EXCEPT: a. PID - Neisseria gonorrheae b. lymphoranuloma venereum - chlamydia trachomatis c. chancroid - haemophilus ducreyi d. condyloma acuminata - treponema pallidum

d. condyloma acuminata - treponema pallidum also called genital warts, are produced by HSV, not treponema pallidum. syphilis is caused by treponema. PID can be caused by neisseria gonorrhoeae and more commonly, chlamydia trachomatis. lymphogranuloma venereum is caused by C. trachomatis and chancroid is caused by haemophilus ducreyi.

134. a patient has been diagnosed with pyelonephritis. he probably will have: a. pain that waxes and wanes b. burning with urination c. a positive Murphy's sign d. costovertebral angle tenderness

d. costovertebral angle tenderness

30. robin, a 45yo female, presents to the clinic with rounded, firm, painless bumps along her lower abd and genital region. you note that the papules are about 3mm and pearly-white. which is the most common form of treatment for the most likely condition? a. aspiration b. antipruritic agents c. laser therapy d. cryoanesthesia with liquid nitrogen

d. cryoanesthesia with liquid nitrogen aspiration usually used to lance buboes and prevent ulceration associated with LGV. antipruritic agents are used for s/s of herpes. laser therapy used for genital warts. laser therapy can remove molluscum contagiosum papules but usually reserved for extensive presentations seen in immunocompromised pts.

163. which assessment finding demonstrates that use of a vaginal dilator for atrophic vaginitis has been effective? a. atrophic labia b. decreased rugae c. narrowing of the vagina d. decreased discomfort with sexual activity

d. decreased discomfort with sexual activity vaginal atrophy (atrophic vaginitis) is thinning, drying and inflammation of the vaginal walls that may occur when body has less estrogen, as with postmenopausal women. if painful sex is a concern, vaginal dilators may relieve vaginal discomfort by stretching to widen the vaginal canal to assist in reducing discomfort during intercourse. atrophic labia and decreased rugae are s/s of atrophic vaginitis.

116. a female patient who is 45yo states that she is having urinary frequency. she describes episodes of "having to go right now" and not being able to wait. her UA results are WNL. what is part of the differential? a. symptomatic bacteriuria b. stress incontinence c. lupus d. diabetes

d. diabetes

17. a woman comes into the clinic stating that she thinks that she has a STD because she has pain in the general groin area. upon exam, you identify genital ulcers on her labia that she confirms are painful, as well as tender inguinal lymphadenopathy. you suspect chancroid but know that a definitive diagnosis is difficult. which statement regarding a morphological diagnosis is true? a. the gram stains for chancroid and gonorrhea identify the same bacteria b. absence of HSV is part of a definitive diagnosis for chancroid c. the bacteria that causes chancroid cannot be identified via gram stain d. diagnosis requires identification of chancroid-causing bacteria, but sensitivity is less than 80%

d. diagnosis requires identification of chancroid-causing bacteria, but sensitivity is less than 80% morphological diagnosis of chancroid is difficult because haemophilus ducreyi, the bacteria that causes it, must be identified on special cx media that is not widely available and has sensitivity of less than 80%. absence of HSV is part of probable, not definitive diagnosis, as in absence of treponema pallidum. gram stain for gonorrhea shows gram-negative diplococci; gram stain for chancroid shows gram-negative bacilli.

139. which of the following meds should be avoided in a 65yo male with BPH? a. propanolol (inderal) b. ciprofloxacin (cipro) c. doxazosin (cardura) d. diphenhydramine (benadryl)

d. diphenhydramine (benadryl) avoid antihistamines. cause urinary retention. also impair urination: muscle relaxants, narcotics, TCAs and meds with anticholinergic properties like antihistamines

30. a 21yo woman complains of left-sided pelvic pain accompanied by dyspareunia. during the GYN exam, the NP notices green cervical discharge. the pt mentions new onset of painful and swollen left knee and denies hx of trauma. this best describes: a. septic arthritis b. reiter's syndrome c. chondromalacia of the patella d. disseminated gonorrheal infection

d. disseminated gonorrheal infection s/s of PID with painful, swollen joints of extremities indicate disseminated gonorrheal inection. untx can lead to septic arthritis. s/s may be mild, ranging from slight joint pain and no fever to severe joint pain with high fever. PID s/s do not occur with septic arthritis, reiter's syndrome, or chondromalacia of patella.

26. a pt arrives for a regular injection of depo-provera. however, in consulting your records you find that it has been 14wks since the pt received her last injection. when you ask when her cycle begins, she says "i don't know." after administering the injection, you should caution her to use backup contraception for how long? a. during the first 2 days after injection b. during first 5 days after injection c. during first week after injection d. during first 2wks after injection

d. during first 2wks after injection must be given every 12-13wks for full efficacy, should pt miss the window, should use backup for first 2wks.

41. The NP is discussing tetracycline with a young sexually active female patient being treated for a bacterial sinus infection. The NP will advise the patient: a. not take birth control pills while on tetracycline b. practice abstinence while on abx and then resume BCP c. not take tetracycline and BCP at the same time d. employ an additional method of contraception while on the abx and for one pill cycle after treatment is complete

d. employ an additional method of contraception while on the abx and for one pill cycle after treatment is complete

32. which of the following is a commercially available vaccine that is implemented to prevent and protect against the hepatitis B virus? a. kinrix-B b. varivax-B c. hiberix-B d. engerix-B

d. engerix-B uses noninfectious HBV surface antigens so that body can build immunity to disease. along with recombivax-HB, it is one of the most widely used vaccines for HBV.

21. a patient with newly dx genital herpes would appropriately receive a prescription for: a. cryotherapy b. ceftriaxone c. imiquimod d. famciclovir

d. famciclovir

22. a sexually active young adult is dx with chlamydia trachomitis and tx with doxycycline 100mg BID for 7 days. during a follow-up visit, the pt reports a new onset of RUQ pain and tenderness on palpation. ALT is 43 and AST is 24. the NP suspects: a. jarisch-herxheimer reaction b. reiter's syndrome c. steven-johnson syndrome d. fitz-hugh-curtis syndrome

d. fitz-hugh-curtis syndrome this is PID. s/s include RUQ pain and pain on palpation, normal liver function and "violin string" adhesions revealed on laparoscopy exam. reiter's is more common in men and is secondary reaction to bacteria such as chlamydia. tx is supportive, and syndrome spontaneously resolves. stevens-johnson syndrome is an adverse reaction to abx that produces severe rash and flu-like s/s. jarisch-herxheimer is an acute response from syphilis or other spirochete tx in first 24 hours and produces acute fever, chills, headache and myalgia.

40. an 86-year-old male has symptomatic BPH. what choice may exacerbate the s/s of BPH? a. voiding in a seated position b. NSAIDs c. stool softeners d. furosemide

d. furosemide

24. a patient who takes oral contraceptive pills is at increased risk of: a. varicose veins b. hypothyroidism c. depression d. gallbladder disease

d. gallbladder disease one of the major components of gallstones is estrogen.

16. melissa, a 25yo female, comes into the clinic after noticing "yellow-green" vaginal discharge. she also reports pain and swelling "down there"; a physical exam reveals labial swelling. based on these findings, which is the most likely diagnosis? a. chancroid b. lymphogranuloma venereum c. HIV d. gonorrhea

d. gonorrhea mucopurulent vaginal discharge and labial pain & swelling usually indicates gonorrhea. other s/s - dysuria, increased frequency, lower abd pain, abnormal periods. chancroid does not usually cause mucopurulent discharge. LGV may cause swelling in rectal or inguinal region, but not labial, more often has painful ulcers. women with HIV more often get yeast infections, typically odorless, white discharge.

116. which of the following is found in tertiary syphilis? a. arthralgia b. lymphadenopathy c. macular or papular lesions involving the palms and soles d. gumma

d. gumma

69. which of the following is found in tertiary syphilis? a. arthralgia b. lymphadenopathy c. maculopapular lesions involving palms and soles d. gumma

d. gumma

51. in advising a perimenopausal woman about HT, you consider that it can: a. reduce the risk of venous thrombotic events b. significantly reduce serum triglyceride levels c. worsen hypertension in most women d. help preserve bone density

d. help preserve bone density

29. an exam of a patient reveals a painful lesion on her cervix. she is also experiencing a fever and has been feeling extremely tired. you know that which of the following is the most likely diagnosis? a. lymphogranuloma venereum b. genital warts c. chancroid d. herpes

d. herpes although LGV and genital warts also present with vesicles or ulcers, these lesions are painless. chancroid is usually asymptomatic in women; men are more likely to develop painful ulcers surrounded by an erythematous halo.

21. a 22yo male comes to the clinic complaining of pain in the groin area. upon exam of his groin area, you identify abscess formations. based on the p's presentation, what is the best diagnosis for his skin condition? a. erysipelas b. candidiasis c. impetigo d. hidrandenitis suppurativa

d. hidrandenitis suppurativa candidiasis does not produce an abscess.

1. all of the following electrolyte disorders are commonly found in a person with chronic renal failure except: a. hypernatremia b. hypercalcemia c. hyperkalemia d. hypophosphatemia

d. hypophosphatemia

12. a 50-year-old female believes that she is menopausal. she complains of hot flashes and has not had menses in 12mos. which of the following test results would be expected during menopause? a. increased TSH b. decreased FSH c. hypoestrogenemia d. increased FSH

d. increased FSH

32. which of the following would you LEAST expect to occur during menopause? a. loss of elasticity in the skin b. changes in skin pigmentation c. loss of muscle tone d. increased sebaceous gland activity

d. increased sebaceous gland activity may experience decreased activity rather than increased.

132. 5-alpha-reductase inhibitors work by producing: a. dilation of the detrusor vessels b. a decrease in the size of the prostate c. increased blood flow to the prostate d. inhibition in the prostate tissue synthesis

d. inhibition in the prostate tissue synthesis

37. an 18-year-old woman requests emergency contraception after having unprotected vaginal intercourse approximately 18 hours ago. today is day 12 of her normally 27-29 day menstrual cycle and she has no contraindications to the use of any currently available forms of emergency contraception. you advise her that: a. emergency hormonal contraception use reduces the risk of pregnancy by approximately 33% b. all forms of emergency contraception must be used within 12 hours after unprotected intercourse c. the likelihood of conception is minimal d. insertion of a copper containing IUD offers an effective form of emergency and ongoing contraception

d. insertion of a copper containing IUD offers an effective form of emergency and ongoing contraception

32. when comparing the etonogestrels, it is important to note that nexplanon: a. has a higher concentration of progestin b. is effective for a longer period of time compared with implanon c. can be self-administered d. is visible on x-ray

d. is visible on x-ray

92. undiagnosed diabetes might present as: a. paresthesias in the UE b. varicose veins c. chronic halitosis d. recurrent vaginal candidiasis

d. recurrent vaginal candidiasis candida is part of normal vaginal flora. however when glucose levels rise, candida dramatically increases in number. it is an ideal environment to grow yeast.

24. which of the following choices is NOT a standard advantage of using a diaphragm or cervical cap? a. it is relatively safe and easy to use b. it provides immediate protection c. when used with spermicidal gel, it may protect against STDs d. it remains in place during intercourse

d. it remains in place during intercourse can be disturbed during act, this is an advantage of using sponge. advantages are being relatively safe and easy to use, provides immediate protection and guarding against STIs when used with spermicidal gel. same with cervical cap, but protection from STIs is limited, can provide some.

10. which recommendation below reflects CDC's recommendation for administration of the zoster vaccine? a. it can be given regardless of immune status at age 60 b. it can be given regardless of immune status at age 50 c. it should be given to immunocompetent adults age 65 and older d. it should be given to immunocompetent adults age 60 or older

d. it should be given to immunocompetent adults age 60 or older

12. for which of the following types of condoms is use of oil-based lubricants most strongly discouraged? a. synthetic condoms b. polyurethane condoms c. natural membrane condoms d. latex

d. latex can increase risk of condom breakage when used with latex

42. which of the following is not a risk factor for bladder cancer? a. occupational exposure to textile dyes b. cigarette smoking c. occupational exposure to heavy metals d. long-term aspirin use

d. long-term aspirin use

19. you inform your patient, jack, a 25yo male that he is in the latent stage of syphilis. which s/s would best characterize this stage of the disease? a. mucous patches b. indurated ulcers c. skin rash on palmar and plantar surfaces d. no s/s

d. no s/s asymptomatic. indurated ulcers are finding of primary stage, mucous patches & skin rashes on palmar/plantar surfaces are in secondary stage.

39. a 24-year-old woman who requests emergency contraception in oral form wants to know the effects if pregnancy does occur. you respond that there is the risk of increased rate of: a. spontaneous abortion b. birth defects c. placental abruption d. none of the above

d. none of the above

63. a 23-year-old man has a nontender "bag of worms" mass within the left scrotum that disappears when he is in the supine position. he is diagnosed with a varicocele. what is a risk factor that may have contributed to this condition? a. younger age b. current cigarette smoker c. multiple sexual partners d. none of the above

d. none of the above

83. treatment options for BV include all of the following except: a. oral metronidazole b. clindamycin cream c. oral clindamycin d. oral azithromycin

d. oral azithromycin

141. athletic amenorrhea increases the risk of: a. breast cancer b. covert hypothyroidism c. an eating disorder d. osteoporosis

d. osteoporosis

54. concerning selective estrogen receptor modulator therapy such as raloxifene (evista) which of the following statements is correct? a. concurrent progestin opposition is needed b. hot flashes are reduced in frequency and severity c. use in contraindicated when a woman has a history of breast cancer d. osteporosis risk is reduced with use

d. osteporosis risk is reduced with use

78. following a routine annual exam of a 59-year-old postmenopausal patient, the NP refers the pt to a GYN, who then orders a pelvic/intravaginal US. which finding on the exam most likely prompted the referral and subsequent order? a. positive pap smear b. external genital warts c. vaginal atrophy d. palpable ovary

d. palpable ovary ovaries of postmenopausal women should not be palpable; this is abnormal, needs testing to r/o CA. positive pap smear would be dx with colposcopy with conization. genital warts would also get colposcopy. vaginal atrophy is expected finding.

63. for the woman with a history of DVT who is having significant vasomotor s/s, which of the following can be used for symptom management? a. 17B-estradiol patch b. drospirenone c. estrone d. paroxetine

d. paroxetine

36. as a practitioner, you know that the approximate theoretical and actual failure rates for oral contraceptives are: a. perfect use: 0.1%, typical use 10-15% b. perfect: 5%, typical 10% c. perfect 1-3%, typical 1.5-3% d. perfect 0.1%, typical 3-5%

d. perfect 0.1%, typical 3-5%

97. a female pt complains of dysuria with vaginal discharge. how should she be managed? a. order a UA b. order a urine cx c. perform a pelvic exam d. perform an abd exam, UA and pelvic exam

d. perform an abd exam, UA and pelvic exam

37. all of the following can negatively impact perivaginal lactobacilli colonization except: a. recent antimicrobial use b. exposure to the spermicide nonoxynol-9 c. estrogen deficiency d. postcoital voiding

d. postcoital voiding

5. a 78-year-old male presents with a 3-day history of new onset fatigue & difficulty with bladder emptying. exam reveals distended bladder but otherwise unremarkable. BUN is 88; creatinine is 2.8. the most likely diagnosis is: a. prerenal azotemia b. acute glumerulonephritis c. tubular necrosis d. postrenal azotemia

d. postrenal azotemia

14. which of these most strongly reflects the theory behind natural family planning? a. pregnancy is less likely when the cervical mucus is thin b. pregnancy is less likely when the female's temperature drops and rises prior to ovulation c. pregnancy is less likely when there are "strawberry patches" on the cervix d. pregnancy is less likely when the female is lactating

d. pregnancy is less likely when the female is lactating lactational amenorrhea holds that pregnancy is less likely when female is not menstruating and fully breast feeding her infant.

57. all of the following can cause an elevated PSA level except: a. current prostate infection b. recent cystoscopy c. BPH d. prostatectomy

d. prostatectomy

79. all of the following are likely reported in a man with an initial episode of genital HSV-2 infection except: a. painful ulcer b. inguinal lymphadenopathy c. fever and bodyaches d. pustular lesions

d. pustular lesions

90. all of the following are likely reported in a woman with an initial episode of genital HSV-2 infection except: a. painful ulcer b. inguinal lymphadenopathy c. thin vaginal discharge d. pustular lesions

d. pustular lesions

25. with the use of LNG-IUD, a form of long-acting reversible contraceptive (LARC), which one of the following is normally noted? a. endometrial hyperplasia b. hypermenorrhea c. increase in PID rates d. reduction in menstrual flow

d. reduction in menstrual flow

59. a 24-year-old man presents with sudden onset of left-sided scrotal pain. he reports having intermittent unilateral testicular pain in the past but not as severe as this current episode. confirmation of testicular torsion would include all of the following findings except: a. unilateral loss of the cremasteric reflex b. the affected testicle held higher in the scrotum c. testicular swelling d. relief of pain with scrotal elevation

d. relief of pain with scrotal elevation

3. a 35 year old pt being worked up for microscopic hematuria. all of the following are differential diagnoses of microscopic hematuria except: a. kidney stones b. bladder cancer c. acute pyelonephritis d. renal artery stenosis

d. renal artery stenosis narrowing of kidney arteries, usually in individuals older than 50 years and associated with atherosclerosis and hypertension. hematuria is not associated with this. evidence of blood in urine is seen in kidney stones, bladder cancer and acute pyelonephritis.

58. metformin is a good choice for many older adults with T2DM. however, while on metformin, the NP should carefully monitor for: a. hypoglycemia b. fluid retention c. lactic alkalosis d. renal dysfunction

d. renal dysfunction

60. you see a 51-year-old woman who is considering HT. she has a family hx of endometrial cancer, HLD, and VTE. you advise her on all of the following except: a. the use of progestin can minimize the risk of endometrial cancer for a woman on HT and who has not had a hysterectomy b. supplemental estrogen should be avoided in women who are at high risk of breast cancer or uterine cancer c. supplemental estrogen should be avoided in women who are at high risk of cardiovascular disease d. short-term studies demonstrate that oral HT is associated with lower tromboembolic risk than transdermal forms of HT

d. short-term studies demonstrate that oral HT is associated with lower tromboembolic risk than transdermal forms of HT

27. which of the following contraceptives almost always contains the chemicals nonoxynol-9 and octoxynol? a. sponge b. condoms c. diaphragms d. spermicides

d. spermicides destroys sperm cells. sponge has nonoxynol-9 but does not have octoxynol.

30. which tanner stage of breast development is characterized by the projection of the areola and the nipple as a secondary mound? a. tanner stage 1 b. tanner stage 2 c. tanner stage 3 d. tanner stage 4

d. tanner stage 4

103. which of the following is most consistent with a woman with PID? a. temp 99.6F, WBC 8000 b. temp 101.4, WBC 6000 c. temp 99, WBC 14000 d. temp 101.5, WBC 16000

d. temp 101.5, WBC 16000

85. which of the following is not a common risk factor for ED? a. diabetes mellitus b. hypertension c. cigarette smoking d. testosterone deficiency

d. testosterone deficiency

8. what is the effect of DRE on a male's PSA level if it is measured the same day as a DRE? a. there will be a change, but it is not predictable b. an increase in PSA will occur c. a decrease in PSA will occur d. the change is insignificant

d. the change is insignificant there is an inconsequential rise in PSA levels within 72 hours after DRE. should not prevent PSA level measured at any time.

59. when counseling a 46-year-old woman who is experiencing debilitating hot flashes, you advise all of the following regarding higher and lower dose hormone replacement therapy except: a. current clinical guidelines recommend using the lowest effective dose possible b. higher dose HT will relieved hot flashes faster than lower dose regimens c. lower dose HT is better tolerated than higher dose HT d. the duration of lower dose HT is usually shorter than that of higher dose regimens

d. the duration of lower dose HT is usually shorter than that of higher dose regimens

69. which of the following statements regarding HIV is correct? a. CD4 counts are the first abnormality seen in patients with HIV b. a normal CD4 count is < 200 c. CD4 cell counts vary greatly in immunocompetent patients d. there are few conditions that cause depletion of CD4 cells other than HIV

d. there are few conditions that cause depletion of CD4 cells other than HIV

Which test will confirm the diagnosis of prostate cancer?

d. tissue biopsy Biopsy and pathology exam of prostate tissue is used to confirm a diagnosis of prostate cancer. CBC is not used to diagnose prostate cancer. A mass can be palpated with DRE; however, this does not confirm a diagnosis of prostate cancer. PSA is a screening test for prostate cancer, but it is not used to confirm final diagnosis.

125. which oral abx will the NP rx for 18-month-old infant with symptomatic UTI? a. ampicillin b. ceftriaxone c. ciprofloxacin d. trimethoprim-sulfamethoxazole

d. trimethoprim-sulfamethoxazole children < 24mos should get empiric tx of UTI. this is the first-line choice including first-generation cephalosporins, augmentin (but this may be avoided due to resistance to amoxicillin). top choice for empiric IV/IM is ceftriaxone. fluoroquinolones are only approved for complicated UTI (pyelonephritis) and for post exposure PPX and tx of inhalation anthrax in individuals < 18yrs

2. all of the following are common precipitating factors in acute renal failure except: a. anaphylaxis b. infection c. myocardial infarction d. type 1 diabetes

d. type 1 diabetes

40. all of the following s/s are common indicators of an initial outbreak of herpes EXCEPT: a. fever b. dysuria c. malaise d. urinary frequency

d. urinary frequency more common with gonorrhea than herpes. dysuria and urinary retention are common indications of initial outbreak of herpes, and fever & malaise.

7. a 38-year-old nulliparous woman who smokes 2.5 packs per day is in an "on & off" relationship. the woman presents seeking contraception. which of the following represents the most appropriate method? a. contraceptive ring (NuvaRing) b. COC c. contraceptive patch (ortho evra) d. vaginal diaphragm

d. vaginal diaphragm

74. which of the following is not a normal finding in a woman during reproductive years? a. vaginal pH ≤ 4.5 b. lactobacillus as the predominant vaginal organism c. thick, white vaginal secretions during the luteal phase 4.

d. vaginal epithelial cells with adherent bacteria

27. of all these treatments for herpes, which is most useful for asymptomatic viral shedding of HSV-2? a. calamine lotion b. adefovir c. colloidal oatmeal baths d. valacyclovir

d. valacyclovir adefovir used to treat HBV. calamine lotion and colloidal ointment baths are often given for symptomatic tx of HSV2, not to reduce asymptomatic viral shedding.

all of the following would require the NP to delay testing a patient's PSA except: a. urinary retention b. UTI c. after a DRE d. vigorous exercise 3 days prior

d. vigorous exercise 3 days prior others can falsely elevate discourage vigorous exercise right before testing because it will falsely elevate ejaculation within 48 hours will also falsely raise PSA level

17. the risk of HIV transmission is increased: a. in pts with diabetes b. when pts are aware of their HIV status c. in females d. when other STDs are present

d. when other STDs are present

86. a patient who is 62yo asks if she can get the shingles vaccine. she has never had shingles but states that she wants to make sure she doesnt get it. what should the NP advise? a. the immunization will protect you from acquiring shingles b. you are not old enough to receive the immunization c. the immunization is offered only to those who have had shingles d. you are eligible to receive it but you still may get shingles

d. you are eligible to receive it but you still may get shingles

19. which of the following meds should be avoided in a 65yo male with BPH?

diphenhydramine

13. what is the recommendation of the american cancer society for initial screening of an african-american male for prostate cancer?

discussion starting at age 40-45yrs

35. T/F, nausea with COC use can be minimized by taking the pill on an empty stomach.

false

25. a 25yo male pt is training for a marathon, he reports acute onset of scrotal pain after a 10mile run he has nausea and is found to have an asymmetric, high-riding testes on the right side. what should be suspected?

testicular torsion

36. T/F, the calcium and vitamin D supplementation is recommended for those taking DMPA injections to minimize the risk of a loss in bone density.

true

24. what choice below has no precautions for oral contraceptive pill use?

varicose veins

28. is scrotal swelling present in epididymitis?

yes


Conjuntos de estudio relacionados

US History Ch. 2 Study Guide: Transplantations and Borderlands

View Set

Fighting for Equality Unit Test 100%

View Set

EMS Chapter 17 - Neurologic Emergencies

View Set

PNE 104. Ch. 42 - Antidiabetic Drugs. Clinical Pharmacology. Susan Ford 11th. ED.

View Set